You are on page 1of 199

1

TNG HP 63 THI TUYN SINH LP 10 TRONG TON QUC


NM HC 2012 2013
MN TON

S GIO DC V O TO K THI TUYN SINH LP 10 THPT
H NI Nm c: 2012 2013

Mn thi: Ton
Ngy thi: 21 thng 6 nm 2012
Thi gian lm bi: 120 pht
Bi I (2,5 im)
1) Cho biu thc
x 4
A
x 2
+
=
+
. Tnh gi tr ca A khi x = 36
2) Rt gn biu thc
x 4 x 16
B :
x 4 x 4 x 2
| |
+
= +
|
|
+ +
\ .
(vi x 0; x 16 > = )
3) Vi cc ca biu thc A v B ni trn, hy tm cc gi tr ca x nguyn gi tr ca biu thc B(A
1) l s nguyn
Bi II (2,0 im). Gii bi ton sau bng cch lp phng trnh hoc h phng trnh:
Hai ngi cng lm chung mt cng vic trong
12
5
gi th xong. Nu mi ngi lm mt mnh th
ngi th nht hon thnh cng vic trong t hn ngi th hai l 2 gi. Hi nu lm mt mnh th mi ngi
phi lm trong bao nhiu thi gian xong cng vic?
Bi III (1,5 im)
1) Gii h phng trnh:
2 1
2
x y
6 2
1
x y

+ =


2) Cho phng trnh: x
2
(4m 1)x + 3m
2
2m = 0 (n x). Tm m phng trnh c hai nghim
phn bit x
1
, x
2
tha mn iu kin :
2 2
1 2
x x 7 + =
Bi IV (3,5 im)
Cho ng trn (O; R) c ng knh AB. Bn knh CO vung gc vi AB, M l mt im bt k
trn cung nh AC (M khc A, C); BM ct AC ti H. Gi K l hnh chiu ca H trn AB.
1) Chng minh CBKH l t gic ni tip.
2) Chng minh ACM ACK =
3) Trn an thng BM ly im E sao cho BE = AM. Chng minh tam gic ECM l tam gic vung
cn ti C
4) Gi d l tip tuyn ca (O) ti im A; cho P l im nm trn d sao cho hai im P, C nm trong
cng mt na mt phng b AB v
AP.MB
R
MA
= . Chng minh ng thng PB i qua trung im ca on
thng HK
Bi V (0,5 im). Vi x, y l cc s dng tha mn iu kin x 2y > , tm gi tr nh nht ca biu thc:
2 2
x y
M
xy
+
=



CHNH THC

2
GI P N

I: 25 m
1) Vi x = 36, ta c : A =
36 4 10 5
8 4 36 2
+
= =
+

2) Vi x >, x = 16 ta c :
B =
x( x 4) 4( x 4) x 2
x 16 x 16 x 16
| |
+ +
+
|
|
+
\ .
=
(x 16)( x 2) x 2
(x 16)(x 16) x 16
+ + +
=
+

3) Ta c:
2 4 2 2 2
( 1) . 1 .
16 16 16
2 2
x x x
B A
x x x
x x
| |
+ + +
= = =
|
|

+ +
\ .
.
( 1) B A nguyn, x nguyn th 16 x l c ca 2, m (2) =
} {
1; 2
Ta c bng gi tr tng ng:
16 x 1
1 2
2
x 17 15 18 14
Kt hp K 0, 16 x x > = , ( 1) B A nguyn th
} {
14; 15; 17; 18 xe

II: 20 m
Gi thi gian ngi th nht hon thnh mt mnh xong cng vic l x (gi), K
12
5
x >
Th thi gian ngi th hai lm mt mnh xong cng vic l x + 2 (gi)
Mi gi ngi th nht lm c
1
x
(cv), ngi th hai lm c
1
2 x +
(cv)
V c hai ngi cng lm xong cng vic trong
12
5
gi nn mi gi c hai i lm c
12
1:
5
=
5
12
(cv)
Do ta c phng trnh

1 1 5
x x 2 12
+ =
+

2 5
( 2) 12
x x
x x
+ +
=
+

5x
2
14x 24 = 0
A = 49 + 120 = 169,
,
13 A =
=>

= =
7 13 6
5 5
x (loi) v
+
= = =
7 13 20
4
5 5
x (TMK)
Vy ngi th nht lm xong cng vic trong 4 gi,
ngi th hai lm xong cng vic trong 4+2 = 6 gi.
III: 15 m 1)Gii h:
2 1
2
6 2
1
x y
x y

+ =

, (K: , 0 x y = ).
H
4 2
4 6 10
4 2 4 1 5
2
2 1
2 1 2 1 2 6 2 1
2 2
1 2
x
x x y
x x x
y
y
x y x y
x y


+ = = + = + =


=


+ = =


+ = + =
=


.(TMK)

3
Vy h c nghim (x;y)=(2;1).

2) + Phng trnh cho c A = (4m 1)
2
12m
2
+ 8m = 4m
2
+ 1 > 0, m
Vy phng trnh c 2 nghim phn bit m
+ Theo L Vi t, ta c:
1 2
2
1 2
4 1
3 2
x x m
x x m m
+ =

.
Khi :
2 2 2
1 2 1 2 1 2
7 ( ) 2 7 x x x x x x + = + =
(4m 1)
2
2(3m
2
2m) = 7 10m
2
4m 6 = 0 5m
2
2m 3 = 0
Ta thy tng cc h s: a + b + c = 0 => m = 1 hay m =
3
5

.
Tr li: Vy....



IV: 35 m












1) Ta c
0
90 HCB = ( do chn na ng trn k AB)
0
90 HKB = (do K l hnh chiu ca H trn AB)
=>
0
180 HCB HKB + = nn t gic CBKH ni tip trong ng trn ng knh HB.
2) Ta c ACM ABM = (do cng chn AM ca (O))
v ACK HCK HBK = = (v cng chn HK .ca trn k HB)
Vy ACM ACK =
3) V OC AB nn C l im chnh gia ca cung AB AC = BC v
0
90 sd AC sdBC = =
Xt 2 tam gic MAC v EBC c
MA= EB(gt), AC = CB(cmt) v MAC = MBC v cng chn cung MC ca (O)
MAC v EBC (cgc) CM = CE tam gic MCE cn ti C (1)
Ta li c
0
45 CMB = (v chn cung
0
90 CB = )
.
0
45 CEM CMB = = (tnh cht tam gic MCE cn ti C)
M
0
180 CME CEM MCE + + = (Tnh cht tng ba gc trong tam gic)
0
90 MCE = (2)
T (1), (2) tam gic MCE l tam gic vung cn ti C (pcm).





A
B
C
M

H
K
O
E

4

















4) Gi S l giao im ca BM v ng thng (d), N l giao im ca BP vi HK.
Xt APAM v A OBM :
Theo gi thit ta c
. AP MB AP OB
R
MA MA MB
= = (v c R = OB).
Mt khc ta c PAM ABM = (v cng chn cung AM ca (O))
APAM A OBM
= = = 1
AP OB
PA PM
PM OM
.(do OB = OM = R) (3)
V =
0
90 AMB (do chn na trn(O)) =
0
90 AMS
tam gic AMS vung ti M. + =
0
90 PAM PSM
v + =
0
90 PMA PMS = = PMS PSM PS PM (4)
M PM = PA(cmt) nn = PAM PMA
T (3) v (4) PA = PS hay P l trung im ca AS.
V HK//AS (cng vung gc AB) nn theo L Ta-lt, ta c: = =
NK BN HN
PA BP PS
hay =
NK HN
PA PS

m PA = PS(cmt) = NK NH hay BP i qua trung im N ca HK. (pcm)

V: 05 m

Cch 1(khng s dng BT C Si)
Ta c M =
2 2 2 2 2 2 2
( 4 4 ) 4 3 ( 2 ) 4 3 x y x xy y xy y x y xy y
xy xy xy
+ + + +
= = =
2
( 2 ) 3
4
x y y
xy x

+

V (x 2y)
2
0, du = xy ra x = 2y
x 2y
1 3 3
2 2
y y
x x

s > , du = xy ra x = 2y
T ta c M 0 + 4 -
3
2
=
5
2
, du = xy ra x = 2y
Vy GTNN ca M l
5
2
, t c khi x = 2y
Cch 2:
A
B
C
M

H
K
O
S
P
E
N

5
Ta c M =
2 2 2 2
3
( )
4 4
x y x y x y x y x
xy xy xy y x y x y
+
= + = + = + +
V x, y > 0 , p dng bdt C si cho 2 s dng ;
4
x y
y x
ta c 2 . 1
4 4
x y x y
y x y x
+ > = ,
du = xy ra x = 2y
V x 2y
3 6 3
2 .
4 4 2
x x
y y
> > = , du = xy ra x = 2y
T ta c M 1 +
3
2
=
5
2
, du = xy ra x = 2y
Vy GTNN ca M l
5
2
, t c khi x = 2y
Cch 3:

Ta c M =
2 2 2 2
4 3
( )
x y x y x y x y y
xy xy xy y x y x x
+
= + = + = +
V x, y > 0 , p dng bdt C si cho 2 s dng
4
;
x y
y x
ta c
4 4
2 . 4
x y x y
y x y x
+ > = ,
du = xy ra x = 2y
V x 2y
1 3 3
2 2
y y
x x

s > , du = xy ra x = 2y
T ta c M 4-
3
2
=
5
2
, du = xy ra x = 2y
Vy GTNN ca M l
5
2
, t c khi x = 2y
Cch 4:
Ta c M =
2 2 2 2 2
2 2 2 2
2 2 2
4 3
3 3
4 4 4 4 4
4 4
x x x x x
y y y y
x y x x
xy xy xy xy xy xy y
+ + + + +
+
= = = + = +
V x, y > 0 , p dng bdt Co si cho 2 s dng
2
2
;
4
x
y ta c
2 2
2 2
2 .
4 4
x x
y y xy + > = ,
du = xy ra x = 2y
V x 2y
3 6 3
2 .
4 4 2
x x
y y
> > = , du = xy ra x = 2y
T ta c M
xy
xy
+
3
2
= 1+
3
2
=
5
2
, du = xy ra x = 2y
Vy GTNN ca M l
5
2
, t c khi x = 2y






S GIO DC V O TO K THI TUYN SINH LP 10 THPT
TP.HCM Nm c: 2012 2013
CHNH THC

6
MN: TON
Thi gian lm bi: 120 pht
1: 2 m
Gii cc phng trnh v h phng trnh sau:
a)
2
2 3 0 = x x
b)
2 3 7
3 2 4
=

+ =

x y
x y

c)
4 2
12 0 + = x x
d)
2
2 2 7 0 = x x
2: 15 m
a) V th (P) ca hm s
2
1
4
= y x v ng thng (D):
1
2
2
= + y x trn cng mt h trc to .
b) Tm to cc giao im ca (P) v (D) cu trn bng php tnh.
3: 15 m
Thu gn cc biu thc sau:
1 2 1
1
= +
+
x
A
x x x x x
vi x > 0; 1 = x
(2 3) 26 15 3 (2 3) 26 15 3 = + + B
4: 15 m
Cho phng trnh
2
2 2 0 + = x mx m (x l n s)
a) Chng minh rng phng trnh lun lun c 2 nghim phn bit vi mi m.
b) Gi x
1
, x
2
l cc nghim ca phng trnh.
Tm m biu thc M =
2 2
1 2 1 2
24
6

+ x x x x
t gi tr nh nht
5: 35 m
Cho ng trn (O) c tm O v im M nm ngoi ng trn (O). ng thng MO ct (O) ti E
v F (ME<MF). V ct tuyn MAB v tip tuyn MC ca (O) (C l tip im, A nm gia hai im M
v B, A v C nm khc pha i vi ng thng MO).
a) Chng minh rng MA.MB = ME.MF
b) Gi H l hnh chiu vung gc ca im C ln ng thng MO. Chng minh t gic AHOB
ni tip.
c) Trn na mt phng b OM c cha im A, v na ng trn ng knh MF; na ng
trn ny ct tip tuyn ti E ca (O) K. Gi S l giao im ca hai ng thng CO v KF.
Chng minh rng ng thng MS vung gc vi ng thng KC.
d) Gi P v Q ln lt l tm ng trn ngoi tip cc tam gic EFS v ABS v T l trung im
ca KS. Chng minh ba im P, Q, T thng hng.

BI GII
1: 2 m
Gii cc phng trnh v h phng trnh sau:
a)
2
2 3 0 = x x (a)
V phng trnh (a) c a - b + c = 0 nn
(a)
3
1
2
= = x hay x
b)
2 3 7 (1)
3 2 4 (2)
=

+ =

x y
x y

2 3 7 (1)
5 3 (3) ((2) (1) )
=

+ =

x y
x y



7

13 13 ((1) 2(3))
5 3 (3) ((2) (1) )
=

+ =

y
x y


1
2
=

y
x

c)
4 2
12 0 + = x x (C)
t u = x
2
> 0, phng trnh thnh : u
2
+ u 12 = 0 (*)
(*) c A = 49 nn (*)
1 7
3
2
+
= = u hay
1 7
4
2

= = u (loi)
Do , (C) x
2
= 3 x = 3
Cch khc : (C) (x
2
3)(x
2
+ 4) = 0 x
2
= 3 x = 3

d)
2
2 2 7 0 = x x (d)
A = 2 + 7 = 9 do (d) x = 2 3

2:
a) th:











Lu : (P) i qua O(0;0),
( ) ( ) 2;1 , 4; 4
(D) i qua
( ) ( ) 4;4 , 2;1
b) PT honh giao im ca (P) v (D) l
2
1 1
2
4 2
= + x x x
2
+ 2x 8 = 0 4 2 = = x hay x
y(-4) = 4, y(2) = 1
Vy to giao im ca (P) v (D) l
( ) ( ) 4;4 , 2;1 .
B 3:Thu gn cc biu thc sau:
1 2 1
1
= +
+
x
A
x x x x x

2
2
1

= +

x x x x x
x x x


2 2
( 1) 1

= +

x x
x x x
2 1
1
1
(
= +
(


x
x x

2 ( 1)
( 1)

x x
x x

2
=
x
vi x > 0; 1 = x
(2 3) 26 15 3 (2 3) 26 15 3 = + + B
1 1
(2 3) 52 30 3 (2 3) 52 30 3
2 2
= + +
2 2
1 1
(2 3) (3 3 5) (2 3) (3 3 5)
2 2
= + +

8
M
E
F
K
S A
B
T
P
Q
C
H
O
V
1 1
(2 3)(3 3 5) (2 3)(3 3 5) 2
2 2
= + + =

Cu 4:
a/ Phng trnh (1) c = m
2
- 4m +8 = (m - 2)
2
+4 > 0 vi mi m nn phng trnh (1) c 2 nghim phn
bit vi mi m.
b/ Do , theo Viet, vi mi m, ta c: S = 2
b
m
a
= ; P = 2 =
c
m
a

M =
2
1 2 1 2
24
( ) 8

+ x x x x
=
2 2
24 6
4 8 16 2 4

=
+ + m m m m

2
6
( 1) 3

=
+ m
. Khi m = 1 ta c
2
( 1) 3 + m nh nht
2
6
( 1) 3
=
+
M
m
ln nht khi m = 1
2
6
( 1) 3

=
+
M
m
nh nht khi m = 1
Vy M t gi tr nh nht l - 2 khi m = 1

Cu 5
a) V ta c do hai tam gic ng dng MAE v MBF
Nn
MA MF
ME MB
= MA.MB = ME.MF
(Phng tch ca M i vi ng trn tm O)
b) Do h thc lng trong ng trn ta c
MA.MB = MC
2
, mt khc h thc lng
trong tam gic vung MCO ta c
MH.MO = MC
2
MA.MB = MH.MO
nn t gic AHOB ni tip trong ng trn.
c) Xt t gic MKSC ni tip trong ng
trn ng knh MS (c hai gc K v C vung).
Vy ta c : MK
2
= ME.MF = MC
2
nn MK = MC.
Do MF chnh l ng trung trc ca KC
nn MS vung gc vi KC ti V.
d) Do h thc lng trong ng trn ta c MA.MB = MV.MS ca ng trn tm Q.
Tng t vi ng trn tm P ta cng c MV.MS = ME.MF nn PQ vung gc vi MS v l ng trung
trc ca VS (ng ni hai tm ca hai ng trn). Nn PQ cng i qua trung im ca KS (do nh l trung
bnh ca tam gic SKV). Vy 3 im T, Q, P thng hng.








S GIO DC V O TO K THI TUYN SINH LP 10 THPT
TP. NNG Nm c: 2012 2013
MN: TON
Thi gian lm bi: 120 pht

Bi 1: (2,0 im)
CHNH THC

9
1) Gii phng trnh: (x + 1)(x + 2) = 0
2) Gii h phng trnh:
2 1
2 7
+ =

x y
x y

Bi 2: (1,0 im)
Rt gn biu thc ( 10 2) 3 5 = + A
Bi 3: (1,5 im)
Bit rng ng cong trong hnh v bn l mt parabol y = ax
2
.
1) Tm h s a.
2) Gi M v N l cc giao im ca ng thng
y = x + 4 vi parabol. Tm ta ca cc im M v N.
Bi 4: (2,0 im)
Cho phng trnh x
2
2x 3m
2
= 0, vi m l tham s.
1) Gii phng trnh khi m = 1.
2) Tm tt c cc gi tr ca m phng trnh c hai nghim x
1
, x
2
khc 0 v tha iu kin
1 2
2 1
8
3
=
x x
x x
.
Bi 5: (3,5 im)
Cho hai ng trn (O) v (O) tip xc ngoi ti A. K tip tuyn chung ngoi BC,B e (O),Ce(O).
ng thng BO ct (O) ti im th hai l D.
1) Ch`ng minh rng t gic COOB l mt hnh thang vung.
2) Chng minh rng ba im A, C, D thng hng.
3) T D k tip tuyn DE vi ng trn (O) (E l tip im). Chng minh rng DB = DE.

BI GII
Bi 1:
1) (x + 1)(x + 2) = 0 x + 1 = 0 hay x + 2 = 0 x = -1 hay x = -2
2)
2 1 (1)
2 7 (2)
+ =

x y
x y

5y 15 ((1) 2(2))
x 7 2y
=

= +


y 3
x 1
=



Bi 2: ( 10 2) 3 5 = + A = ( 5 1) 6 2 5 + =
2
( 5 1) ( 5 1) + = ( 5 1)( 5 1) + = 4
Bi 3:
1) Theo th ta c y(2) = 2 2 = a.2
2
a =
2) Phng trnh honh giao im ca y =
2
1
2
x v ng thng y = x + 4 l :
x + 4 =
2
1
2
x x
2
2x 8 = 0 x = -2 hay x = 4
y(-2) = 2 ; y(4) = 8. Vy ta cc im M v N l (-2 ; 2) v (4 ; 8).
Bi 4:
1) Khi m = 1, phng trnh thnh : x
2
2x 3 = 0 x = -1 hay x = 3 (c dng ab + c = 0)
2) Vi x
1
, x
2
= 0, ta c :
1 2
2 1
8
3
=
x x
x x

2 2
1 2 1 2
3( ) 8 = x x x x 3(x
1
+ x
2
)(x
1
x
2
) = 8x
1
x
2

Ta c : a.c = -3m
2
s 0 nn A > 0, m
Khi A > 0 ta c : x
1
+ x
2
= 2 =
b
a
v x
1
.x
2
=
2
3 =
c
m
a
s 0
iu kin phng trnh c 2 nghim = 0 m m = 0 A > 0 v x
1
.x
2
< 0 x
1
< x
2

0
1 2
2
y=ax
2

y
x

10
B
C
E
D
A
O
O
Vi a = 1 x
1
= ' ' A b v x
2
= ' ' + A b x
1
x
2
=
2
2 ' 2 1 3 A = + m
Do , ycbt
2 2
3(2)( 2 1 3 ) 8( 3 ) + = m m v m = 0

2 2
1 3 2 + = m m (hin nhin m = 0 khng l nghim)
4m
4
3m
2
1 = 0 m
2
= 1 hay m
2
= -1/4 (loi) m = 1
Bi 5:













1) Theo tnh cht ca tip tuyn ta c OB, OC vung gc vi BC t gic COOB l hnh thang
vung.
2) Ta c gc ABC = gc BDC gc ABC + gc BCA = 90
0
gc BAC = 90
0

Mt khc, ta c gc BAD = 90
0
(ni tip na ng trn)
Vy ta c gc DAC = 180
0
nn 3 im D, A, C thng hng.
3) Theo h thc lng trong tam gic vung DBC ta c DB
2
= DA.DC
Mt khc, theo h thc lng trong ng trn (chng minh bng tam gic ng dng) ta c DE
2
=
DA.DC DB = DE.


















S GD&T
VNH PHC

K THI TUYN SINH LP 10 THPT NM HC 2012-2013
THI MN : TON
T gan lm b 120 pt kng k t gan gao
Ngy thi: 21 thng 6 nm 2012

CHNH THC

11
Cu 1 (2,0 im). Cho biu thc :P=
2
3 6 4
1 1 1
x x
x x x

+
+

1. Tm iu kin xc nh ca biu thc P.
2. Rt gn P
Cu 2 (2,0 im). Cho h phng trnh :
2 4
ax 3 5
x ay
y
+ =


1. Gii h phng trnh vi a=1
2. Tm a h phng trnh c nghim duy nht.
Cu 3 (2,0 im). Mt hnh ch nht c chiu rng bng mt na chiu di. Bit rng nu gim mi chiu i
2m th din tch hnh ch nht cho gim i mt na. Tnh chiu di hnh ch nht cho.
Cu 4 (3,0 im). Cho ng trn (O;R) (im O c nh, gi tr R khng i) v im M nm bn ngoi
(O). K hai tip tuyn MB, MC (B,C l cc tip im ) ca (O) v tia Mx nm gia hai tia MO v MC. Qua
B k ng thng song song vi Mx, ng thng ny ct (O) ti im th hai l A. V ng knh BB ca
(O). Qua O k ng thng vung gc vi BB,ng thng ny ct MC v BC ln lt ti K v E. Chng
minh rng:
1. 4 im M,B,O,C cng nm trn mt ng trn.
2. on thng ME = R.
3. Khi im M di ng m OM = 2R th im K di ng trn mt ng trn c nh, ch r tm v bn
knh ca ng trn .
Cu 5 (1,0 im). Cho a,b,c l cc s dng tha mn a+ b + c =4. Chng minh rng :
3 3 3 4 4 4
2 2 a b c + + >
S GD&T VNH PHC


K THI TUYN SINH LP 10 THPT NM HC 2012-2013
P N THI MN : TON
Ngy thi: 21 thng 6 nm 2012
Cu p n, gi im
C1.1
(0,75
im)
Biu thc P xc nh

=
= +
=

0 1
0 1
0 1
2
x
x
x

=
=

1
1
x
x


0,5


0,25
C1.2
(1,25
im)
P=
) 1 )( 1 (
) 4 6 ( ) 1 ( 3 ) 1 (
) 1 )( 1 (
4 6
1
3
1 +
+ +
=
+

+
+
x x
x x x x
x x
x
x x
x


) 1 (
1
1
) 1 )( 1 (
) 1 (
) 1 )( 1 (
1 2
) 1 )( 1 (
4 6 3 3
2
2 2
=
+

=
+

=
+
+
=
+
+ + +
=
x voi
x
x
x x
x
x x
x x
x x
x x x x

0,25

0,5

0,5

12
C2.1
(1,0
im)
Vi a = 1, h phng trnh c dng:

=
= +
5 3
4 2
y x
y x

=
=

=
=

=
=

=
= +

2
1
5 3 1
1
5 3
7 7
5 3
12 3 6
y
x
y
x
y x
x
y x
y x

Vy vi a = 1, h phng trnh c nghim duy nht l:

=
=
2
1
y
x

0,25


0,25

0,25

0,25
C2.2
(1,0
im)
-Nu a = 0, h c dng:

=
=

=
=
3
5
2
5 3
4 2
y
x
y
x
=> c nghim duy nht
-Nu a 0 = , h c nghim duy nht khi v ch khi:
3
2

=
a
a

6
2
= a (lun ng, v 0
2
> a vi mi a)
Do , vi a 0 = , h lun c nghim duy nht.
Vy h phng trnh cho c nghim duy nht vi mi a.
0,25



0,25

0,25

0,25
C3 (2,0
im)
Gi chiu di ca hnh ch nht cho l x (m), vi x > 4.
V chiu rng bng na chiu di nn chiu rng l:
2
x
(m)
=> din tch hnh ch nht cho l:
2 2
.
2
x x
x = (m
2
)
Nu gim mi chiu i 2 m th chiu di, chiu rng ca hnh ch nht ln lt
l: 2
2
2
x
va x (m)
khi , din tch hnh ch nht gim i mt na nn ta c phng trnh:
2 2
1
) 2
2
)( 2 (
2
x x
x =
0 16 12
4
4 2
2
2
2 2
= + = + x x
x
x x
x

.=> 5 2 6
1
+ = x (tho mn x>4);
5 2 6
2
= x (loi v khng tho mn x>4)
Vy chiu di ca hnh ch nht cho l 5 2 6 + (m).
0,25


0,25


0,25



0,25
0,25


0,5

0,25
C4.1
(1,0
im)
1) Chng minh M, B, O, C cng thuc 1 ng trn
Ta c:
0
90 = ZMOB (v MB l tip tuyn)
0
90 = ZMCO (v MC l tip tuyn)
=> ZMBO + ZMCO =
= 90
0
+ 90
0
= 180
0

=> T gic MBOC ni tip
(v c tng 2 gc i =180
0
)
=>4 im M, B, O, C cng thuc 1 ng trn


0,25

0,25
0,25

0,25
C4.2
(1,0
im)
2) Chng minh ME = R:
Ta c MB//EO (v cng vung gc vi BB)
=> ZO
1
= ZM
1
(so le trong)
M ZM
1
= ZM
2
(tnh cht 2 tip tuyn ct nhau) => ZM
2
= ZO
1
(1)



0,25
M
O
B
C
K
E
B
1
2 1
1

13
C/m c MO//EB (v cng vung gc vi BC)
=> ZO
1
= ZE
1
(so le trong) (2)
T (1), (2) => ZM
2
= ZE
1
=> MOCE ni tip
=> ZMEO = ZMCO = 90
0

=> ZMEO = ZMBO = ZBOE = 90
0
=> MBOE l hnh ch nht
=> ME = OB = R (iu phi chng minh)

0,25

0,25

0,25
C4.3
(1,0
im)
3) Chng minh khi OM=2R th K di ng trn 1 ng trn c nh:
Chng minh c Tam gic MBC u => ZBMC = 60
0

=> ZBOC = 120
0

=> ZKOC = 60
0
- ZO
1
= 60
0
- ZM
1
= 60
0
30
0
= 30
0

Trong tam gic KOC vung ti C, ta c:
3
3 2
2
3
:
30
0
R
R
Cos
OC
OK
OK
OC
CosKOC = = = =
M O c nh, R khng i => K di ng trn ng trn tm O, bn knh =
3
3 2 R
(iu phi chng minh)


0,25
0,25


0,25

0,25
C5 (1,0
im)
( ) ( ) ( )
3 3 3 4 4 4
3 3 3
4 4 4
4 4 4 4 4 4
4 4 4
4
a b c
a b c a a b c b a b c c
a b c
a b c
+ +
= + + + + + + + +
> + +
= + +
=

Do
3 3 3 4 4 4
4
4 4
2 2
4 2
a b c + + > = =

0,25

0,25

0,25

0,25
Ch : -Cu 4, tha gi thit tia Mx v im A gy ri.
-Mi cu u c cc cch lm khc
cu 5
Cach 2: t x =
4 4 4
= = a; y b;z c=> x, y , z > 0 v x
4
+ y
4
+ z
4
= 4.
BT cn CM tng ng: x
3
+ y
3
+ z
3
> 2 2
hay 2 (x
3
+ y
3
+ z
3
) > 4 = x
4
+ y
4
+ z
4

x
3
( 2 -x) + y
3
( 2 -y)+ z
3
( 2 -z) > 0 (*).
Ta xt 2 trng hp:
- Nu trong 3 s x, y, z tn ti it nht mt s 2 > , gi s x 2 > th x
3
2 2 > .
Khi o: x
3
+ y
3
+ z
3
> 2 2 ( do y, z > 0).
- Nu c 3 s x, y, z u nh 2 < th BT(*) lun ung.
Vy x
3
+ y
3
+ z
3
> 2 2 c CM.
Cach 3: C th dng BT thc Csi kt hp phng php lm tri v nh gi cng cho kt qu nhng hi
di, phc tp).

14
S GD V O TO K THI TUYN SINH VO 10 THPT NM HC 2012-2013
KLK MN THI : TON
Thi gian lm bi: 120 pht,(khng k giao )
Ngy thi: 22/06/2012


Cu 1. (2,5)
1) Gii phng trnh:
a) 2x
2
7x + 3 = 0. b) 9x
4
+ 5x
2
4 = 0.
2) Tm hm s y = ax + b, bit th hm s ca n i qua 2 im A(2;5) ; B(-2;-3).
Cu 2. (1,5)
1) Hai t i t A n B di 200km. Bit vn tc xe th nht nhanh hn vn tc xe th hai l 10km/h
nn xe th nht n B sm hn xe th hai 1 gi. Tnh vn tc mi xe.
2) Rt gn biu thc:
( )
1
A= 1 x x ;
x 1
| |
+
|
+ \ .
vi x 0.
Cu 3. (1,5 )
Cho phng trnh: x
2
2(m+2)x + m
2
+ 4m +3 = 0.
1) Chng minh rng : Phng trnh trn lun c hai nghim phn bit x
1
, x
2
vi mi gi tr ca m.
2) Tm gi tr ca m biu thc A =
2 2
1 2
x x + t gi tr nh nht.
Cu 4. (3,5)
Cho tam gic ABC c ba gc nhn ni tip ng trn tm O (AB < AC). Hai tip tuyn ti B v C ct nhau
ti M. AM ct ng trn (O) ti im th hai D. E l trung im on AD. EC ct ng trn (O) ti im
th hai F. Chng minh rng:
1) T gic OEBM ni tip.
2) MB
2
= MA.MD.
3) BFC MOC = .
4) BF // AM
Cu 5. (1)
Cho hai s dng x, y tha mn: x + 2y = 3. Chng minh rng:
1 2
3
x y
+ >
g s lc:
Cu 1. (2,5)
1) Gii phng trnh:
a) 2x
2
7x + 3 = 0.
A = (-7)
2
4.2.3 = 25 > 0
A = 5. Phng trnh c hai nghim phn bit:
1
2
7 5
x 3.
4
7 5 1
x
4 2
+
= =

= =

b) 9x
4
+ 5x
2
4 = 0. t x
2
= t , k : t 0.
Ta c pt: 9t
2
+ 5t 4 = 0.
a b + c = 0 t
1
= - 1 (khng TMK, loi)
t
2
=
4
9
(TMK)
t
2
=
4
9
x
2
=
4
9
x =
4 2
9 3
= .
CHNH THC

15
E
F
D
A
M
O
C
B
Vy phng trnh cho c hai nghim: x
1,2
=
2
3

2) th hm s y = ax + b i qua hai im A(2;5) v B(-2;-3)
2a b 5 a 2
2a b 3 b 1
+ = =


+ = =


Vy hm s cn tm l : y = 2x + 1
Cu 2.
1) Gi vn tc xe th hai l x (km/h). k: x > 0
Vn tc xe th nht l x + 10 (km/h)
Thi gian xe th nht i qung ng t A n B l :
200
x 10 +
(gi)
Thi gian xe th hai i qung ng t A n B l :
200
x
(gi)
Xe th nht n B sm 1 gi so vi xe th hai nn ta c phng trnh:
200 200
1
x x 10
=
+

Gii phng trnh ta c x
1
= 40 , x
2
= -50 ( loi)
x
1
= 40 (TMK). Vy vn tc xe th nht l 50km/h, vn tc xe th hai l 40km/h.
2) Rt gn biu thc:
( ) ( )
1 x 1 1
A 1 x x x x
x 1 x 1
| |
| | +
= + = + |
|
|
+ + \ .
\ .

=
( )
x
x x 1
x 1
| |
+ |
|
+
\ .
= x, vi x 0.
Cu 3. (1,5 )
Cho phng trnh: x
2
2(m+2)x + m
2
+ 4m +3 = 0.
1) Chng minh rng : Phng trnh trn lun c hai nghim phn bit x
1
, x
2
vi mi gi tr ca m.
Ta c
2
2
(m 2) m 4m 3 1 ' ( A = + =

> 0 vi mi m.
Vy phng trnh cho lun c hai nghim phn bit x
1
, x
2
vi mi gi tr ca m.
2) phng trnh cho lun c hai nghim phn bit x
1
, x
2
vi mi gi tr ca m. Theo h thc Vi-t ta
c :
1 2
2
1 2
x x 2(m 2)
x .x m 4m 3
+ = +

= + +


A =
2 2
1 2
x x + = (x
1
+ x
2
)
2
2 x
1
x
2
= 4(m + 2)
2
2(m
2
+ 4m +3) = 2m
2
+ 8m+ 10
= 2(m
2
+ 4m) + 10
= 2(m + 2)
2
+ 2 2 vi mi m.
Suy ra minA = 2 m + 2 = 0 m = - 2
Vy vi m = - 2 th A t min = 2
Cu 4.
1) Ta c EA = ED (gt) OE AD ( Quan h gia ng knh v dy)
OEM = 90
0
; OBM = 90
0
(Tnh cht tip tuyn)
E v B cng nhn OM di mt gc vung T gic OEBM ni tip.
2) Ta c
1
MBD
2
= s BD ( gc ni tip chn cung BD)

1
MAB
2
= s BD ( gc to bi tia tip tuyn v dy cung chn cung BD)
MBD MAB = . Xt tam gic MBD v tam gic MAB c:

16
Gc M chung, MBD MAB = MBD A ng dng vi MAB A
MB MD
MA MB
=
MB
2
= MA.MD
3) Ta c:
1
MOC
2
= BOC=
1
2
s BC ( Tnh cht hai tip tuyn ct nhau);
1
BFC
2
= s BC(gc ni
tip) BFC MOC = .
4) T gic MFOC ni tip ( F C + = 180
0
) MFC MOC = ( hai gc ni tip cng chn cung MC), mt
khc MOC BFC = (theo cu 3) BFC MFC = BF // AM.
Cu 5.
( )
2
2 2
a b
a b
x y x y
+
+ >
+

Ta c x + 2y = 3 x = 3 2y , v x dng nn 3 2y > 0
Xt hiu
1 2
3
x y
+ =
2
1 2 y 6 4y 3y(3 2y) 6(y 1)
3
3 2y y y(3 2y) y(3 2y)
+
+ = =

0 ( v y > 0 v 3 2y > 0)

1 1
3
x 2y
+ > du = xy ra
x 0,y 0 x 0,y 0
x 1
x 3 2y x 1
y 1
y 1 0 y 1
> > > >
=

= =

=

= =











17

S GIO DC VO O
TO HI DNG
-----------------

K THI TUYN SINH LP 10 THPT
NM HC 2012-2013
MN THI: TON
T gan lm bi 120 pht (khng k thi gian giao )

Ngy t: Ngy 12 tng 7 nm 2012
t gm: 01 trang
Cu 1 20 m:
Gii cc phng trnh sau:
a) x(x-2)=12-x.
b)
2
2
8 1 1
16 4 4
x
x x x

= +
+

Cu 2 20 m:
a) Cho h phng trnh
3 2 9
5
x y m
x y
+ = +

+ =

c nghim (x;y). Tm m biu thc (xy+x-1) t gii


tr ln nht.
b) Tm m ng thng y = (2m-3)x-3 ct trc honh ti im c honh bng
2
3
.
Cu 3 20 m:
a) Rt gn biu thc
( )
3 1
. 2
2 1
P x
x x x
| |
= +
|
+
\ .
vi 0 x > v 4 x = .
b) Nm ngoi, hai n v sn xut nng nghip thu hoch c 600 tn thc. Nm nay, n v
th nht lm vt mc 10%, n v th hai lm vt mc 20% so vi nm ngoi. Do c
hai n v thu hoch c 685 tn thc. Hi nm ngoi, mi n v thu hoch c bao nhiu
tn thc?
Cu 4 30 m:
Cho tam gic ABC c ba gc nhn, ni tip ng trn (O). V cc ng cao BE, CF ca
tam gic y. Gi H l giao im ca BE v CF. K ng knh BK ca (O) .
a) Chng minh t gic BCEF l t gic ni tip.
b) Chng minh t gic AHCK l mnh bnh hnh.
c) ng trn ng knh AC ct BE M, ng trn ng knh AB ct CF N. Chng minh
AM = AN.
Cu 5 10 m:
Cho a, b, c, d l cc s thc tha mn: b + d = 0 v 2
ac
b d
>
+
. Chng minh rng phng trnh
(x
2
+ ax +b)(x
2
+ cx + d)=0 (x l n) lun c nghim.
---------------------Ht--------------------








CHNH THC

18

HNG DN - P N
Cu 1: a ) x = - 3 v x = 4. b) x = - 2; loi x = 4.

Cu 2: a) H => x = m + 2 v y = 3 - m => A = (xy+x-1) = = 8 - ( m -1)
2

A
max

= 8 khi m = 1.
b) Thay x = 2/3 v y = 0 vo pt ng thng => m = 15/4

Cu 3: a) A = 1
b) x + y = 600 v 0,1x + 0,2y = 85 hay x + 2y = 850.
T tnh c y = 250 tn, x = 350 tn

Cu 4 30 m:
a)
0
90

= = C E B C F B

b) AH//KC ( cng vung gc vi BC)
CH // KA ( cng vung gc vi AB)

c) C AN
2
= AF.AB; AM
2
= AE.AC
( H thc lng trong tam gic vung)

AF
. AF.AB
AC
AE
AEF ABC AE AC
AB
A A = =

AM = AN
N
M
K
H
F
E
O
C B
A


Cu 5 10 m Xt 2 png trn:
x
2
+ ax + b = 0 (1) v x
2
+ cx + d = 0 (2)

| | | | ) ( 2 2 ) ( ) ( 2 2 2 ) 4 ( ) 4 (
2 2 2 2 2
2 1
d b ac c a d b ac c ac a d c b a + + = + + + = + = A + A

+ Vi b+d <0 b; d c t nht mt s nh hn 0

1
A >0 hoc
2
A >0 pt cho c nghim
+ Vi 0 > + d b
. T
2
ac
b d
>
+
ac > 2(b + d) => 0
2 1
> A + A
=> t nht mt trong hai biu gi tr
2 1
, A A 0 > => t nht mt trong hai pt (1) v (2) c
nghim.
Vy vi a, b, c, d l cc s thc tha mn: b + d = 0 v 2
ac
b d
>
+
,
phng trnh (x
2
+ ax +b)(x
2
+ cx + d)=0 (x l n) lun c nghim.








19

S GIO DC VO O
TO HI DNG
-----------------

K THI TUYN SINH LP 10 THPT
NM HC 2012-2013
MN THI: TON
T gan lm b 120 pt khng k thi gian giao )

Ngy thi: Ngy 14 tng 7 nm 2012
t gm: 01 trang
Cu 1 20 m: Gii cc phng trnh sau:
a)
2 4
5 3 0
3 5
x x
| || |
+ =
| |
\ .\ .

b) | 2x 3 | = 1.
Cu 2 20 m: Cho biu thc:
A = :
2
a a a a
b a a b a b a b ab
| | | |
+ +
| |
+ + + +
\ . \ .
vi a v b l cc s dng khc nhau.
a) Rt gn biu thc A
2 a b ab
b a
+ +

.
b) Tnh gi tr ca A khi a = 7 4 3 v b = 7 4 3 + .
Cu 3 20 m:
a) Tm m cc ng thng y = 2x + m v y = x 2m + 3 ct nhau ti mt im nm
trn trc tung.
b) Cho qung ng t a im A ti a im B di 90 km. Lc 6 gi mt xe my i t
A ti B Lc 6 gi 30 pht cng ngy, mt t cng i t A ti B vi vn tc ln hn vn tc
xe my 15 km/h (Hai xe chy trn cng mt con ng cho). Hai xe ni trn u n B cng lc.
Tnh vn tc mi xe.
Cu 4 30 m: Cho na ng trn tm O ng knh AB = 2R (R l mt di cho trc). Gi C, D
l hai im trn na ng trn sao cho C thuc cung AD v COD = 120
0
. Gi giao im ca hai dy
AD v BC l E, giao im ca cc ng thng AC v BD l F.
a) Chng minh rng bn im C, D, E, F cng nm trn mt ng trn.
b) Tnh bn knh ca ng trn i qua C, E, D, F ni trn theo R.
c) Tm gi tr ln nht ca in tch tam gic FAB theo R khi C, D thay i nhung vn tha mn gi
thit bi ton
Cu 5 10 m: Khng dng my tnh cm tay , tm s nguyn ln nht khng vt qu S, trong
S =
( )
6
2 3 +
-------------------- Ht --------------------


HNG DN GII .
Cu 1.

CHNH THC

20
2 4
) 5 3 0
3 5
2 15
5 0
2 15
3 2
4 4 15 15
3 0
5 4
a x x
x x
x
x
x x
| || |
+ =
| |
\ .\ .

= =

=
=>

=

+ = =



Vy nghim ca phng trnh cho l S = {
15 15
;
2 4

} b)
2 3 1 2 4 2
2 3 1
2 3 1 2 2 1
x x x
x
x x x
= = =
= =>

= = =


Vy nghim ca phng trnh cho l S = {1;2}
Cu 2 .
Ta c :
( )
( )
( )
2
2
2
:
2
:
( )( )
( ) ( )
:
( )( )
.
( )( )
a a a a
A
b a a b a b a b ab
a a a a
A
a b b a b a a b
a b
a b a a a a b a
A
b a b a
a b
a b
ab
A
b a b a ab
a b
A
b a
| | | |
= +
| |
| |
+ + + +
\ . \ .
(
| |
(
= +
|
| (
+ + +
\ . +
(

+ +
=
+
+
+
=
+
+
=


a) Ta c :
2
2 2
2
( )
( ) ( )
0
a b ab
A
b a
a b a b
b a b a
a b a b
b a
+ +

+ +
=

+ +
= =


Vy
2 a b ab
A
b a
+ +

= 0

b) Ta c :

( )
2
7 4 3
4 4 3 3
2 3
2 3
a
a
a
a
=
= +
=
=> =


21
( )
2
7 4 3
4 4 3 3
2 3
2 3
b
b
b
b
= +
= + +
= +
=> = +

Thay 2 3; 2 3 a b = = + vo biu thc
a b
A
b a
+
=

ta c :
2 3 2 3
2 3 2 3
4
2 3
2 3
3
A
A
A
+ +
=
+ +
=
=
Vy vi a = 7 - 4 3 ; b = 7 + 4 3 th A =
2 3
3
.

Cu 3 .
a) hai ng thng y = 2x + m v y = x 2m + 3 ct nhau ti mt im trn trc tung th m = -2m + 3 =>
3m = 3 => m = 1.
Vy vi m = 1 th hai ng thng y = 2x + m v y = x 2m + 3 ct nhau ti mt im trn trc tung.
b) Xe my i trc t thi gian l : 6 gi 30 pht - 6 gi = 30 pht =
1
2
h .
Gi vn tc ca xe my l x ( km/h ) ( x > 0 )
V vn tc t ln hn vn tc xe my 15 km/h nn vn tc ca t l x + 15 (km/h)
Thi gian xe my i ht qung ng AB l :
90
( ) h
x

Thi gian t i ht qung ng AB l :
90
( )
15
h
x +

Do xe my i trc t
1
2
gi v hai xe u ti B cng mt lc nn ta c phng trnh :
2
2
90 1 90
2 15
90.2.( 15) ( 15) 90.2
180 2700 15 180
15 2700 0
x x
x x x x
x x x x
x x
=
+
=> + + =
+ =
+ =

Ta c :
2
15 4.( 2700) 11025 0
11025 105
A = = >
A = =

1
15 105
60
2
x

= = ( khng tha mn iu kin )
2
15 105
45
2
x
+
= = ( tha mn iu kin )
Vy vn tc ca xe my l 45 ( km/h ) , vn tc ca t l 45 + 15 = 60 ( km/h ).

Cu 4.
a) Ta c : C, D thuc ng trn nn :

22
0
90 ACB ADB = = ( gc ni tip chn na ng trn )
=>
0 0
90 ; 90 FCE FDE = = ( gc k b )
Hai im C v D cng nhn on thng FE di mt gc bng nhau
bng 90
0
nn 4 im C,D,E,F cng thuc ng trn ng knh EF.
b) Gi I l trung im EF th ID = IC l bn knh ng trn i qua
4 im C, D, E, F ni trn.
Ta c : IC = ID ; OC = OD ( bn knh ng trn tm O )
suy ra IO l trung trc ca CD => OI l phn gic ca COD
=>
0
0
120
60
2
IOD = =
Do O l trung im AB v tam gic ADB vung ti D nn tam gic ODB cn ti O
=> ODB OBD = (1)
Do ID = IF nn tam gic IFD cn ti I => IFD IDF = (2)
Tam gic AFB c hai ng cao AD, BC ct nhau ti E nn E l trc tm tam gic => FE l ng cao th
ba => FE vung gc AB ti H =>
0
IF 90 OBD D + = (3)
T (1) , (2) , (3) suy ra
0
90 IDF ODB + = =>
0
90 IDO = .
Xt tam gic vung IDO c
0
60 IOD = .
Ta c : ID = OD.tan IOD = R.tan60
0
= R 3 .
Vy bn knh ng trn i qua 4 im C,D,E,F l R 3 .
c) Theo phn b) : OI =
2 2 2 2
3 2 ID OD R R R + = + = .
t OH = x th 0 x R s s => IH =
2 2
4R x .
=> FH = R 3 +
2 2
4R x .
2 2
2 2 2
1 1
. . .2 .( 3 4 )
2 2
3 4
FAB
FAB
S AB FH R R R x
S R R R x
= = +
= +

Ta c : 4R
2
- x
2
s 4R
2
. Du bng xy ra khi x = 0.
Khi : S
FAB
= R
2
3 + 2R
2
v H O => O, I, F thng hng => CD // AB =>
0
15 ADO DAO = = => BD =
AC = 2RSin15
0
.
Vy din tch ln nht t c ca tam gic AFB l R
2
3 + 2R
2
khi AC = BD = 2Rsin15
0
.
Cu 5
Xt hai s a = 2 + 3 v b = 2 - 3 .
Ta c : a + b = 4 v ab = 1, 0< b < 1.
(a+b)
3
= 4
3
= 64 => a
3
+ b
3
= 64 - 3ab(a + b) = 64 - 3.1.4 = 52
(a
3
+b
3
)(a
3
+ b
3
) = 52.52 => a
6
+ b
6
= 2704 - 2(ab)
3
= 2704 - 2 = 2702
=> a
6
= S = 2702 - b
6
(*).
Do 0<b<1 nn 0 < b
6
< 1
Kt hp (*) th s nguyn ln nht khng vt qu S l 2701.







I
H
D
C
E
F
B O
A
15
0
30
0
I
H
D
C
E
F
B O
A

23
S GIO DC V O TO
HI DNG


K THI TUYN SINH LP 10 THPT CHUYN
NGUYN TRI NM HC 2012- 2013
Mn thi: TON (khng chuyn)
Thi gian lm bi: 120 pht
Ngy thi 19 thng 6 nm 2012
t gm : 01 trang

Cu I 20 m
1) Gii phng trnh
1
1
3
x
x

= + .
2) Gii h phng trnh
3 3 3 0
3 2 11
x
x y

+ =

.
Cu II 10 m
Rt gn biu thc
1 1 a + 1
P = + :
2 a - a 2 - a a - 2 a
| |
|
\ .
vi a > 0v a 4 = .
Cu III (1,0 m
Mt tam gic vung c chu vi l 30 cm, di hai cnh gc vung hn km nhau 7cm. Tnh di
cc cnh ca tam gic vung .
Cu IV 20 m
Trong mt phng ta Oxy, cho ng thng (d): y = 2x- m+1 v parabol (P):
2
1
y= x
2
.
1) Tm m ng thng (d) i qua im A(-1; 3).
2) Tm m (d) ct (P) ti hai im phn bit c ta (x
1
; y
1
) v (x
2
; y
2
) sao cho
( )
1 2 1 2
x x y +y 48 0 + = .
Cu V 30 m
Cho ng trn tm O ng knh AB. Trn ng trn ly im C sao cho AC < BC (C= A). Cc
tip tuyn ti B v C ca (O) ct nhau im D, AD ct (O) ti E (E= A) .
1) Chng minh BE
2
= AE.DE.
2) Qua C k ng thng song song vi BD ct AB ti H, DO ct BC ti F. Chng minh t gic
CHOF ni tip .
3) Gi I l giao im ca AD v CH. Chng minh I l trung im ca CH.
Cu VI 10 m
Cho 2 s dng a, b tha mn
1 1
2
a b
+ = . Tm gi tr ln nht ca biu thc

4 2 2 4 2 2
1 1
2 2
Q
a b ab b a ba
= +
+ + + +
.








CHNH THC

24
S GIO DC V O TO
TUYN QUANG


THI TUYN SINH VO LP 10 THPT
Nm c 2011 - 2012
MN THI: TON
Thi gian: 120 pht (khng k thi gian giao )

Cu 1 (3,0 im)
a) Gii phng trnh:
2
6 9 0 x x + =

b) Gii h phng trnh:
4 3 6
3 4 10
x y
y x
=

+ =


c) Gii phng trnh:
2
6 9 2011 x x x + =

Cu 2 (2,5 im)
Mt ca n chy xui dng t A n B ri chy ngc dng t B n A ht tt c 4 gi. Tnh vn tc ca
n khi nc yn lng, bit rng qung sng AB di 30 km v vn tc dng nc l 4 km/gi.
Cu 3 (2,5 im)
Trn ng trn (O) ly hai im M, N sao cho M, O, N khng thng hng. Hai tip tuyn ti M , N vi
ng trn (O) ct nhau ti A. T O k ng vung gc vi OM ct AN ti S. T A k ng vung
gc vi AM ct ON ti I. Chng minh:
a) SO = SA
b) Tam gic OIA cn
Cu 4 (2,0 im).
a) Tm nghim nguyn ca phng trnh: x
2
+ 2y
2
+ 2xy + 3y 4 = 0
b) Cho tam gic ABC vung ti A. Gi I l giao im cc ng phn gic trong. Bit AB = 5 cm,
IC = 6 cm. Tnh BC.
Hng dn cm bu m
MN THI: TON CHUNG
N dung
m
Cu 1 (3,0 im)
a G png trn:
2
6 9 0 x x + =
1,0
Bi gii: Ta c
' 2
( 3) 9 0 A = = 0,5
Phng trnh c nghim:
6
3
2
x

= = 0,5
b) G png trn:
4 3 6 (1)
3 4 10 (2)
x y
y x
=

+ =


1,0
CHNH THC

25
Bi gii: Cng (1) v (2) ta c: 4x - 3y + 3y + 4x = 16 8x = 16 x = 2 0,5
Thay x = 2 vo (1): 4. 2 3y = 6 y =
2
3
. Tp nghim:
2
2
3
x
y
=

0,5
c G png trn:
2
6 9 2011 x x x + =
(3)
1,0

Bi gii: Ta c
( )
2
2
6 9 3 3 x x x x + = =

0,5
Mt khc:
2
6 9 0 2011 0 2011 3 3 x x x x x x + > > > =

Vy: (3) 3 2011 3 2011 x x = = . Phng trnh v nghim
0,5

Cu 2 (2,5 im )
2,5
Bi gii: Gi vn tc ca ca n khi nc yn lng l x km/gi ( x > 4) 0,5
Vn tc ca ca n khi xui dng l x +4 (km/gi), khi ngc dng l x - 4 (km/gi). Thi gian
ca n xui dng t A n B l
30
4 x +
gi, i ngc dng
t B n A l
30
4 x
gi.
0,5
Theo bi ra ta c phng trnh:
30 30
4
4 4 x x
+ =
+
(4) 0,5
2
(4) 30( 4) 30( 4) 4( 4)( 4) 15 16 0 1 x x x x x x x + + = + = =
hoc x = 16. Nghim x = -1 <0 nn b loi
0,5
Vy vn tc ca ca n khi nc yn lng l 16km/gi. 0,5
Cu 3 (2,5 im)

A
S
O
N
M
I


0,5
a Cng mn: SA = SO
1,0
V AM, AN l cc tip tuyn nn:
MAO SAO =
(1)
0,5
V MA//SO nn: MAO SOA = (so le trong) (2)
0,5
T (1) v (2) ta c: SAO SOA = ASAO cn SA = SO (.p.c.m)
b) Chng minh tam gic OIA cn
1,0

26
V AM, AN l cc tip tuyn nn:
MOA NOA =
(3)
0,5
V MO // AI nn: gc MOA bng gc OAI (so le trong) (4)
0,5
T (3) v (4) ta c: IOA IAO = AOIA cn (.p.c.m)
Cu 4 (2,0 im).

a Tm ngm nguyn ca png trn: x
2
+ 2y
2
+ 2xy + 3y 4 = 0 (1)
1,0
Bi gii: (1) (x
2
+ 2xy + y
2
) + (y
2
+ 3y 4) = 0
0,5
(x

+ y)
2
+ (y - 1)(y + 4) = 0
(y - 1)(y + 4) = - (x

+ y)
2
(2)
V - (x

+ y)
2
s 0 vi mi x, y nn: (y - 1)(y + 4) s 0 -4 s y s 1
0,5
V y nguyn nn y e
{ } 4; 3; 2; 1; 0; 1
Thay cc gi tr nguyn ca y vo (2) ta tm c cc cp nghim nguyn (x; y) ca PT cho
l: (4; -4), (1; -3), (5; -3), ( -2; 0), (-1; 1).
b Co tam gc AC vung t A. G I l gao m cc ng pn gc trong. t
AB = 5 cm, IC = 6 cm. Tnh BC.

























5
x
6
D
B
A
C
I
E


Bi gii:
Gi D l hnh chiu vung gc ca C
trn ng thng BI, E l giao im
ca AB v CD. ABIC c DIC l gc
ngoi nn: DIC =
0 0
1
( ) 90 : 2 45
2
IBC ICB B C + = + = =
DIC A vung cn DC = 6 : 2
Mt khc BD l ng phn gic v
ng cao nn tam gic BEC cn ti B
EC = 2 DC = 12: 2 v BC = BE









0,5
Gi x = BC = BE. (x > 0). p dng nh l Pi-ta-go vo cc tam gic vung ABC v ACE ta
c: AC
2
= BC
2
AB
2
= x
2
5
2
= x
2
-25
EC
2
= AC
2
+ AE
2
= x
2
-25 + (x 5)
2
= 2x
2
10x
(12: 2 )
2
= 2x
2
10x
x
2
- 5x 36 = 0
Gii phng trnh ta c nghim x = 9 tho mn. Vy BC = 9 (cm)





O,5

27

CHNH THC

28

















29

30

31

32

33

















34
S GIO DC V O TO K THI TUYN SINH LP 10 THPT
THANH HA NM HC 2012-2013
Mn thi : Ton
Thi gian : 120 pht khng k thi gian giao
Ngy thi 29 thng 6 nm 2012

Bi 1: (2.0 im)
1- Gii cc phng trnh sau :
a) x - 1 = 0 .
b) x
2
- 3x + 2 = 0
2- Gii h phng trnh :

= +
=
2
7 2
y x
y x

Bi 2: (2.0 im) Cho biu thc : A =
a 2 2
1
+
+
a 2 2
1

-
2
2
1
1
a
a

+

1- Tm iu kin xc nh v rt gn biu thc A
2- Tm gi tr ca a ; bit A <
3
1

Bi 3: (2.0 im)
1- Cho ng thng (d) : y = ax + b .Tm a; b ng thng (d) i qua im A( -1 ;
3) v song song vi ng thng (d) : y = 5x + 3
2- Cho phng trnh ax
2
+ 3(a + 1)x + 2a + 4 = 0 ( x l n s ) .Tm a phmg trnh
cho c hai nghim phn bit x
1
; x
2
tho mn
2
1
x +
2
2
x = 4
Bi 4: (3.0 im) Cho tam tam gic u ABC c ng cao AH . Trn cnh BC ly im M
bt k ( M khng trng B ; C; H ) T M k MP ; MQ ln lt vung gc vi cc cnh AB ;
AC ( P thuc AB ; Q thuc AC)
1- Chng minh :T gic APMQ ni tip ng trn
2- Gi O l tm ng trn ngoi tip t gic APMQ .Chng minh OH PQ
3- Chng minh rng : MP +MQ = AH
Bi 5: (1.0 im) Cho hai s thc a; b thay i , tho mn iu kin a + b > 1 v a > 0
Tm gi tr nh nht ca biu thc A =
2
2
4
8
b
a
b a
+
+


---------------------------------------HT ----------------------------------


THI CHNH THC
A

35






36







37




38




39















































40
S GIO DC O TO K THI VO LP 10 CHUYN LAM SN
THANH HO NM HC 2012 - 2013
Mn thi : TON

( gm c 01 trang) Mn cung co tt cc t sn
Thi gian lm bi :120 pht (Khng k thi gian giao )
Ngy thi : 17 thng 6 nm 2012
Cu 1: (2.0 im ) Cho biu thc :
1 1 1
4
1 1 2
a a
P a
a a a a
| |
+
= +
|
|
+
\ .
, (Vi a > 0 , a =1)
1. Chng minh rng :
2
1
P
a
=


2. Tm gi tr ca a P = a
Cu 2 (2,0 im ) : Trong mt phng to Oxy, cho Parabol (P) : y = x
2
v ng thng (d) : y = 2x + 3
1. Chng minh rng (d) v (P) c hai im chung phn bit
2. Gi A v B l cc im chung ca (d) v (P) . Tnh din tch tam gic OAB ( O l gc to )
Cu 3 (2.0 im) : Cho phng trnh : x
2
+ 2mx + m
2
2m + 4 = 0
1. Gii phng trnh khi m = 4
2. Tm m phng trnh c hai nghim phn bit
Cu 4 (3.0 im) : Cho ng trn (O) c ng knh AB c nh, M l mt im thuc (O) ( M khc A v B
) . Cc tip tuyn ca (O) ti A v M ct nhau C. ng trn (I) i qua M v tip xc vi ng thng AC
ti C. CD l ng knh ca (I). Chng minh rng:
1. Ba im O, M, D thng hng
2. Tam gic COD l tam gic cn
3. ng thng i qua D v vung gc vi BC lun i qua mt im c nh khi M di ng trn ng
trn (O)
Cu 5 (1.0 im) : Cho a,b,c l cc s dng khng m tho mn :
2 2 2
3 a b c + + =
Chng minh rng :
2 2 2
1
2 3 2 3 2 3 2
a b c
a b b c c a
+ + s
+ + + + + +

CHNH THC

41
HNG DN GII:

CU NI DUNG IM
1
1. Chng minh rng :
2
1
P
a
=


1 1 1
4
1 1 2
a a
P a
a a a a
| |
+
= +
|
|
+
\ .

( ) ( ) ( )( )
( )( )
2 2
1 1 4 1 1
1
.
2
1 1
a a a a a
P
a a
a a
+ + +
=
+

( )( )
2 1 2 1 4 4 1
.
2
1 1
a a a a a a a
P
a a
a a
+ + + +
=
+

4 1 2
.
1 1 2
a a
P
a a a a
= =

(PCM)

1.0
2. Tm gi tr ca a P = a. P = a
=>
2
2
2 0
1
a a a
a
= => =

.
Ta c 1 + 1 + (-2) = 0, nn phng trnh c 2 nghim
a
1
= -1 < 0 (khng tho mn iu kin) - Loi
a
2
=
2
2
1
c
a

= =
(Tho mn iu kin)
Vy a = 2 th P = a
1.0
2
1. Chng minh rng (d) v (P) c hai im chung phn bit
Honh giao im ng thng (d) v Parabol (P) l nghim ca phng trnh
x
2
= 2x + 3 => x
2
2x 3 = 0 c a b + c = 0
Nn phng trnh c hai nghim phn bit
x
1
= -1 v x
2
=
3
3
1
c
a

= =

Vi x
1
= -1 => y
1
= (-1)
2
= 1 => A (-1; 1)
Vi x
2
= 3 => y
2
= 3
2
= 9 => B (3; 9)
Vy (d) v (P) c hai im chung phn bit A v B
1.0
2. Gi A v B l cc im chung ca (d) v (P) . Tnh din tch tam gic OAB ( O l
gc to )
Ta biu din cc im A v B trn mt phng to Oxy nh hnh v

1
D C
B
A
9
3 -1
0

1.0

42
1 9
. .4 20
2 2
ABCD
AD BC
S DC
+ +
= = =
. 9.3
13, 5
2 2
BOC
BCCO
S = = =
. 1.1
0, 5
2 2
AOD
ADDO
S = = =
Theo cng thc cng din tch ta c:
S
(ABC)
= S
(ABCD)
- S
(BCO)
- S
(ADO)

= 20 13,5 0,5 = 6 (vdt)
3
1. Khi m = 4, ta c phng trnh
x
2
+ 8x + 12 = 0 c A = 16 12 = 4 > 0
Vy phng trnh c hai nghim phn bit
x
1
= - 4 + 2 = - 2 v x
2
= - 4 - 2 = - 6
1.0
2. Tm m phng trnh c hai nghim phn bit
x
2
+ 2mx + m
2
2m + 4 = 0
C D = m
2
(m
2
2m + 4) = 2m 4
phng trnh c hai nghim phn bit th D > 0
=> 2m 4 > 0 => 2(m 2) > 0 => m 2 > 0 => m > 2
Vy vi m > 2 th phng trnh c hai nghim phn bit
1.0
4
1
2
N
K
H
D
I C
O
A B
M

1. Ba im O, M, D thng hng:
Ta c MC l tip tuyn ca ng trn (O) MC MO (1)
Xt ng trn (I) : Ta c
0
90 CMD = MC MD (2)
T (1) v (2) => MO // MD MO v MD trng nhau
O, M, D thng hng
1.0
2. Tam gic COD l tam gic cn
CA l tip tuyn ca ng trn (O) CA AB(3)
ng trn (I) tip xc vi AC ti C CA CD(4)
T (3) v (4) CD // AB => DCO COA = (*)
( Hai gc so le trong)
CA, CM l hai tip tuyn ct nhau ca (O) COA COD = (**)
T (*) v (**) DOC DCO = Tam gic COD cn ti D
1.0
3. ng thng i qua D v vung gc vi BC lun i qua mt im c nh khi M di 1.0

43
ng trn ng trn (O)
* Gi chn ng vung gc h t D ti BC l H.
0
90 CHD = H e (I) (Bi ton
qu tch)
DH ko di ct AB ti K.
Gi N l giao im ca CO v ng trn (I)
=>
0
90
can tai D
CND
NC NO
COD

=
=


Ta c t gic NHOK ni tip
V c
2 1
H O DCO = = ( Cng b vi gc DHN)
0
180 NHO NKO + = (5)
* Ta c : NDH NCH = (Cng chn cung NH ca ng trn (I))
( )
CBO HND HCD = = ADHN ACOB (g.g)
...
...
HN OB
HD OC
OB OA HN ON
OC OC HD CD
OA CN ON
OC CD CD

= =
`

= =

)
M ONH CDH =
ANHO ADHC (c.g.c)

0
90 NHO = M
0
180 NHO NKO + = (5)
0
90 NKO = , NK AB NK // AC
K l trung im ca OA c nh (PCM)
5
Cu 5 (1.0 im) : Cho a,b,c l cc s dng khng m tho mn :
2 2 2
3 a b c + + =

Chng minh rng :
2 2 2
1
2 3 2 3 2 3 2
a b c
a b b c c a
+ + s
+ + + + + +

* C/M b :
( )
2
2 2
a b
a b
x y x y
+
+ >
+
v
( )
2
2 2 2
a b c
a b c
x y x x y z
+ +
+ + >
+ +
.
Tht vy
( )
( )( ) ( ) ( )
2
2 2
2 2
2 2
0
a b
a b
a y b x x y xy a b ay bx
x y x y
+
+ > <=> + + > + <=> >
+

(ng) PCM
p dng 2 ln , ta c:
( )
2
2 2 2
a b c
a b c
x y x x y z
+ +
+ + >
+ +

* Ta c :
2 2
2 3 2 1 2 2 2 2 a b a b a b + + = + + + > + +
, tng t Ta c:
2 2 2
2 3 2 3 2 3 2 2 2 2 2 2 2 2 2
a b c a b c
A
a b b c c a a b b c c a
= + + s + +
+ + + + + + + + + + + +

1
(1)
2 1 1 1
B
a b c
A
a b b c c a
| |
s + +
|
+ + + + + +
\ .

Ta chng minh 1
1 1 1
a b c
a b b c c a
+ + s
+ + + + + +

1.0

44

( )
( )( )
( )
( )( )
( )
( )( )
2 2 2
3
1 1 1 2
1 1 1
1 1 1
2
1 1 1
1 1 1
2
1 1 1
1 1 1
2 (2)
1 1 1 1 1 1
B
a b c
a b b c c a
b c a
a b b c c a
b c a
a b b c c a
b c a
a b b b c c c a a

+ + s
+ + + + + +

+ + s
+ + + + + +
+ + +
+ + >
+ + + + + +
+ + +
+ + >
+ + + + + + + + +

* p dng B trn ta c:

( )
( )( ) ( )( ) ( )( )
2
3
3
1 1 1 1 1 1
a b c
B
a b b b c c c a a
+ + +
>
+ + + + + + + + + + +


( )
2
2 2 2
3
3 (3)
3( ) 3
a b c
B
a b c ab bc ca a b c
+ + +
>
+ + + + + + + + +

* M:

( )
( )
2 2 2
2 2 2
2 2 2 2 2 2
2 2 2
2
2
2 2 2
2 3( ) 3
2 2 2 2 2 2 6 6 6 6
2 2 2 2 2 2 6 6 6 6 ( : 3)
2 2 2 6 6 6 9
3
3
3( )
a b c ab bc ca a b c
a b c ab bc ca a b c
a b c ab bc ca a b c Do a b c
a b c ab bc ca a b c
a b c
a b c
a b c ab bc ca a b c
( + + + + + + + + +

= + + + + + + + + +
= + + + + + + + + + + + =
= + + + + + + + + +
= + + +
+ + +

+ + + + + + + +
2 (4)
3
=
+

T (3) v (4) (2)
Kt hp (2) v (1) ta c iu phi chng minh.
Du = xy ra khi a = b = c = 1


















45
S GIO DC V O TO
THNH PH CN TH



K THI TUYN SINH LP 10 THPT
NM HC 2012-2013
Kha ngy:21/6/2012
MN: TON
Thi gian lm bi: 120 pht (khng k thi gian pht )
Cu 1: (2,0 im)
Gii h phng trnh , cc phng trnh sau y:
1.
43
3 2 19
x y
x y
+ =


2. 5 2 18 x x + =
3.
2
12 36 0 x x + =
4. 2011 4 8044 3 x x + =
Cu 2: (1,5 im)
Cho biu thc:
2
1 1 1
2 :
1
a
K
a a a a
| |
+ | |
=
|
|

\ .
\ .
(vi 0, 1 a a > = )
1. Rt gn biu thc K.
2. Tm a 2012 K = .

Cu 3: (1,5 im)
Cho phng trnh (n s x):
( )
2 2
4 3 0 * x x m + = .
1. Chng minh phng trnh (*) lun c hai nghim phn bit vi mi m.
2. Tm gi tr ca m phng trnh (*) c hai nghim
1 2
, x x tha
2 1
5 x x = .

Cu 4: (1,5 im)
Mt t d nh i t A n B cch nhau 120 km trong mt thi gian quy nh. Sau khi i c 1 gi
th t b chn bi xe cu ha 10 pht. Do n B ng hn xe phi tng vn tc thm 6 km/h.
Tnh vn tc lc u ca t.

Cu 5: (3,5 im)
Cho ng trn
( ) O , t im A ngoi ng trn v hai tip tuyn ABv AC( , B Cl cc tip
im). OAct BC ti E.
1. Chng minh t gic ABOC ni tip.
2. Chng minh BC vung gc vi OA v . . BABE AE BO = .
3. Gi I l trung im ca BE, ng thng qua I v vung gc OI ct cc tia , AB ACtheo th t
ti Dv F . Chng minh IDO BCO = v DOF A cn ti O.
4. Chng minh F l trung im ca AC.

GI GII:
Cu 1: (2,0 im)
Gii h phng trnh , cc phng trnh sau y:
1.
43 2 2 86 5 105 21
3 2 19 3 2 19 43 22
x y x y x x
x y x y x y y
+ = + = = =


= = + = =


2. 5 2 18 ; : 9 x x K x + = >
CHNH THC

46

23( )
5 2 18
13
5 2 18 ( )
3
x TMK
x x
x x x KTMK
=
+ =


+ = + =



3.
2 2
12 36 0 ( 6) 0 6 x x x x + = = =
4.
2011 4 8044 3; : 2011
3 2011 3 2012( )
x x K x
x x TMK
+ = >
= =


Cu 2: (1,5 im)
Cho biu thc:
2
1 1 1
2 :
1
a
K
a a a a
| |
+ | |
=
|
|

\ .
\ .
(vi 0, 1 a a > = )

( )
2
1 1 1 1 1
2 : 2 :
( 1) 1 ( 1)
1 1 1
2 : 2 : ( 1) 2
( 1) ( 1) ( 1)
a a a a
K
a a a a a a a a
a a a
a a a a a a
| | | | | |
+ + + | |
= =
| | |
|

\ .
\ . \ . \ .
| | | | | |
= = =
| | |

\ . \ . \ .

2012 K = 2 a = 2012 a = 503 (TMK)

Cu 3: (1,5 im)
Cho phng trnh (n s x):.
1.
( )
2 2
2 2
4 3 0 *
16 4 12 4 4 4 0;
x x m
m m m
+ =
A = + = + > >

Vy (*) lun c hai nghim phn bit vi mi m.
2. Tm gi tr ca m phng trnh (*) c hai nghim
1 2
, x x tha
2 1
5 x x = .
Theo h thc VI-ET c :x
1
.x
2
= - m
2
+ 3 ;x
1
+ x
2
= 4; m
2 1
5 x x = => x
1
= - 1 ; x
2
= 5
Thay x
1
= - 1 ; x
2
= 5 vo x
1
.x
2
= - m
2
+ 3 => m = 2 2
Cu 4: (1,5 im)
Gi x (km/h) l vt d nh; x > 0 => Thi gian d nh :
120
( ) h
x

Sau 1 h t i c x km => qung ng cn li 120 x ( km)
Vt lc sau: x + 6 ( km/h)
Pt
1 120 120
1
6 6
x
x x

+ + =
+
=> x = 48 (TMK) => KL
HD C3
Tam gic BOC cn ti O => gc OBC = gc OCB
T gic OIBD c gc OID = gc OBD = 90
0
nn OIBD ni tip => gc ODI = gc OBI
Do IDO BCO =
Li c FIOC ni tip ; nn gc IFO = gc ICO
Suy ra gc OPF = gc OFP ; vy DOF A cn ti O.
HD C4
Xt t gic BPFE c IB = IE ; IP = IF ( Tam gic OPF cn c OI l ng cao=> )
Nn BPEF l Hnh bnh hnh => BP // FE
Tam gic ABC c EB = EC ; BA // FE; nn EF l TB ca tam gic ABC => FA = FC


47

S GD T NGH AN thi vo THPT nm hc 2012 -
2013
Mn thi: Ton
Thi
gian 120 pht
Ngy thi 24/ 06/ 2012
Cu 1: 2,5 im:
Cho biu thc A =
1 1 2
.
2 2
x
x x x
| |
+
|
+
\ .

a) Tm iu kin xc nh v t gn A.
b) Tm tt c cc gi tr ca x
1
2
A >
c) Tm tt c cc gi tr ca x
7
3
B A = t gi tr nguyn.
Cu 2: 1,5 im:
Qung -ng AB di 156 km. Mt ng-i i xe my t A, mt ng-i i xe p
t B. Hai xe xut pht cng mt lc v sau 3 gi gp nhau. Bit rng vn
tc ca ng-i I xe my nhanh hn vn tc ca ng-i I xe p l 28 km/h.
Tnh vn tc ca mi xe?
Cu 3: 2 im:
Chjo ph-ng trnh: x
2
2(m-1)x + m
2
6 =0 ( m l tham s).
a) GiI ph-ng trnh khi m = 3
b) Tm m ph-ng trnh c hai nghim x
1
, x
2
tha mn
2 2
1 2
16 x x + =
Cu 4: 4 im
Cho im M nm ngoi -ng trn tm O. V tip tuyn MA, MB vi -ng
trn (A, B l cc tip im). V ct tuyn MCD khng I qua tm O ( C nm
gia M v D), OM ct AB v (O) ln l-t ti H v I. Chng minh.
a) T gic MAOB ni tip.
b) MC.MD = MA
2

c) OH.OM + MC.MD = MO
2

d) CI l tia phn gic gc MCH.





CHNH THC

48





HNG DN GII

Cu 1: (2,5 im)
a, Vi x > 0 v x = 4, ta c:
A =
1 1 2
.
2 2
x
x x x
| |
+
|
+
\ .
=
2 2 2
.
( 2)( 2)
x x x
x x x
+ +
+
= ... =
2
2 x +

b, A =
2
2 x +

2
2 x +
>
1
2
... x > 4.
c, B =
7
3
.
2
2 x +
=
14
3( 2) x +
l mt s nguyn ... 2 x + l c ca 14 hay 2 x + = 1,
2 x + = 7, 2 x + = 14.
(Gii cc pt trn v tm x)

Cu 2: (1,5 im)
Gi vn tc ca xe p l x (km/h), iu kin x > 0
Th vn tc ca xe my l x + 28 (km/h)
Trong 3 gi:
+ Xe p i c qung ng 3x (km),
+ Xe my i c qung ng 3(x + 28) (km), theo bi ra ta c phng trnh:
3x + 3(x + 28) = 156
Gii tm x = 12 (TMK)
Tr li: Vn tc ca xe p l 12 km/h v vn tc ca xe my l 12 + 28 = 40 (km/h)

Cu 3: (2,0 im)
a, Thay x = 3 vo phng trnh x
2
- 2(m - 1)x + m
2
- 6 = 0 v gii phng trnh:
x
2
- 4x + 3 = 0 bng nhiu cch v tm c nghim x
1
= 1, x
2
= 3.
b, Theo h thc Vit, gi x
1
, x
2
l hai nghim ca phng trnh
x
2
- 2(m - 1)x + m
2
- 6 = 0 , ta c:

49
1 2
2
1 2
2( 1)
. 6
x x m
x x m
+ =


v x
1
2
+ x
2
2
= (x
1
+ x
2
)
2
- 2x
1
.x
2
= 16
Thay vo gii v tm c m = 0, m = -4

Cu 4: (4,0 im).
T vit GT-KL


A
D
C
M
I H


B
a, V MA, MB l cc tip tuyn ca ng trn (O) ti A v B nn cc gc ca t gic MAOB vung ti A v
B, nn ni tip c ng trn.
b, AMAC v AMDA c chung M v MAC = MDA (cng chn AC ), nn ng dng. T suy ra
2
.
MA MD
MC MD MA
MC MA
= = (fcm)
c, AMAO v AAHO ng dng v c chung gc O v AMO HAO = (cng chn hai cung bng nhau ca
ng trn ni tip t gic MAOB). Suy ra OH.OM = OA
2

p dng nh l Pitago vo tam gic vung MAO v cc h thc OH.OM = OA
2
MC.MD = MA
2
suy ra
iu phi chng minh.
d, T MH.OM = MA
2
, MC.MD = MA
2
suy ra MH.OM = MC.MD
MH MC
MD MO
= (*)
Trong AMHC v AMDO c (*) v DMO chung nn ng dng.

M O
MC MO MO
HC D A
= = hay
O
MC MO
CH A
= (1)
Ta li c MAI IAH = (cng chn hai cung bng nhau) AI l phn gic ca MAH .
Theo t/c ng phn gic ca tam gic, ta c:
A
MI MA
IH H
= (2)
AMHA v AMAO c OMA chung v
0
90 MHA MAO = = do ng dng (g.g)
H
O

50

O A
MO MA
A H
= (3)
T (1), (2), (3) suy ra
MC MI
CH IH
= suy ra CI l tia phn gic ca gc MCH





S GIO DC V O TO
H NAM


K THI TUYN SINH LP 10 THPT
NM HC 2012 2013
Mn: Ton
Thi gian lm bi: 120 pht
Ngy thi : 22/06/2012

Cu 1 (1,5 im) Rt gn cc biu thc sau:

a) A 2 5 3 45 500
8 2 12
b) B 8
3 1
= +


Cu 2: (2 im)
a) Gii phng trnh: x
2
5x + 4 = 0
b) Gii h phng trnh:
3x y 1
x 2y 5
=

+ =


Cu 3: (2 im)
Trong mt phng to Oxy cho Parabol (P) c phng trnh: y = x
2
v ng thng (d) c phng trnh: y =
2mx 2m + 3 (m l tham s)
a) Tm to cc im thuc (P) bit tung ca chng bng 2
b) Chng minh rng (P) v (d) ct nhau ti hai im phn bit vi mi m.
Gi
1 2
y , y l cc tung giao im ca (P) v (d), tm m
1 2
y y 9 + <
Cu 4: (3,5 im)
Cho ng trn tm O, ng knh AB. Trn tip tuyn ca ng trn (O) ti A ly im M ( M khc A).
T M v tip tuyn th hai MC vi (O) (C l tip im). K CH vung gc vi AB ( H AB e ), MB ct (O) ti
im th hai l K v ct CH ti N. Chng minh rng:
a) T gic AKNH l t gic ni tip.
b) AM
2
= MK.MB
c) Gc KAC bng gc OMB
d) N l trung im ca CH.
Cu 5(1 im)
Cho ba s thc a, b, c tho mn a 1; b 4;c 9 > > >
Tm gi tr ln nht ca biu thc :
bc a 1 ca b 4 ab c 9
P
abc
+ +
=

CHNH THC

51





52





53
S GIO DC V OTO
QUNG TR

K THI TUYN SINH LP 10 THPT NM HC 2012-2013
KHA NGY : 19/6/2012
MN : TON
T gan lm b: 120 pt (khng k thi gian giao )

Cu 1:(2 im)
1.Rt gn cc biu thc (khng dng my tnh cm tay):
a) 2 50 - 18
b)
1
1
1
1
1
1

|
|
.
|

\
|
+
+

=
a
a a
P , vi a>0,a = 1
2.Gii h phng trnh (khng dng my tnh cm tay):

=
= +
5 2
4
y x
y x

Cu 2:(1,5 im)
Gi x
1
, x
2
l hai nghim ca phng trnh 0 3 5
2
= x x .Khng gii phng trnh, tnh gi tr cc
biu thc sau:
a, x
1
+ x
2
b,
2 1
1
x x +
c,
2
2
2
1
x x +
Cu 3:(1,5 im)
Trn mt phng ta , gi (P) l th hm s
2
x y =
a, V (P)
b, Tm ta giao im ca (P) v ng thng d: y = -2x+3
Cu 4:(1,5 im)
Hai xe khi hnh cng mt lc i t a im A n a im B cch nhau 100km. Xe th nht chy
nhanh hn xe th hai 10km/h nn n B sm hm 30 pht, Tnh vn tc mi xe.
Cu 5:(3,5 im)
Cho ng trn (O). ng thng (d) khng i qua tm (O) ct ng trn ti hai im A v B theo
th t, C l im thuc (d) ngoi ng trn (O). V ng knh PQ vung gc vi dy AB ti D ( P thuc
cung ln AB), Tia CP ct ng trn (O) ti im th hai l I, AB ct IQ ti K.
a) Chng minh t gic PDKI ni tip ng trn.
b) Chng minh CI.CP = CK.CD
c) Chng minh IC l phn gic ca gc ngoi nh I ca tam gic AIB.
d) Cho ba im A, B, C c nh. ng trn (O) thay i nhng vn i qua A v B. Chng minh
rng IQ lun i qua mt im c nh.














CHNH THC

54
S GIO DC O TO
NINH THUN

K THI TUYN SINH VO LP 10 THPT
NM HC 2012 2013
Kha ngy: 24 6 2012
Mn thi: TON
Thi gian lm bi: 120 pht

Bi 1: (2,0 im)
a) Gii h phng trnh:
2 3
3 4
x y
x y
+ =

+ =


b) Xc nh cc gi tr ca m h phng trnh sau v nghim:
( 2) ( 1) 3
3 4
m x m y
x y
+ + + =

+ =

( m l tham s)
Bi 2: (3,0 im)
Cho hai hm s y = x
2
v y = x + 2.
a) V th hai hm s cho trn cng mt h trc ta Oxy.
b) Bng php tnh hy xc nh ta cc giao im A, B ca hai th trn (im A c
honh m).
c) Tnh din tch ca tam gic OAB (O l gc ta )
Bi 3: (1,0 im)
Tnh gi tr ca biu thc H = ( 10 2) 3 5 +
Bi 4: (3,0 im)
Cho ng trn tm O, ng knh AC = 2R. T mt im E trn on OA (E khng trng vi A
v O). K dy BD vung gc vi AC. K ng knh DI ca ng trn (O).
a) Chng minh rng: AB = CI.
b) Chng minh rng: EA
2
+ EB
2
+ EC
2
+ ED
2
= 4R
2

c) Tnh din tch ca a gic ABICD theo R khi OE =
2
3
R

Bi 5: (1,0 im)
Cho tam gic ABC v cc trung tuyn AM, BN, CP. Chng minh rng:
3
4
(AB + BC + CA) < AM + BN + CP < AB + BC + CA
P N:
Bi 1: (2,0 im)
a) Gii h phng trnh:
2 3 2 3 5 5 1
3 4 2 6 8 3 4 1
x y x y y x
x y x y x y y
+ = + = = =


+ = + = + = =


b) H phng trnh v nghim khi:
2 1
3 6 1
2 1 3 5
1 3
1 3 4 4 9 1 3 4 2
3 4
m m
m m
m m
m
m m
+ +
=

+ = + + +
= = =

+ + =


Bi 2: (3,0 im)
a) V (d) v (P) trn cng mt h trc ta .
x -2 -1 0 1 2
2
y = x (P)
4 1 0 1 4

x - 2 0
y = x + 2(d) 0 2
CHNH THC

55




















b) Ta giao im ca (P) v (d) l nghim ca h phng trnh:
2 2 2
1 2
1 2
1; 2
2 2 0
1; 4 2 2 2
x x
y x x x x x
y y y x y x y x
= = = = + =


= = = + = + = +


Ta cc giao im ca (d) v (P): A (-1;1) v B (2;4)
c) S
OAB
=
1
2
.(1+4).3 -
1
2
.1.1 -
1
2
.2.4 = 3

Bi 3: (1,0 im)
H =
( ) ( )( )
( 10 2) 3 5 5 1 6 2 5 5 1 5 1 5 1 4 + = + = + = =
Bi 4: (3,0 im)
a) Chng minh rng: AB = CI.

Ta c: BDAC (gt)
DBI = 90
0
( gc ni tip chn na ng trn) BDBI
Do : AC // BI AB CI = AB = CI
b) Chng minh rng: EA
2
+ EB
2
+ EC
2
+ ED
2
= 4R
2

V BDAC AB AD = nn AB = AD

Ta c: EA
2
+ EB
2
+ EC
2
+ ED
2
= AB
2
+ CD
2
= AD
2
+

CD
2
= AC
2
= (2R)
2
= 4R
2

c) Tnh din tch ca a gic ABICD theo R khi OE =
2
3
R

S
ABICD
= S
ABD
+ S
ABIC
=
1
2
.DE.AC +
1
2
.EB.(BI + AC)
* OE =
2
3
R
AE =
3
R
v EC =
2
3
R
+ R =
5
3
R

* DE
2
= AE.EC =
3
R
.
5
3
R
=
2
5
9
R
DE =
5
3
R
. Do : EB =
5
3
R

6
4
2
-2
-4
-6
1
-10 -5 5 10
2
O
A
B
1
-2
E
O
A
C
B
D
I

56
* BI = AC 2AE = 2R 2.
3
R
=
4
3
R

Vy: S
ABICD
=
1
2
.
5
3
R
.2R +
1
2
5
3
R
.(
4
3
R
+ 2R) =
5
6
R
.
16
3
R
=
2
8 5
9
R
(vdt)
Bi 5: (1,0 im)

Cho tam gic ABC v cc trung tuyn AM, BN, CP. Chng minh rng:
3
4
(AB + BC + CA) < AM + BN + CP < AB + BC + CA

Gi G l trng tm ca AABC, ta c: GM =
1
3
AM; GN =
1
3
BN; GP =
1
3
CP
V AM, BN, CP cc trung tuyn, nn: M, N, P ln lt l trung im ca BC, AC, AB
Do : MN, NP, MP l cc ng trung bnh ca AABC
Nn: MN =
1
2
AB; NP =
1
2
BC; MP =
1
2
AC
p dng bt ng thc tam gic, ta c:
* AM < MN + AN hay AM <
1
2
AB +
1
2
AC (1)
Tng t: BN <
1
2
AB +
1
2
BC (2)
CP <
1
2
BC +
1
2
AC (3)
T (1), (2), (3) suy ra: AM + BN + CP < AB + BC + CA (*)
* GN + GM > MN hay
1
3
BN +
1
3
AM >
1
2
AB (4)
Tng t:
1
3
BN +
1
3
CP >
1
2
BC (5)

1
3
CP +
1
3
AM >
1
2
AC (6)
T (4), (5), (6) suy ra:

1
3
BN +
1
3
AM +
1
3
BN +
1
3
CP +
1
3
CP +
1
3
AM >
1
2
AB +
1
2
BC+
1
2
AC

2
3
(AM + BN + CP) >
1
2
(AB + AC + BC)

3
4
(AB + BC + CA) < AM + BN + CP (**)
T (*), (**) suy ra:
3
4
(AB + BC + CA) < AM + BN + CP < AB + BC + CA








G
M
P N
A
B C

57

CHNH THC

58




59




60
S GIO DC V OTO
THA THIN HU

K THI TUYN SINH LP 10 THPT NM HC 2012-2013
Kha ngy : 24/6/2012
Mn thi : TON
T gan lm b: 120 pt (khng k thi gian giao )
Bi 1:(2,0 im)
a).Cho biu thc: C =
( )
5 3 5 3 3
5 3
5 3 1
+ +
+ +
+
. Chng t C = 3
b) Gii phng trnh :
2
3 x 2 x 4 = 0
Bi 2:(2,0 im)
Cho hm s y = x
2
c th (P) v ng thng (d) i qua im M (1;2) c h s gc k = 0.
a/ Chng minh rng vi mi gi tr k = 0. ng thng (d) lun ct (P) ti hai im phn bit A v B.
b/ Gi x
A
v x
B
l honh ca hai im A v B.Chng minh rng
A B A B
x + x x .x 2 = 0
Bi 3:(2,0 im)
a/ Mt xe la i t ga A n ga B.Sau 1 gi 40 pht, mt xe la khc i t ga A n ga B vi vn
tc ln hn vn tc ca xe la th nht l 5 km/h.Hai xe la gp nhau ti mt ga cch ga B 300
km.Tm vn tc ca mi xe, bit rng qung ng st t ga A n ga B di 645 km.
b/ Gii h phng trnh :
( ) ( ) 2 5
20 20
7
x y x y
x y x y
+ =

+ =


Bi 4:(3,0 im)
Cho na ng trn (O) ng knh BC.Ly im A trn tia i ca tia CB.K tip tuyn AF vi na
ng trn (O) ( F l tip im), tia AF ct tia tip tuyn Bx ca na ng trn (O) ti D ( tia tip
tuyn Bx nm trong na mt phng b BC cha na ng trn (O)) .Gi H l giao im ca BF vi
DO ; K l giao im th hai ca DC vi na ng trn (O).
a/ Chng minh rng : AO.AB=AF.AD.
b/ Chng minh t gic KHOC ni tip.
c/ K OM BC ( M thuc on thng AD).Chng minh
BD DM
= 1
DM AM

Bi 5:(1,0 im)
Cho hnh ch nht OABC,
0
COB = 30 .Gi CH l ng cao ca tam gic
COB, CH=20 cm.Khi hnh ch nht OABC quay mt vng quanh cnh OC c
nh ta c mt hnh tr, khi tam gic OHC to thnh hnh (H).Tnh th
tch ca phn hnh tr nm bn ngoi hnh (H).
(Cho 3,1416 t ~ )



30
12 cm
K
H
C
B
A
O
0
CHNH THC

61



62



63




64



























65








Cu 1 (2)
a) Gii phng trnh 2x 5 =1
b) Gii bt phng trnh 3x 1 > 5

Cu 2 (2)
a) Gii h phng trnh

=
= +
7 2
3 3
y x
y x

b) Chng minh rng
7
6
2 3
1
2 3
1
=

+
+

Cu 3 (2)
Cho phng trnh x
2
2(m 3)x 1 = 0
a) Gii phng trnh khi m = 1
b) Tm m phng trnh c nghim x
1
; x
2
m biu thc
A = x
1
2
x
1
x
2
+ x
2
2
t gi tr nh nht? Tm gi tr nh nht .
Cu 4 (3)
Cho tam gic ABC vung ti A. Ly B lm tm v ng trn tm B bn knh AB.Ly C lm tm v ng
trn tm C bn knh AC, hai ng trn ny ct nhau ti im th 2 l D.V AM, AN ln lt l cc dy
cung ca ng trn (B) v (C) sao cho AM vung gc vi AN v D nm gia M; N.
a) CMR: AABC=ADBC
b) CMR: ABDC l t gic ni tip.
c) CMR: ba im M, D, N thng hng
d) Xc nh v tr ca cc dy AM; AN ca ng trn (B) v (C) sao cho on MN c di ln nht.

Cu 5 (1) Gii H PT

+ = +
=
y x y x y x y x
y y x
2 ) 3 2 4 ( 1 2 ) 1 4 2 (
3 8 5
2 2


---------------------------Ht--------------------------
GI GII
Cu 1 (2) a) Gii phng trnh 2x 5 = 1
b) Gii bt phng trnh 3x 1 > 5
p n a) x = 3 ; b) x > 2

Cu 2 (2) a) Gii h phng trnh

=
= +
7 2
3 3
y x
y x

b) Chng minh rng
7
6
2 3
1
2 3
1
=

+
+

p n a) x = 2 ; y = 3
b) VT =
7
6
2 9
2 3 2 3
=

+ +
=VP (pcm)
Cu 3 (2) Cho phng trnh x
2
2(m 3)x 1 = 0
S GIO DC V O TO
PH TH

CHNH THC

K THI TUYN SINH
VO LP 10 TRUNG HC PH THNG
NM HC 2012-2013
Mn ton
Thi gian lm bi: 120 pht, khng k thi gian giao
thi c 01 trang
-------------------------------------------

66
c) Gii phng trnh khi m = 1
d) Tm m phng trnh c nghim x
1
; x
2
m biu thc
A = x
1
2
x
1
x
2
+ x
2
2
t gi tr nh nht? Tm gi tr nh nht .
p n a) x
1
= 5 2 ; x
2
= 5 2 +
e) Thy h s ca pt : a = 1 ; c = A 1 pt lun c 2 nghim
Theo vi- t ta c x
1
+ x
2
=2(m 3) ; x
1
x
2
= 1
M A=x
1
2
x
1
x
2
+ x
2
2
= (x
1
+ x
2
)
2
3x
1
x
2
= 4(m 3)
2
+ 3 > 3
GTNN ca A = 3 m = 3
Cu 4 (3)
Hng dn
a) C AB = DB; AC = DC; BC chung AABC = ADBC (c-c-c)
b) AABC = ADBC gc BAC =BDC = 90
0
ABDC l t gic ni tip
c) C gcA
1
= gcM
1
( AABM cn ti B)
gcA
4
= gcN
2
( AACN cn ti C)
gcA
1
= gcA
4
( cng ph A
2;3
)
gcA
1
= gcM
1
=gcA
4
= gcN
2

gcA
2
= gcN
1
( cng chn cung AD ca (C) )
Li c A
1
+A
2
+ A
3
= 90
0
=> M
1
+ N
1
+ A
3
= 90
0

M AAMN vung ti A => M
1
+ N
1
+ M
2
= 90
0

=> A
3
= M
2
=> A
3
= D
1

ACDN cn ti C => N
1;2
= D
4

D
2;3
+ D
1
+ D
4
=D
2;3
+ D
1
+ N
1;2
= D
2;3
+ M
2
+ N
1
+ N
2

= 90
0
+ M
2
+ N
1
+ M
1
( M
1
= N
2
)
= 90
0
+ 90
0
= 180
0

M; D; N thng hng.
d) AAMN ng dng AABC (g-g)
Ta c NM
2
= AN
2
+AM
2
NM ln nht th AN ; AM ln nht
M AM; AN ln nht khi AM; AN ln lt l ng knh ca (B) v (C)
Vy khi AM; AN ln lt l ng knh ca (B) v (C) th NM ln nht.
Cu 5 (1): Gii H PT

+ = +
=
y x y x y x y x
y y x
2 ) 3 2 4 ( 1 2 ) 1 4 2 (
3 8 5
2 2

Hng dn

+ = +
=
y x y x y x y x
y y x
2 ) 3 2 4 ( 1 2 ) 1 4 2 (
3 8 5
2 2

+ > < = > + <


=
) 2 ( 2 ) 1 1 2 2 ( 1 2 ) 1 2 2 (
) 1 ( 3 8 5
2 2
y x y x y x y x
y y x

T (2) t x +2y = a ; 2xy 1 = b (a:b >0)
Ta dc (2a-1) b =(2b 1) a ( b a )(2 ) 1 + ab = 0 a = b
x = 3y + 1 thay vo (1) ta dc
2y
2
y 1= 0 => y
1
= 1 ; y
2
= 1/2
=> x
1
= 4 ; x
2
= 1/2
Thy x
2
+ 2y
2
= 1 < 0 (loi)
Vy h c nghim (x; y) = (4 ; 1)




2
1
4
3
2
1
2 1
4
3
2
1
2
1 M
D
N
C
B
A

67


S gio dc v o to
H-ng yn


( thi c 01 trang)

k thi tuyn sinh vo lp 10 thpt chuyn
Nm hc 2012 - 2013
Mn thi: Ton
(Dnh cho th sinh d thi cc lp chuyn: Ton,
Tin)
Thi gian lm bi: 150 pht
Bi 1: (2 im)
a) Cho A =
2 2 2 2
2012 2012 .2013 2013 + + . Chng minh A l mt s t nhin.
b) Gii h phng trnh
2
2
1 x
x 3
y y
1 x
x 3
y y

+ + =

+ + =


Bi 2: (2 im)
a) Cho Parbol (P): y = x
2
v ng thng (d): y = (m +2)x m + 6. Tm m ng thng (d) ct
Parabol (P) ti hai im phn bit c honh dng.
b) Gii phng trnh: 5 + x + 2 (4 x)(2x 2) 4( 4 x 2x 2) = +
Bi 3: (2 im)
a) Tm tt c cc s hu t x sao cho A = x
2
+ x+ 6 l mt s chnh phng.
b) Cho x > 1 v y > 1. Chng minh rng :
3 3 2 2
(x y ) (x y )
8
(x 1)(y 1)
+ +
>


Bi 4 (3 im)
Cho tam gic ABC nhn ni tip ng trn tm O, ng cao BE v CF. Tip tuyn ti B v C ct nhau ti
S, gi BC v OS ct nhau ti M
a) Chng minh AB. MB = AE.BS
b) Hai tam gic AEM v ABS ng dng
c) Gi AM ct EF ti N, AS ct BC ti P. CMR NP vung gc vi BC
Bi 5: (1 im)
Trong mt gii bng c 12 i tham d, thi u vng trn mt lt (hai i bt k thi u vi nhau ng
mt trn).
a) Chng minh rng sau 4 vng u (mi i thi u ng 4 trn) lun tm c ba i bng i mt cha thi
u vi nhau.
b) Khng nh trn cn ng khng nu cc i thi u 5 trn?

CHNH THC

68

HNG DN GII
Bi 1: (2 im)
a) Cho A =
2 2 2 2
2012 2012 .2013 2013 + +
t 2012 = a, ta c
2 2 2 2
2012 2012 .2013 2013 + +
2 2 2 2
a a (a 1) (a 1) = + + + +
2 2 2
(a a 1) a a 1 = + + = + +
b) t
x
a
y
1
x b
y

+ =

Ta c
2
2
1 x
x 3
y y
1 x
x 3
y y

+ + =

+ + =


2
1 x
x 3
y y
1 x
x 3
y y

| |
+ =
|

\ .

+ + =


nn
2 2
b a 3 b b 6 0
b a 3 b a 3
= + =


+ = + =


a 6 a 1
v
b 3 b 2
= =

= =


Bi 2:
a) ycbt tng ng vi PT x
2
= (m +2)x m + 6 hay x
2
- (m +2)x + m 6 = 0 c hai nghim dng
phn bit.
b) t t = 4 x 2x 2 +
Bi 3:
a) x = 0, x = 1, x= -1 khng tha mn. Vi x khc cc gi tr ny, trc ht ta chng minh x phi l s
nguyn.
+) x
2
+ x+ 6 l mt s chnh phng nn x
2
+ x phi l s nguyn.
+) Gi s
m
x
n
= vi m v n c c nguyn ln nht l 1.
Ta c x
2
+ x =
2 2
2 2
m m m mn
n n n
+
+ = l s nguyn khi
2
m mn + chia ht cho n
2

nn
2
m mn + chia ht cho n, v mn chia ht cho n nn m
2
chia ht cho n v do m v n c c
nguyn ln nht l 1, suy ra m chia ht cho n( mu thun vi m v n c c nguyn ln nht l
1). Do x phi l s nguyn.
t x
2
+ x+ 6 = k
2

Ta c 4x
2
+ 4x+ 24 = 4 k
2
hay (2x+1)
2
+ 23 = 4 k
2
tng ng vi 4 k
2
- (2x+1)
2
= 23
3 3 2 2 2 2
(x y ) (x y ) x (x 1) y (y 1)
(x 1)(y 1) (x 1)(y 1)
+ + +
=

=
2 2
x y
y 1 x 1
+

2 2
(x 1) 2(x 1) 1 (y 1) 2(y 1) 1
y 1 x 1
+ + + +
= +

2 2
(x 1) (y 1) 2(y 1) 2(x 1) 1 1
y 1 x 1 x 1 y 1 y 1 x 1
( ( (
= + + + + +
( ( (


.
Theo BT Csi

69
2 2 2 2
(x 1) (y 1) (x 1) (y 1)
2 . 2 (x 1)(y 1)
y 1 x 1 y 1 x 1

+ > =


2(y 1) 2(x 1) 2(y 1) 2(x 1)
. 4
x 1 y 1 x 1 y 1

+ > =


1 1 1 1
2 .
y 1 x 1 y 1 x 1
+ >

1 1 1 1
2 . (x 1)(y 1) 2.2 . . (x 1)(y 1) 4
y 1 x 1 y 1 x 1
(
+ > =
(




Bi 4


a) Suy ra t hai tam gic ng dng l ABE v BSM
b) T cu a) ta c
AE MB
AB BS
= (1)
M MB = EM( do tam gic BEC vung ti E c M l trung im ca BC
Nn
AE EM
AB BS
=
C
0 0
MOB BAE, EBA BAE 90 , MBO MOB 90 = + = + =
Nn MBO EBA = do MEB OBA( MBE) = =
Suy ra MEA SBA = (2)
T (1) v (2) suy ra hai tam gic AEM v ABS ng dng(pcm.)
c) D thy SM vung gc vi BC nn chng minh bi ton ta chng minh NP //SM.
+ Xt hai tam gic ANE v APB:
T cu b) ta c hai tam gic AEM v ABS ng dng nn NAE PAB = ,
M AEN ABP = ( do t gic BCEF ni tip)
P
N
F
E
M
S
O
A
B
C
Q

70
Do hai tam gic ANE v APB ng dng nn
AN AE
AP AB
=
Li c
AM AE
AS AB
= ( hai tam gic AEM v ABS ng dng)
Suy ra
AM AN
AS AP
= nn trong tam gic AMS c NP//SM( nh l Talet o)
Do bi ton c chng minh.
Bi 5
a. Gi s kt lun ca bi ton l sai, tc l trong ba i bt k th c hai i u vi nhau ri. Gi
s i gp cc i 2, 3, 4, 5. Xt cc b (1; 6; i) vi i {7; 8; 9;;12}, trong cc b ny phi c t nht
mt cp u vi nhau, tuy nhin 1 khng gp 6 hay i nn 6 gp i vi mi i {7; 8; 9;;12} , v l v
i 6 nh th u hn 4 trn. Vy c pcm.

b. Kt lun khng ng. Chia 12 i thnh 2 nhm, mi nhm 6 i. Trong mi nhm ny, cho tt c
cc i i mt thi u vi nhau. Lc ny r rng mi i u 5 trn. Khi xt 3 i bt k, phi c 2 i
thuc cng mt nhm, do 2 i ny u vi nhau. Ta c phn v d.

th gii quyt n gin hn cho cu a. nh sau:
Do mi i u 4 trn nn tn ti hai i A, B cha u vi nhau. Trong cc i cn li, v A v B ch
u 3 trn vi h nn tng s trn ca A, B vi cc i ny nhiu nht l 6 v do , tn ti i C trong s cc
i cn li cha u vi c A v B. Ta c A, B, C l b ba i i mt cha u vi nhau.






























71
S GIO DC V O TO
HNG YN
K THI TUYN SINH VO LP 10 THPT
NM HC 2012 - 2013
Thi gian lm bi: 120 pht (khng k thi gian giao )
PHN A: TRC NGHIM KHCH QUAN 2 m
T cu 1 n cu 8, hy chn phng n ng v vit ch ci ng trc phng n vo bi lm
Cu 1: gi tr ca biu thc 2 8 + bng:
A. 10 B. 3 2 C. 6 D. 2 4 +
Cu 2: Biu thc 1 2 x x + c ngha khi:
A. x < 2 B. 2 x = C. 1 x = D. 1 x >
Cu 3: ng thng y = (2m 1)x + 3 song song vi ng thng y = 3x 2 khi:
A. m = 2 B. m = - 2 C. 2 m= D. 2 m=
Cu 4: H phng trnh
2 3
3
x y
x y
=

+ =

c nghim (x;y) l:
A. (-2;5) B. (0;-3) C. (1;2) D. (2;1)
Cu 5: Phng trnh x
2
6x 5 = 0 c tng hai nghim l S v tch hai nghim l P th:
A. S = 6; P = -5 B. S = -6; P = 5 C. S = -5; P = 6 D. S = 6; P = 5
Cu 6: th hm s y = -x
2
i qua im:
A. (1;1) B. (-2;4) C. (2;-4)
D. ( 2 ;-1)
Cu 7: Tam gic ABC vung ti A c AB = 4cm; AC = 3cm th di ng cao AH l:
A.
3
4
cm B.
12
5
cm C.
5
12
cm D.
4
3
cm
Cu 8: Hnh tr c bn knh y v chiu cao cng bng R th th tch l
A.
3
2 R t B.
2
R t C.
3
R t D.
2
2 R t
PHN : T LUN 80 m
Bi 1: (1 im)
a) Tm x bit
( )
3 2 2 2 x x + = +
b) Rt gn biu thc:
( )
2
1 3 3 A=
Bi 2: (1,5 im)
CHNH THC

72
Cho ng thng (d): y = 2x + m 1
a) Khi m = 3, tm a im A(a; -4) thuc ng thng (d).
b) Tm m ng thng (d) ct cc trc ta Ox, Oy ln lt ti M v N sao cho tam gic OMN c
din tch bng 1.
Bi 3: (1,5 im) Cho phng trnh x
2
2(m + 1)x + 4m = 0 (1)
a) Gii phng trnh (1) vi m = 2.
b) Tm m phng trnh (1) c nghim x
1
, x
2
tha mn (x
1
+ m)(x
2
+ m) = 3m
2
+ 12
Bi 4: (3 im) T im A bn ngoi ng trn (O), k cc tip tuyn Am, AN vi ng trn (M, N l
cc tip im). ng thng d i qua A ct ng trn (O) ti hai im phn bit B,C (O khng thuc (d), B
nm gia A v C). Gi H l trung im ca BC.
a) Chng minh cc im O, H, M, A, N cng nm trn mt ng trn,
b) Chng minh HA l tia phn gic ca MHN .
c) Ly im E trn MN sao cho BE song song vi AM. Chng minh HE//CM.
Bi 5 (1,0 im) Cho cc s thc dng x, y , z tha mn x + y + z = 4.
Chng minh rng
1 1
1
xy xz
+ >
HNG DN GII:
Pn trc ngm:
Cu
Cu Cu Cu Cu Cu Cu Cu
B D A D A B B C
Pn t lun:
Bi 1:
a) Tm x bit
( )
3 2 2 2 x x + = + 3 2 2 2 2 x x + = + 2 x = . Vy 2 x =
b) Rt gn biu thc:
( )
2
1 3 3 1 3 3 3 1 3 1 A= = = = . Vy 1 A=
Bi 2:
a) Thay m = 3 vo phng trnh ng thng ta c: y = 2x + 2.
im A(a; -4) thuc ng thng (d) khi v ch khi: -4 = 2a + 2 suy ra a = -3.
b) Cho x = 0 suy ra y = m 1 suy ra: 1 ON m = , cho y = 0 suy ra
1
2
m
x

=
suy ra
1 1
2 2
m m
OM hayOM

= =
din tch tam gic OMN = 1 khi v ch khi: OM.ON = 2 khi v ch khi 1 m .
1
2
2
m
=
Khi v ch khi (m 1)
2
= 4 khi v ch khi: m 1 = 2 hoc m 1 = -2 suy ra m = 3 hoc m = -1

73
Vy din tch tam gic OMN = 1 khi v ch khi m = 3 hoc m = -1.
Bi 3: Cho phng trnh x
2
2(m + 1)x + 4m = 0 (1)
a) Gii phng trnh (1) vi m = 2.
b) Tm m phng trnh (1) c nghim x
1
, x
2
tha mn (x
1
+ m)(x
2
+ m) = 3m
2
+ 12
HD:
a) Thay m = 2 vo phng trnh (1) ta c phng trnh:
x
2
6x + 8 = 0 Khi v ch khi (x 2)(x 4) = 0 khi v ch khi x = 2 hoc x = 4
Vy vi m = 2 th phng trnh c 2 nghim x
1
= 2 , x
2
= 4.
b) Ta c ( ) ( )
2 2
' 1 4 1 0 m m m A = + = > vy phng trnh lun c nghim vi mi m.
p dng nh l Vi-et ta c:
( ) 2 1
4
S m
P m
= +


(x
1
+ m)(x
2
+ m) = 3m
2
+ 12 khi v ch khi x
1
x
2
+ (x
1
+ x
2
) m - 2 m
2
12 = 0. S khi v ch khi : 4m
+ m.2(m + 1) 2m
2
12 = 0 khi v ch khi 6m = 12 khi v ch khi m= 2
Bi 5 :
a) Theo tnh cht tip tuyn ct nhau ta c :
0
90 AMO ANO = =
Do H l trung im ca BC nn ta c:
0
90 AHO =
Do 3 im A, M, H, N, O thuc ng trn ng knh AO
b) Theo tnh cht hai tip tuyn ct nhau ta c: AM = AN
Do 5 im A, M, H, O, N cng thuc mt ng trn nn:
AHM AHN = (gc ni tip chn hai cung bng nhau)
Do HA l tia phn gic ca MHN
c) Theo gi thit AM//BE nn MAC EBH = ( ng v) (1)
Do 5 im A, M, H, O, N cng thuc mt ng trn nn:
MAH MNH = (gc ni tip chn cung MH) (2)
T (1) v (2) suy ra ENH EBH =
Suy ra t gic EBNH ni tip
Suy ra EHB ENB =
M ENB MCB = (gc ni tip chn cung MB)
Suy ra: EHB MCB =
Suy ra EH//MC.
Bi 5 (1,0 im) Cho cc s thc dng x, y , z tha mn x + y + z = 4.
E
B
H
N
O A
M
C
E
B
H
N
O A
M
C

74
Chng minh rng
1 1
1
xy xz
+ >
Hng dn:
V x + y + z = 4 nn suy ra x = 4 (y + z)
Mt khc:
1 1 1 1 1 1 1
1 1 x
xy xz x y z y z
| |
+ > + > + >
|
\ .
do x dng. (*)
Thay x = 4 (y + z) vo (*) ta c :
( )
2
2
1 1 1 1 1 1
4 2 2 0 0 y z y z y z
y z y z y z
| |
| |
+ > + + + + > + >
|
|
|
\ .
\ .

Lun ng vi mi x, y, z dng, du bng xy ra khi v ch khi : y = z = 1, x = 2.



























75


S GIO DC V O TO K THI TUYN SINH VO LP 10 THPT NM HC 2012
NG NAI Kha ngy : 29 , 30 / 6 / 2012
Mn t : TON HC
Thi gian lm bi : 120 pht
( ny c 1 trang , 5 cu )
Cu 1 : ( 1,5 im )
1 / Gii phng trnh : 7x
2
8x 9 = 0 .
2 / Gii h phng trnh :
3x+2y =1
4x +5y = 6


Cu 2 : ( 2,0 im )
1 / Rt gn cc biu thc :
12 +3 3 2 2
M ; N
3 2 1

= =
2 / Cho x
1
; x
2
l hai nghim ca phng trnh : x
2
x 1 = 0 .
Tnh :
1 2
1 1
+
x x
.
Cu 3 : ( 1,5 im )
Trong mt phng vi h trc ta Oxy cho cc hm s :
y = 3x
2
c th ( P ) ; y = 2x 3 c th l ( d ) ; y = kx + n c th l ( d
1
) vi k v n l nhng s
thc .
1 / V th ( P ) .
2 / Tm k v n bit ( d
1
) i qua im T( 1 ; 2 ) v ( d
1
) // ( d ) .

Cu 4 : ( 1,5 im )
Mt tha t hnh ch nht c chu vi bng 198 m , din tch bng 2430 m
2
. Tnh chiu di v chiu
rng ca tha t hnh ch nht cho .

Cu 5 : ( 3,5 im )
Cho hnh vung ABCD . Ly im E thuc cnh BC , vi E khng trng B v E khng trng C . V
EF vung gc vi AE , vi F thuc CD . ng thng AF ct ng thng BC ti G . V ng thng a i
qua im A v vung gc vi AE , ng thng a ct ng thng DE ti im H .
1 / Chng minh
AE CD
AF DE
= .
2 / Chng minh rng t gic AEGH l t gic ni tip c ng trn .
3 / Gi b l tip tuyn ca ng trn ngoi tip tam gic AHE ti E , bit b ct ng trung trc ca
on thng EG ti im K . Chng minh rng KG l tip tuyn ca ng trn ngoi tip tam gic AHE .

HNG DN GII:

Cu 1 : ( 1,5 im )
1 / Gii phng trnh : 7x
2
8x 9 = 0 ( x
1,2
=
4 79
7

)
2 / Gii h phng trnh :
3x+2y =1
4x +5y = 6

( x ; y ) = (1 ; 2 )

Cu 2 : ( 2,0 im )
CHNH THC

76
1 / Rt gn cc biu thc :
12 +3 2 3 3
M 2 3
3 3
+
= = = +
( )
2
2 1
3 2 2
N 2 1
2 1 2 1

= =

=
2 / Cho x
1
; x
2
l hai nghim ca phng trnh : x
2
x 1 = 0 .
S =
b
1
a
= ; P =
c
1
a
=
Nn :
1 2
1 2 1 2
1
1
1
x x 1 1
+
x x x x
+
= = =


Cu 3 : ( 1,5 im )
1 / V th ( P ) .
2 / ( d
1
) // ( d ) nn k = 2 ; n =3 v i qua im T( 1 ; 2 ) nn x = 1 ; y = 2 . Ta c phng trnh : 2
= 1.2 + n n = 0

Cu 4 : ( 1,5 im )
Gi x ( m ) l chiu di tha t hnh ch nht ( 49,5 < x < 99 )
Chiu rng ca tha t hnh ch nht l : 99 x ( m )
Theo bi ta c phng trnh : x ( x 99 ) = 2430
Gii c : x
1
= 54 ( nhn ) ; x
2
= 45 ( loi )
Vy chiu di tha t hnh ch nht l 54 ( m )
Chiu rng ca tha t hnh ch nht l : 99 54 = 45 ( m )

Cu 5 : ( 3,5 im )
1 / Chng minh t gic AEFD ni tip
1 1
A D =
AAEF ADCE ( g g )
AE AF
=
DC DE
AE DC
=
AF DE


2 / Ta c
2
A ph vi
1
A
Ta c
1
E ph vi
1
D
M
1 1
A D =
2 1
A E =
Suy ra t gic AEFD ni tip ng trn ng knh HE
Gi I trung im ca HEI l tm ng trn ngoi tip t gic AEFD cng l ng trn ngoi
tip AHE
I nm trn ng trung trc EG IE = IG
V K nm trn ng trung trc EG KE = KG
Suy ra AIEK =AIGK ( c-c-c )
0
IGK IEK 90 = =
KG IG ti G ca ng trn ngoi tip AHE
KG l tip tuyn ca ng trn ngoi tipAHE
1
2
1
1
K I
b
a
G
H
F
E
D
C
B
A

77




THI TUYN SINH VO LP 10 CHUYN TNH NG NAI
NM HC 2012 - 2013
Mn thi: Ton chung
Thi gian lm bi: 120 pht ( khng k thi gian giao )
( thi ny gm mt trang, c bn cu)
Cu 1: ( 2,5 im) .
1/ Gii cc phng trnh :
a/
4 2
20 0 x x =
b/ 1 1 x x + =
2/ Gii h phng trnh :
3 1
3
x y
y x
+ =


Cu 2 : ( 2,0 im) .
Cho parabol y = x
2
(P) v ng thng y = mx (d), vi m l tham s.
1/ Tm cc gi tr ca m (P) v (d) ct nhau ti im c tung bng 9.
2/ Tm cc gi tr ca m (P) v (d) ct nhau ti 2 im, m khong cch gia hai im ny bng
6

Cu 3 : ( 2,0 im)
1/ Tnh :
1 1 3 1
( ).
2 3 2 3 3 3
P

=
+

2/ Chng minh :
5 5 3 2 2 3
a b a b a b + > + , bit rng 0 a b + > .

Cu 4 : (3,5 im)
Cho tam gic ABC vung A, ng cao AH. V ng trn tm O, ng knh AH, ng trn ny
ct cc cnh AB, AC theo th t ti D v E .
1/ Chng minh t gic BDEC l t gic ni tip c ng trn.
2/ Chng minh 3 im D, O, E thng hng.
3/ Cho bit AB = 3 cm, BC = 5 cm. Tnh din tch t gic BDEC.

--------HT------















CHNH THC

78





THI TUYN SINH VO LP 10 CHUYN TNH NG NAI
NM HC 2012 - 2013
Mn thi: Ton ( mn chuyn)
Thi gian lm bi: 150 pht ( khng k thi gian giao )
( thi ny gm mt trang, c nm cu)

Cu 1. (1,5 im)
Cho phng trnh
4 2
16 32 0 x x + = ( vi x R e )
Chng minh rng 6 3 2 3 2 2 3 x = + + + l mt nghim ca phng trnh cho.

Cu 2. (2,5 im)
Gii h phng trnh
2 ( 1)( 1) 6
2 ( 1)( 1) yx 6
x x y xy
y y x
+ + + =

+ + + =

( vi , x R y R e e ).

Cu 3.(1,5 im)
Cho tam gic u MNP c cnh bng 2 cm. Ly n im thuc cc cnh hoc pha trong tam gic u
MNP sao cho khong cch gia hai im tu ln hn 1 cm ( vi n l s nguyn dng). Tm n ln
nht tho mn iu kin cho.

Cu 4. (1 im)
Chng minh rng trong 10 s nguyn dng lin tip khng tn ti hai s c c chung ln hn 9.

Cu 5. (3,5 im)
Cho tam gic ABC khng l tam gic cn, bit tam gic ABC ngoi tip ng trn (I). Gi D,E,F ln
lt l cc tip im ca BC, CA, AB vi ng trn (I). Gi M l giao im ca ng thng EF v
ng thng BC, bit AD ct ng trn (I) ti im N (N khng trng vi D), gii K l giao im
ca AI v EF.
1) Chng minh rng cc im I, D, N, K cng thuc mt ng trn.
2) Chng minh MN l tip tuyn ca ng trn (I).

----------HT-----------














CHNH THC

79





GII THI VO LP 10
CHUYN LNG TH VINH NG NAI
NM 2012 2013
Mn: Ton chung
-----------------
Cu 1: ( 2,5 im) .
1/ Gii cc phng trnh :
a/
4 2
20 0 x x = (*) t
2
; ( 0) x t t = >
(*) t
2
t 20 = 0 (t
1
= 5 (nhn) v t
2
= - 4 ( loi)); Vi t = 5 => x
2
= 5 x = 5
Vy phng trnh c hai nghim x = 5 v x = - 5
b/ 1 1 x x + = ( iu kin 1 x > )
2 2 2 2
( 1) ( 1) 1 2 1 3 0 x x x x x x x + = + = + = x(x-3) = 0
x = 0 ( loi) v x = 3 ( nhn).
Vy phng trnh c mt nghim x = 3.
2/ Gii h phng trnh :
3 1
3
x y
y x
+ =


T 3 3 3 0 3 3 y x y x y y y = = > =
1
3 1 3 1 4 2 1
2
3 3 3 3 7
2
x
x y x y x y x
y x y x y x y x
y

=
+ = + = + = =



= = = = +

=

(nhn)
Vy h phng trnh c 2 nghim (x; y):
1 7 1 7
( ; ), ( ; )
2 2 2 2


Cu 2 : ( 2,0 im) .
1/ P.trnh honh giao im (P) v (d) :
1 2
2
0
0 ( ) 0
x
x mx x x m
x m
=
= =

=


V giao im
2 2
( ) : P y x y m e = = . Vi y = 9 => m
2
= 9 (m = 3 v m = -3)
Vy vi 3 m= th (P) v (d) ct nhau ti im c tung bng 9.

2/ T cu 1 => (P) v (d) lun ct nhau ti hai im phn bit khi 0 m= .
Khi giao im th nht l gc to O ( x = 0; y = 0), giao im th 2 l im A c ( x = m; y = m
2
).
Khong cch gia hai giao im : AO =
2 4 4 2
6 6 0 m m m m + = + = (1)
t
2
; ( 0) t m t = > (1)
2
6 0 t t + = (t
1
= 3 ( nhn ) v t
2
= - 2 ( loi))
Vi t
1
= 3 m
2
= 3 , 3 m = ( nhn)
Vy vi 3 m = th (P) ct (d) ti hai im c khong cch bng 6 .

Cu 3 : ( 2,0 im)
1/ Tnh:

80

1 1 3 1 2 3 2 3 3 1
( ). . 2
4 3 2 3 2 3 3 3 3( 3 1)
P
+ +
= = =
+

2/ Ta c:
5 5 3 2 2 3 5 5 3 2 2 3 3 2 2 3 2 2 3 3 2 2
2 2 2
0 ( ) ( ) 0 ( )( ) 0
( ) ( )( ) 0
a b a b a b a b a b a b a a b b a b a b a b
a b a b a b ab
+ > + + > > >
+ + + >

V :
2
( ) 0 a b > (vi mi a, b R e ).
0 a b + > ( theo gi thit)

2 2
0 a b ab + + > ( vi mi a, b R e )
Nn bt ng thc cui ng. Vy
5 5 3 2 2 3
a b a b a b + > + vi 0 a b + > (pcm)

Cu 4 : (3,5 im)
E
D
O
H
C B
A

1/ Ni H vi E .
+
0
90 HEA Z = ( v AH l ng knh),
0
90 AHC Z = ( AH l ng cao)
=> AHE ACB Z =Z (cng ph vi EHC Z ) (1)
+ ADE AHE Z =Z ( gc ni tip cng chn cung AE) (2)
T (1) v (2) => ZADE = ZACB =>T gic BDEC ni tip ng trn ( c gc i bng gc k
b gc i)
2/ V
0
90 DAE Z = => DE l ng knh => D, O, E thng hng (pcm).
3/ Ta c
BDEC ABC ADE
S S S
A A
=
+ ABC A vung c AH l ng cao:

2 2
4 AC BC AB cm = = =>
.
6
2
ABC
AB AC
s
A
= = (cm
2
)
. 12
5
AB AC
DE AH
BC
= = = (cm) ( cng l ng knh t O).

+AADE v AABC c : ZA chung , ZADE = ZACB ( cu 1)
=> AADE ~ AABC (g.g) => t s din tch bng bnh phng t ng dng :

2
2
2
.
ABC AED
AED
ABC
S DE S DE
S
S BC BC
A A
A
A
| |
= =
|
\ .


+
2 2
2 2 2
12
(1 ) 6(1 )
5 .5
BDEC ABC ADE ABC
DE
S S S S
BC
A A A
= = = = 4,6176 (cm
2
)
---------HT---------






81







GII THI VO LP 10
CHUYN LNG TH VINH NG NAI
NM 2012 2013
Mn: Ton chuyn
-----------------
Cu 1: Phng trnh cho :
4 2
16 32 0 x x + = ( vi x R e )
2 2
( 8) 32 0 x = (1)
Vi 6 3 2 3 2 2 3 x = + + + 3 2 2 3 2 2 3 x = + + +
=>
2
8 2 2 3 2 3 2 3 x = +
Th x vo v phi ca (1) ta c:
2 2 2
( 8) 32 (8 2 2 3 2 3 2 3 8) 32 4(2 3) 4 3 12(2 3) 32 x = + = + + +
=8 4 3 8 3 24 12 3 32 0 + + + = ( v phi bng v tri)
Vy 6 3 2 3 2 2 3 x = + + + l mt nghim ca phng trnh cho ( pcm)

Cu 2: H pt cho
2 ( 1)( 1) 6
2 ( 1)( 1) yx 6
x x y xy
y y x
+ + + =

+ + + =


(1)
(2)

`
)

2 ( 1)( 1) 6
2 ( 1)( 1) 6
x x y xy
y y x xy
+ + =

+ + =


Thay x = 0, y = 0 th h khng tho . Thay x = -1 v y = -1 vo, h khng tho
=>( ; ) (0;0); 0; 1 0; 1 0 6 0 x y xy x y xy = = + = + = = (*)
- Chia tng v ca hai phng trnh cho nhau : =>
6
( ) 6( )
6
x xy
xy x y x y
y xy

= = +


Thay x = y, h pt c v phi bng nhau, v tri khc nhau (khng tho) => 0 x y = ) (**)
=>
6( ) x y
xy
x y
+
=

(3)
- Cng tng v (1) v (2) ca h ta c pt: 2(x+y)(x+1)(y+1) + 2xy = 0 (4)
(x + y) ( x + y + xy + 1) + xy = 0
6( ) 6( )
( )( 1 ) 0
x y x y
x y x y
x y x y
+ +
+ + + + + =

6( 1)
( )( 1 ) 0
x y
x y x y
x y
+ +
+ + + + =


6
( )( 1)(1 ) 0 x y x y
x y
+ + + + =


0
1 0
6
1 0
x y
x y
x y
+ =

+ + =

+ =


- Vi x + y = 0 x = - y. Th vo h => -2y
2
= 0 (y = 0 v x = 0) khng tho (*)
- Vi x + y +1 =0 x = -y - 1 th vo phng trnh (1) ca h ta c :

3 2 2
2 3 6 0 ( 2)(2 3) 0 y y y y y y + + + = + + =
2
2 0 2
2 3 0( )
y y
y y vn
+ = =

+ =


Vi y = - 2 => x = 1.Th vo h tho, vy c nghim 1: (x; y) = (1; - 2)
- Vi
6
1 0 6 0 6 x y x y
x y
+ = + = =


Th x = y -6 vo pt (2) ca h :

82
(2)
3 2
2 7 16 6 0 y y y =
2
2
2 1 0
(2 1)( 4 6) 0
4 6 0
y
y y y
y y
+ =
+ =

=


y
2
- 4y - 6 = 0
1
2
2 10
2 10
y
y

= +


2y +1 = 0 y
3
=
1
2

T ba gi tr ca y trn ta tm c ba gi tr x tng ng:
1
2
3
4 10
4 10
13
2
x
x
x

= +


Th cc gi tr (x; y) tm c vo h (tho).

Vy h phng trnh cho c 4 nghim ( x;y):
(1; -2), (
13 1
4 10; 2 10), ( 4 10; 2 10), ( ; ).
2 2
+ +
Cu 3. (Cch 1)
Tam gic u c cnh bng 2 cm th din tch bng 3 cm
2
, tam gic u c cnh bng 1 cm th din
tch bng
3
4
cm
2
. Nu tam gic u c cnh > 1cm th din tch >
3
4
cm
2

Gi t l s tam gic u c cnh bng > 1cm cha c trong tam gic u c cnh 2 cm:
1 4 t s ( vi t l s nguyn dng) => t
max
= 3.
Theo nguyn l Drichen s c 1 trong t tam gic u c cnh > 1cm cha ti a 2 im tho mn khong
cch gia hai im bt k lun > 1 cm.
Vy s im tho yu cu bi ton l : 2 4 n s s Vy n
max
= 4

(Cch 2): Gii theo kin thc hnh hc

Nu ta chn 3 im 3 nh ca tam gic u cnh bng 2 cm v 3 ng trn ng knh 1 cm, cc
ng trn ny tip xc vi nhau trung im mi cnh tam gic. => Cc im khc trong tam gic cch 3
nh > 1cm ch c th nm trong phn din tch cn li ca tam gic (ngoi phn din tch b ba hinh trn
che ph), c gii hn bi 3 cung trn bn kinh 1 cm.
V 3 dy cung l 3 ng trung bnh ca tam gic c di 1 cm => khong cch gia hai im bt
k nm trong phn din tch cn li ca tam gic lun s1 cm.
=> trong phn din tch ch ly c 1 im m khong cch n 3 nh ca tam gic lun > 1 cm.
Vy s im ln nht tho mn khong cch gia hai im bt k > 1cm l :

83
n
max
= 3 + 1 = 4 im.

Cu 4. Gi a v b l hai s bt k trong 10 s nguyn dng lin tip vi a > b ( a; b nguyn dng)
1 9 a b s s .
Gi n l c chung ca a v b, khi : a = n.x v b = n.y ( n, x, y l s nguyn dng).
V a > b => x > y => 1 x y >
1 9
1 . . 9 n x n y x y
n n
s s s s
9
1 9 n
n
> s
Vy trong 10 s nguyn dng lin tip khng tn ti hai s c c chung ln hn 9.

Cu 5.
D
K
F
N
E
M
I
C
B
A

1)Ni N v F, D v F.
- Xt AANF v A AAFD c: ZAFN = ZADF ( v AF l tt) v ZFAD chung =>AANFAAFD (g.g)
=>
2
AF
AF .
AF
AN
AN AD
AD
= = (1)
- Xt AAFI c: AFIF ( v AF tip tuyn, FI l bn knh) v FK AI ( v AF v AE tt chung v AI ni
tm) => AAFI vung ti F c FK l ng cao) => AK.AI = AF
2
(2)
- Xt AANK v AAID c:
+ ZIAD chung.
+ T (1) v (2) => AN.AD = AK.AI =>
AN AI
AK AD
=
=>AANKAAID (c.g.c) =>ZNKA = ZIDN (3)
- T (3) => t gic DIKN ni tip t (v c gc i bng gc k b gc i)
=> cc im I,D,N,K cng thuc mt ng trn. (pcm).
2) Ta c IDDM ( DM l tip tuyn, DI l bn knh) v IKKM ( cu 1) => t gic DIKM ni tip
ng trn ng knh MI. V 4 im D, I, K, N cng thuc mt ng trn ( cu 1) => hai ng trn
ny cng ngoi tip A DIK => hai ng trn trng nhau => N cng nm trn ng trn ng knh MI
=> INM Z = 90
0
.
V IN l bn knh ng trn (I), MN IN => MN l tip tuyn ca ng trn (I) ti tip im N.
(pcm).
-----------HT----------








84


CHNH THC

85

GI GII:

Cu 1c C = 1
Cu 2a ( 2;1) ; Cu 2b b = - 1
Cu 3a a = 1
Cu 3b A ( -1 ; 1 ) ; B (2 ; 4 )
Cu 4a
1
12 0 A = > ; nn pt lun c 2 nghim phn bit vi mi x
Cu 4 a
2
=> x
1
+ x
2
= - 5 ; x
1
x
2
= 3
Cu 4b
Gi x ( km/h) l vt xe II => vt xe I l x + 10 ( km/h ) ; x> 0

Th gian xe I i ht qg :
100
x
(h)
Th gian xe II i ht qg :
100
10 x +
(h)
PT
100
x
-
100
10 x +
=
1
2
=> x = 40
KL

Cu 5 : a

1. MH = 20 ( cm ) ; ME = 12 ( cm)
2. NPFE l h thang cn
b )
b
1

b
2

Tam gic ABC vung ti A c AH l g cao => AB
2
= BH.BC (1)

86
Tam gic BHE g dng vi tam gic BDC => . .
BH BE
BH BC BDBE
BD BC
= => = (2)
T (1) v (2) => AB
2
= BD . BE






















Cu 1: (2,0 im)
1. Cho biu thc P = x + 5. Tnh gi tr biu thc P ti x = 1.
2. Hm s bc nht y = 2x + 1 ng bin hay nghch bin trn R? V sao?
3. Gii phng trnh x
2
+ 5x + 4 = 0
Cu 2: (2,5 im)
1. Gii h phng trnh:
2 1
3 2 5
x y
x y
+ =


2. Cho biu thc Q =
1 1 1 2
:
1 1 1 x x x x x
| | | |
+ +
| |
+
\ . \ .
vi x > 0 v x = 1.
a) Rt gn Q.
b) Tnh gi tr ca Q vi x = 7 4 3 .
Cu 3: (1,5 im)
Khong cch gia hai bn sng A v b l 30 km. Mt ca n i xui dng t bn A n bn B ri li
ngc dng t bn B v bn A. Tng thi gian ca n i xui dng v ngc dng l 4 gi . Tm vn tc ca
ca n khi nc yn lng, bit vn tc ca dng nc l 4 km/h.
Cu 4: (3,0 im)
Cho ng trn tm O bn knh R. Mt ng thng d khng i qua O v ct ng trn ti hai im
phn bit A v B. Trn d ly im M sao cho A nm gia M v B. T M k hai tip tuyn MC v MD vi
ng trn (C, D l cc tip im).
S GIO DC - O TO
TNH NINH NH


K THI TUYN SINH VO LP 10 THPT
NM HC 2012 2013
Mn thi: TON
Thi gian lm bi: 120 pht (khng k thi gian giao )
thi gm 05 cu trong 01 trang
CHNH THC

87
1. Chng minh rng MCOD l t gic ni tip.
2. Gi I l trung im ca AB. ng thng IO ct tia MD ti K. Chng minh rng KD. KM = KO.
KI
3. Mt ng thng i qua O v song song vi CD ct cc tia MC v MD ln lt ti E v F. Xc
nh v tr ca M trn d sao cho din tch tam gic MEF t gi tr nh nht.
Cu 5: (1,0 im)
Cho a, b, c l cc s thc dng. Chng minh rng:
4
b c c a a b a b c
a b c b c c a a b
| | + + +
+ + > + +
|
+ + +
\ .

------------------- Ht ----------------------
HNG DN GII:
Cu 1:
1) Thay x = 1 vo biu thc P c: P = x + 5 = 1 + 5 = 6.
2) Hm s ng bin trn R v a = 2 > 0
3)Ta thy a b + c = 1 5 + 4 = 0 nn pt c 2 nghim: x
1
= 1; x
2
= 4
Cu 2:
1.

Vy h pt c nghim : x = 1 v y = 1.
2. Vi x > 0 v x = 1, ta c:

a) Q

b) Vi
Suy ra :
Cu 3:
Gi vn tc ca ca n khi nc yn lng l x(km/h) (k: )
Vn tc ca ca n khi xui dng: x + 4 (km/h)
Vn tc ca ca n khi ngc dng: x 4 (km/h)

88
Thi gian ca n i xui dng:
30
4 x +
(h)
Thi gian ca n i ngc dng:
30
4 x
(h)
Tng thi gian ca n i xui dng v ngc dng l 4h nn ta c phng trnh:
30
4 x +
+
30
4 x
= 4 x
2
15x 16 = 0
Gii phng trnh trn ta c:
=

1
2
1( )
16( )
x khong thoa K
x thoa K

Vy vn tc ca ca n khi nc yn lng l 16km/h

















89





Cu 5: (cch 2)
p dng bt ng thc Csi ta c:: du bng xy ra a = b








Tng t ta c : du = xy ra a = b = c











90
















Cu 1 (2 im). Cho phng trnh bc hai n x, tham s m: x
2
+ 2mx 2m 3 = 0 (1)
a) Gii phng trnh (1) vi m = -1.
b) Xc nh gi tr ca m phng trnh (1) c hai nghim x
1
, x
2
sao cho
2
2
2
1
x x +
nh nht. Tm
nghim ca phng trnh (1) ng vi m va tm c.
Cu 2 (2,5 im).
1. Cho biu thc A=
|
|
.
|

\
|

+
+
|
|
.
|

\
|
+ +

+
x
x
x
x x
x
x
x
3
3 1
3 3 1
4 3 2 3
3
8 3 3
4 6
3
3

a) Rt gn biu thc A.
b) Tm cc gi tr nguyn ca x biu thc A nhn gi tr nguyn.
2. Gii phng trnh: ( ) 1 1 1 = + + x x x x
Cu 3 (1,5 im). Mt ngi i xe p t A ti B, qung ng AB di 24 km. Khi i t B tr v A ngi
tng vn tc thm 4 km/h so vi lc i, v vy thi gian v t hn thi gian i l 30 pht. Tnh vn tc ca xe
p khi i t A ti B.
Cu 4 (3 im). Cho AABC nhn ni tip (O). Gi s M l im thuc on thng AB (M=A, B); N l
im thuc tia i ca tia CA sao cho khi MN ct BC ti I th I l trung im ca MN. ng trn ngoi tip
AAMN ct (O) ti im P khc A.
S GIO DC - O TO
TNH NINH NH


K THI TUYN SINH VO LP 10 THPT CHUYN
Mn thi: TON
Ngy thi: 26 / 6 / 2012
Thi gian lm bi: 120 pht
CHNH THC

91
1. C MR cc t gic BMIP v CNPI ni tip c.
2. Gi s PB = PC. Chng minh rng A ABC cn.
Cu 5 (1 im). Cho x; y R e , tha mn x
2
+ y
2
= 1. Tm GTLN ca :
2 +
=
y
x
P
HNG DN GII:
2) Gii pt : 1 ) 1 ( 1 = + + x x x x K : 1 0 s s x
t 0 1 ; 0 > = > = b x a x
Ta c

= +
= + +
(**) 1
(*) 1
2 2
b a
ab b a

T tm c nghim ca pt l x = 0
Cu 5 :
T 2 1 2 1 2 1 , 1 1
2 2
+ s + s s s = + y y x y x
V ) 2 (
2
+ =
+
= y P x
y
x
P thay vo 1
2 2
= + y x
a v pt: 0 1 2 2 2 ) 1 (
2 2 2 2
= + + + P y P y P
Dng iu kin c nghim ca pt bc hai 1 s P
2
2
1
2
2
Max
x
P
y



S GIO DC V O TO K THI TUYN SINH VO 10 - THPT
TNH LO CAI NM HC: 2012 2013
MN: TON
Thi gian: 120 pht (khng k thi gian giao )

Cu I: (2,5 m
1. Thc hin php tnh:
( ) ( )
2 3
3
3
a) 2 10 36 64 b) 2 3 2 5 . + +
2. Cho biu thc: P =
2
3
2a 4 1 1
1 a 1 a 1 a
+

+

a) Tm iu kin ca a P xc nh b) Rt gn biu thc P.
Cu II: 15 m
1. Cho hai hm s bc nht y = -x + 2 v y = (m+3)x + 4. Tm cc gi tr ca m th ca hm s cho
l:
a) Hai ng thng ct nhau
b) Hai ng thng song song.
2. Tm cc gi tr ca a th hm s y = ax
2
(a = 0) i qua im M(-1; 2).
Cu III: 15 m
1. Gii phng trnh x
2
7x 8 = 0
2. Cho phng trnh x
2
2x + m 3 = 0 vi m l tham s. Tm cc gi tr ca m phng trnh c hai
nghim x
1
; x
2
tha mn iu kin
3 3
1 2 1 2
x x x x 6 + =
Cu IV: 15 m
CHNH THC

92
1. Gii h phng trnh
3x 2y 1
.
x 3y 2
=

+ =


2. Tm m h phng trnh
2x y m 1
3x y 4m 1
=

+ = +

c nghim (x; y) tha mn iu kin x + y > 1.


Cu V: (3,0 m Cho na ng trn tm O ng knh AB = 2R v tip tuyn Ax cng pha vi na
ng trn i vi AB. T im M trn Ax k tip tuyn th hai MC vi na ng trn (C l tip im).
AC ct OM ti E; MB ct na ng trn (O) ti D (D khc B).
a) Chng minh AMOC l t gic ni tip ng trn.
b) Chng minh AMDE l t gic ni tip ng trn.
c) Chng mnh ADE ACO =
-------- Ht ---------
HNG DN GII:
Cu I: (2,5 m
1. Thc hin php tnh:
3 3
a) 2 10 36 64 8 100 2 10 12 + = = =
( ) ( )
2 3
3
b) 2 3 2 5 2 3 2 5 3 2 2 5 2 + = + = + =
2. Cho biu thc: P =
2
3
2a 4 1 1
1 a 1 a 1 a
+

+

a) Tm iu kin ca a P xc nh: P xc nh khi a 0 v a 1 > =
b) Rt gn biu thc P.
P =
2
3
2a 4 1 1
1 a 1 a 1 a
+

+
=
( )
( )
( )
( )
( )( )
2 2 2
2
2a 4 1 a a a 1 1 a a a 1
1 a a a 1
+ + + + + +
+ +

=
( )( )
2 2 2 2
2
2a 4 a a 1 a a a a a a 1 a a a a a
1 a a a 1
+ + + +
+ +

=
( )( )
2
2 2a
1 a a a 1

+ +
=
2
2
a a 1 + +

Vy vi a 0 v a 1 > = th P =
2
2
a a 1 + +

Cu II: 15 m
1. Cho hai hm s bc nht y = -x + 2 v y = (m+3)x + 4. Tm cc gi tr ca m th ca hm s cho
l:
a) hm s y = (m+3)x + 4 l hm s bc nht th m + 3 = 0 suy ra m = -3.
th ca hai hm s cho l hai ng thng ct nhau a = a
-1 =m+3 m = -4
Vy vi m = -3 v m = -4 th th ca hai hm s cho l hai ng thng ct nhau.
b) th ca hm s cho l Hai ng thng song song
a a ' 1 m 3
m 4
b b' 2 4
= = +

=

= =

tha mn iu kin m = -3
Vy vi m = -4 th th ca hai hm s cho l hai ng thng song song.

2. Tm cc gi tr ca a th hm s y = ax
2
(a = 0) i qua im M(-1; 2).

93
V th hm s y = ax
2
(a = 0) i qua im M(-1; 2) nn ta thay x = -1 v y = 2 vo hm s ta c phng
trnh 2 = a.(-1)
2
suy ra a = 2 (tha mn iu kin a = 0)
Vy vi a = 2 th th hm s y = ax
2
(a = 0) i qua im M(-1; 2).
Cu III: 15 m
1. Gii phng trnh x
2
7x 8 = 0 c a b + c = 1 + 7 8 = 0 suy ra x
1
= -1 v x
2
= 8

2. Cho phng trnh x
2
2x + m 3 = 0 vi m l tham s. Tm cc gi tr ca m phng trnh c hai
nghim x
1
; x
2
tha mn iu kin
3 3
1 2 1 2
x x x x 6 + = .
phng trnh c hai nghim x
1
; x
2
th A > 0 1 m + 3 > 0 m s 4
Theo viet ta c: x
1
+ x
2
=2 (1) v x
1
. x
2
= m 3 (2)
Theo u bi:
3 3
1 2 1 2
x x x x 6 + = ( )
2
1 2 1 2 1 2
x x x x 2x x + = 6 (3)
Th (1) v (2) vo (3) ta c: (m - 3)(2)
2
2(m-3)=6 2m =12 m = 6 Khng tha mn iu kin m s 4
vy khng c gi tr no ca m phng trnh c hai nghim x
1
; x
2
tha mn iu kin
3 3
1 2 1 2
x x x x 6 + = .

Cu IV: 15 m
1. Gii h phng trnh
3x 2y 1
.
x 3y 2
=

+ =

( ) 3 3y 2 2y 1 7y 7 y 1
x 3y 2 x 1
x 3y 2
= = =



= =
=


2. Tm m h phng trnh
2x y m 1
3x y 4m 1
=

+ = +

c nghim (x; y) tha mn iu kin x + y > 1.


2x y m 1 5x 5m x m x m
3x y 4m 1 2x y m 1 2m y m 1 y m 1
= = = =



+ = + = = = +


M x + y > 1 suy ra m + m + 1 > 1 2m > 0 m > 0.
Vy vi m > 0 th h phng trnh c nghim (x; y) tha mn iu kin x + y > 1.
Cu V: 30 m Cho na ng trn tm O ng knh AB = 2R v tip tuyn Ax cng pha vi na
ng trn i vi AB. T im M trn Ax k tip tuyn th hai MC vi na ng trn (C l tip im).
AC ct OM ti E; MB ct na ng trn (O) ti D (D khc B).
a) Chng minh AMCO l t gic ni tip ng trn.
b) Chng minh AMDE l t gic ni tip ng trn.
c) Chng mnh ADE ACO =
Gii.
a)
0
MAO MCO 90 = = nn t gic AMCO ni tip
b)
0
MEA MDA 90 = = . T gic AMDE c
D, E cng nhn AM di cng mt gc 90
0

Nn AMDE ni tip
c) V AMDE ni tip nn ADE AMEcngchan cung AE =
V AMCO ni tip nn ACO AMEcngchan cung AO =
Suy ra ADE ACO =





D
O
E
M
C
B
A


94






























S GIO DC V O TO
GIA LAI
chnh thc
Ngy thi: 26/6/2012
K THI TUYN SINH VO LP 10 CHUYN
Nm c 2012 2013
Mn thi: Ton (khng chuyn)
Thi gian lm bi: 120 pht

Cu 1. (2,0 im)
Cho biu thc
( )
x 2 x 2
Q x x
x 1
x 2 x 1
| |
+
= +
|
|

+ +
\ .
, vi x 0, x 1 > =
a. Rt gn biu thc Q
b. Tm cc gi tr nguyn ca x Q nhn gi tr nguyn.
Cu 2. (1,5 im)
Cho phng trnh
2
x 2(m 1)x m 2 0 + + = , vi x l n s, m R e
a. Gii phng trnh cho khi m = 2
b. Gi s phng trnh cho c hai nghim phn bit
1
x v
2
x . Tm h thc lin h gia
1
x v
2
x
m khng ph thuc vo m.
Cu 3. (2,0 im)
Cho h phng trnh
(m 1)x (m 1)y 4m
x (m 2)y 2
+ + =

+ =

, vi m R e
a. Gii h cho khi m = 3
CHNH THC

95
b. Tm iu kin ca m phng trnh c nghim duy nht. Tm nghim duy nht .
Cu 4. (2,0 im)
Cho hm s
2
y x = c th (P). Gi d l ng thng i qua im M(0;1) v c h s gc k.
a. Vit phng trnh ca ng thng d
b. Tm iu kin ca k t d ct th (P) ti hai im phn bit.
Cu 5. (2,5 im)
Cho tam gic nhn ABC (AB < AC < BC) ni tip trong ng trn (O). Gi H l giao im ca hai
ng cao BD v CE ca tam gic ABC (D AC, E AB) e e
a. Chng minh t gic BCDE ni tip trong mt ng trn
b. Gi I l im i xng vi A qua O v J l trung im ca BC. Chng minh rng ba im H, J, I
thng hng
c. Gi K, M ln lt l giao im ca AI vi ED v BD. Chng minh rng
2 2 2
1 1 1
DK DA DM
= +

HNG DN GII:

Cu 1.
a.
( )
x 2 x 2
Q x x
x 1
x 2 x 1
| |
+
= +
|
|

+ +
\ .

( )
( )( )
( )
| |
+
|
= +
|
+
| +
\ .
2
x 2 x 2
x x 1
x 1 x 1
x 1

| |
+
=
|
|
+
\ .
x 2 x 2
x
x 1 x 1
| |
+ +
=
|
|
+
\ .
x 1 1 x 1 1
x
x 1 x 1
| |
= + +
|
+
\ .
1 1
1 1 x
x 1 x 1

| |
= +
|
+
\ .
1 1
x
x 1 x 1

+ +
=

x 1 x 1
. x
x 1
=

2 x
. x
x 1
=

2x
x 1

Vy =

2x
Q
x 1


b.
Q nhn gi tr nguyn

+
= = = +

2x 2x 2 2 2
Q 2
x 1 x 1 x 1

e Q khi e

2
x 1
khi 2 chia ht cho x 1

=

x 1 1
x 1 2
=

x 0
x 2
x 1
x 3
i chiu iu kin th
x 2
x 3
=



Cu 2. Cho pt
2
x 2(m 1)x m 2 0 + + = , vi x l n s, m R e
a. Gii phng trnh cho khi m = 2
Ta c phng trnh
2
x 2x 4 0 + =

2 2
x 2x 4 0 x 2x 1 5 + = + + = ( )
( )
2
2
x 1 5 5 + = =
x 1 5 + =
x 1 5 x 1 5
x 1 5 x 1 5

+ = =

+ = = +


Vy phng trinh c hai nghim x 1 5 = v x 1 5 = +


96
b.
Theo Vi-et, ta c
1 2
1 2
x x 2m 2 (1)
x x m 2 (2)
+ = +

1 2
1 2
x x 2m 2
m x x 2
+ = +


= +

( )
1 2 1 2
1 2
x x 2 x x 2 2
m x x 2
+ = + +


= +


Suy ra
( )
1 2 1 2
x x 2 x x 2 2 + = + +
1 2 1 2
x x 2x x 6 0 + =

Cu 3. Cho h phng trnh
(m 1)x (m 1)y 4m
x (m 2)y 2
+ + =

+ =

, vi m R e
a. G co k m = 3
Ta c h phng trnh
2x 2y 12
x 5y 2
+ =


x y 6
x 5y 2
+ =


x 7
y 1
=


Vy h phng trnh c nghim
( ) x; y vi
( ) 7;1

b. u kn c ngm ca png trn

( ) m 1
m 1
1 m 2
+
+
=

( )( ) ( ) m 1 m 2 m 1 + = +

( )( ) ( ) m 1 m 2 m 1 0 + + + = ( )( ) m 1 m 1 0 + =

m 1 0
m 1 0
+ =

m 1
m 1
=


Vy phng trnh c nghim khi m 1 = v m 1 =
Gii h phng trnh
(m 1)x (m 1)y 4m
x (m 2)y 2
+ + =

+ =

khi
m 1
m 1
=


(m 1)x (m 1)y 4m
x (m 2)y 2
+ + =

+ =

+ =

4m
x y
m 1
x (m 2)y 2

= +

+
4m
x y
m 1
2
y
m 1


=

+
4m 2
x
m 1
2
y
m 1
.
Vy h c nghim (x; y) vi
| |
|
+ +
\ .
4m 2 2
;
m 1 m 1


Cu 4.
a. Vt png trn ca ng tng d
ng thng d vi h s gc k c dng y kx b = +
ng thng d i qua im M(0; 1) nn 1 k.0 b = + b 1 =
Vy d: y kx 1 = +

b.
Phng trnh honh giao im ca (P) v d

2
x kx 1 = +
2
x kx 1 0 + + = , c
2
k 4 A =
d ct (P) ti hai im phn bit khi 0 A >

2
k 4 0 >
2
k 4 >
2 2
k 2 > k 2 >
k 2
k 2
<


>



Cu 5.
a. BCDE ni tip

0
BEC BDC 90 = =
Suy ra BCDE ni tip ng trn
ng knh BC



b. H, J, I thng hng

97
IB AB; CE AB (CH AB)
Suy ra IB // CH
IC AC; BD AC (BH AC)
Suy ra BH // IC
Nh vy t gic BHCI l hnh bnh hnh
J trung im BC J trung im IH
Vy H, J, I thng hng
c.
1
ACB AIB AB
2
= =
ACB DEA = cng b vi gc DEB ca t gic ni tip BCDE

0
BAI AIB 90 + = v AABI vung ti B
Suy ra
0
BAI AED 90 + = , hay
0
EAK AEK 90 + =
Suy ra AAEK vung ti K
Xt AADM vung ti M (suy t gi thit)
DK AM (suy t chng minh trn)www.VNMATH.
Nh vy
2 2 2
1 1 1
DK DA DM
= +













S GIO DC V O TO K THI TUYN SINH LP 10 THPT
QUNG NINH NM HC 2012 2013

MN: TON(Dng cho mi th sinh d thi)
Ngy thi: 28/6/2012
Thi gian lm bi: 120 pht (Khng k thi gian giao )
( thi ny c 01 trang)
Cu I. (2,0 im)
1) Rt gn cc biu thc sau:
a) A =
1
2 18
2
+ b) B =
1 1 2
1 1 1 x x x
+
+
vi x > 0, x = 1
2. Gii h phng trnh:
2x 5
2 4
y
x y
+ =

+ =


Cu II. 20 m
Cho phng trnh (n x): x
2
ax 2 = 0 (*)
1. Gii phng trnh (*) vi a = 1.
2. Chng minh rng phng trnh (*) c hai nghim phn bit vi mi gi tr ca a.
CHNH THC

98
3. Gi x1, x2 l hai nghim ca phng trnh (*). Tm gi tr ca a biu thc:
N=
2 2
1 1 2 2
( 2)( 2) x x x x + + + + c gi tr nh nht.
Cu III. (2,0 im)Gii bi ton bng cch lp phng trnh hoc h phng trnh.
Qung ng sng AB di 78 km. Mt chic thuyn my i t A v pha B. Sau 1 gi, mt chic
ca n i t B v pha A. Thuyn v ca n gp nhau ti C cch B 36 km. Tnh thi gian ca thuyn, thi gian
ca ca n i t lc khi hnh n khi gp nhau, bit vn
tc ca ca n ln hn vn tc ca thuyn l 4 km/h.
Cu IV. (3,5 im)
Cho tam gic ABC vung ti A, trn cnh AC ly im D (D A, D C). ng trn (O)
ng knh DC ct BC ti E (E C).
1. Chng minh t gic ABED ni tip.
2. ng thng BD ct ng trn (O) ti im th hai I. Chng minh ED l tia phn gic ca gc AEI.
3. Gi s tg ABC = 2 Tm v tr ca D trn AC EA l tip tuyn ca ng trn ng knh DC.
CuV. (0.5 im) Gii phng trnh:
7 2 (2 ) 7 x x x x + = +

HNG DN GII:
Cu IV :
c. EA l tip tuyn ca .Trn, . knh CD th gc E
1
= gc C
1
(1)
M t gic ABED ni tip nn gc E
1
= gc B
1
(2)
T (1) v (2) gc C
1
= gc B
1
ta li c gc BAD chung nn
AABD ~ AACB
AB
AD
AC
AB
= AB
2
= AC.AD AD =
AC
AB
2
( I )
Theo bi ra ta c : tan (ABC) =
AB
AC
= 2 nn
2
1
AC
AB
( II )
T (I) v (II) AD =
2
AB
.
Vy AD =
2
AB
th EA l tip tuyn ca T, knh CD



Cu V:
Gii phng trnh: 7 2 (2 ) 7 x x x x + = +
t t x = 7 ; v x = K v, t 0
t v v t ). 2 ( 2
2
+ = + ... 0 ) 2 )( ( = t v t v t = hoc t=2
Nu t= 2 th 2 7 = x x = 3 (TM)
Nu t = v th x x = 7 x = 3,5







99








































S GIO DC V O TO
K THI TUYN SINH VO LP 10 THPT
KHNH HA
NM HC 2011 - 2012
Mn thi: TON
Ngy thi : 21/06/2011
Thi gian lm bi: 120 pht

Bi 1( 2 im)
1) n gin biu thc: A
2 3 6 8 4
2 3 4
+ + + +
=
+ +

CHNH THC

100
2) Cho biu thc:
1 1
( ); ( 1)
1 1
P a a
a a a a
= >
+

Rt gn P v chng t P >0

Bi 2( 2 im)
1) Cho phng trnh bc hai x
2
+ 5x + 3 = 0 c hai nghim x
1
; x
2
. Hy lp mt phng trnh bc
hai c hai nghim (x
1
2
+ 1 ) v ( x
2
2
+ 1).
2) Gii h phng trnh
2 3
4
2
4 1
1
2
x y
x y

+ =


Bi 3( 2 im)
Qung ng t A n B di 50km.Mt ngi d nh i xe p t A n B vi vn tc khng
i.Khi i c 2 gi,ngi y dng li 30 pht ngh.Mun n B ng thi gian nh,ngi
phi tng vn tc thm 2 km/h trn qung ng cn li.Tnh vn tc ban u ca ngi i xe
p.

Bi 4( 4 im)
Cho tam gic ABC c ba gc nhn v H l trc tm.V hnh bnh hnh BHCD.ng thng i qua
D v song song BC ct ng thng AH ti E.
1) Chng minh A,B,C,D,E cng thuc mt ng trn
2) Chng minh BAE DAC Z =Z
3) Gi O l tm ng trn ngoi tip tam gic ABC v M l trung im ca BC,ng thng
AM ct OH ti G.Chng minh G l trng tm ca tam gicABC.
4) Gi s OD = a.Hy tnh di ng trn ngoi tip tam gic BHC theo a














HNG DN GII:
Bi 1
3) A
2 3 2 6 8 2 ( 2 3 4)(1 2)
1 2
2 3 4 2 3 4
+ + + + + + + +
= = = +
+ + + +


101
4)
2
1 1
( ); 1
1
2 1 1 2 1 1; : 1
( 1 1) 0; 1
a a a a
P a a
a a
a a a a vi a
P a a
+ +
= >
+
= = + >
= > >

Bi 2 x
2
+ 5x + 3 = 0
1) C 25 12 13 0 A= = >
Nn pt lun c 2 nghim phn bit
x
1
+ x
2
= - 5 ; x
1
x
2
= 3
Do S = x
1
2
+ 1 + x
2
2
+ 1 = (x
1
+ x
2
)
2
- 2 x
1
x
2
+ 2 = 25 6 + 2 = 21
V P = (x
1
2
+ 1) (x
2
2
+ 1) = (x
1
x
2
)
2
+ (x
1
+ x
2
)
2
- 2 x
1
x
2
+ 1 = 9 + 20 = 29
Vy phng trnh cn lp l x
2
21x + 29 = 0
2) K 0; 2 x y = =
2 3
14
4 2 7
2 2
3
2 3 1 4 12 3 3
4
3 2
2
2
x
x x y
x
y
y
x y
x y


+ = = =


=


+ = =


+ =
=


Vy HPT c nghim duy nht ( x ;y) = ( 2 ;3)
Bi 3 :
Gi x(km/h) l vtc d nh; x > 0 ; c 30 pht = (h)
Th gian d nh :
50
( ) h
x

Qung ng i c sau 2h : 2x (km)
Qung ng cn li : 50 2x (km)
Vn tc i trn qung ng cn li : x + 2 ( km/h)
Th gian i qung ng cn li :
50 2
( )
2
x
h
x

+

Theo bi ta c PT:
1 50 2 50
2
2 2
x
x x

+ + =
+

Gii ra ta c : x = 10 (tha K bi ton)
Vy Vn tc d nh : 10 km/h
Bi 4 :
Gii cu c)
V BHCD l HBH nn H,M,D thng hng
Tam gic AHD c OM l TBnh => AH = 2 OM
V AH // OM
2 tam gic AHG v MOG c
( ) HAG OMG slt Z = Z
AGH MGO Z = Z ( )

( )
2
AHG MOG G G
AH AG
MO MG
A A
= =

Hay AG = 2MG
Tam gic ABC c AM l trung tuyn; G e AM
Do G l trng tm ca tam gic ABC
d) BHC BDC A = A ( v BHCD l HBH)
c B ;D ;C ni tip (O) bn knh l a
A
B C
E
D

H
O
M
G

102
Nn tam gic BHC cng ni tip (K) c bn knh a
Do C
(K)
= 2 a t ( VD)
S GIO DC-O TO K THI TUYN SINH VO 10 THPT NM 2012
NH NH Kha ngy 29 thng 6 nm 2012

Mn thi: TON
Ngy thi: 30/6/2012
Thi gian lm bi: 120 pht (khng k thi gian giao )

Bi 1: (3, 0 im)
Hc sinh khng s dng my tnh b ti
a) Gii phng trnh: 2x 5 = 0
b) Gii h phng trnh:
y x 2
5x 3y 10
=


c) Rt gn biu thc
2
5 a 3 3 a 1 a 2 a 8
A
a 4
a 2 a 2
+ + +
= +

+
vi a 0, a 4 > =
d) Tnh gi tr ca biu thc B 4 2 3 7 4 3 = + +
Bi 2: (2, 0 im)
Cho parabol (P) v ng thng (d) c phng trnh ln lt l
2
y mx = v
( ) 2 1 y m x m = + (m l tham s, m = 0).
a) Vi m = 1 , tm ta giao im ca (d) v (P).
b) Chng minh rng vi mi m = 0 ng thng (d) lun ct parabol (P) ti hai im phn bit.
Bi 3: (2, 0 im)
Qung ng t Quy Nhn n Bng Sn di 100 km. Cng mt lc, mt xe my khi hnh t Quy
Nhn i Bng Sn v mt xe t khi hnh t Bng Sn i Quy Nhn. Sau khi hai xe gp nhau, xe my i 1
gi 30 pht na mi n Bng Sn. Bit vn tc hai xe khng thay i trn sut qung ng i v vn tc
ca xe my km vn tc xe t l 20 km/h. Tnh vn tc mi xe.
Bi 4: (3, 0 im)
Cho ng trn tm O ng knh AB = 2R. Gi C l trung im ca OA, qua C k dy MN vung
gc vi OA ti C. Gi K l im ty trn cung nh BM, H l giao im ca AK v MN.
a) Chng minh t gic BCHK l t gic ni tip.
b) Chng minh AK.AH = R
2

c) Trn KN ly im I sao cho KI = KM, chng minh NI = KB.


HNG DN GII:
Bi 1:
a) 2x 5 = 0
5
2 5 0 2 5
2
x x x = = =
b)
y x 2 5x 5y 10 2y 20 y 10
5x 3y 10 5x 3y 10 y x 2 x 8
= + = = =


= = = =


c)
CHNH THC

103

( )( ) ( )( ) ( )
( )( )
( )( ) ( )( )
( )
( )( )
2
2
2
2 2
5 a 3 a 2 3 a 1 a 2 a 2 a 8
5 a 3 3 a 1 a 2 a 8
A
a 4 a 2 a 2
a 2 a 2
a 8a 16
5a 10 a 3 a 6 3a 6 a a 2 a 2 a 8 a 8a 16
a 2 a 2 a 2 a 2 a 2 a 2
+ + + + +
+ + +
= + =
+
+
+
+ + + +
= = =
+ + +

( )
( )
2
a 4
a 4 4 a
a 4

= = =


d)
( ) ( )
2 2
B 4 2 3 7 4 3 3 1 2 3 3 1 2 3 3 1 2 3 3 = + + = + + = + + = + + =
Bi 2:
a) Vi 1 m =
( ) P v
( ) d ln lt tr thnh
2
; 2 y x y x = = .
Lc phng trnh honh giao im ca
( ) P v
( ) d l:
2 2
2 2 0 x x x x = + = c
1 1 2 0 a b c + + = + = nn c hai nghim l
1 2
1; 2 x x = = .
Vi
1 1
1 1 x y = =
Vi
2 2
2 4 x y = =
Vy ta giao im ca
( ) P v
( ) d l
( ) 1; 1 v
( ) 2; 4 .
b) Phng trnh honh giao im ca
( ) P v
( ) d l:
( ) ( ) ( )
2 2
2 1 2 1 0 * mx m x m mx m x m = + + = .
Vi 0 m = th
( ) * l phng trnh bc hai n x c
( ) ( )
2
2 2 2
2 4 1 4 4 4 4 5 4 0 m m m m m m m m A = + = + + = + > vi mi m. Suy ra
( ) * lun c hai
nghim phn bit vi mi m. Hay vi mi m = 0 ng thng (d) lun ct parabol (P) ti hai im phn bit.
Bi 3:
i
'
1 30 1, 5 h h =
t a im :
- Quy Nhn l A
- Hai xe gp nhau l C
- Bng Sn l B
Gi vn tc ca xe my l
( ) / x km h . K : 0 x > .
Suy ra :
Vn tc ca t l
( ) 20 / x km h + .
Qung ng BC l :
( ) 1,5x km
Qung ng AC l :
( ) 100 1,5x km
Thi gian xe my i t A n C l : ( )
100 1, 5x
h
x


Thi gian t my i t B n C l : ( )
1, 5
20
x
h
x +

V hai xe khi hnh cng lc, nn ta c phng trnh :
100 1, 5 1, 5
20
x x
x x

=
+

Gii pt :
100-1,5x
1,5x
A
B C

104
( )( )
2 2 2
2
100 1, 5 1, 5
100 1, 5 20 1, 5 100 2000 1, 5 30 1, 5
20
3 70 2000 0
x x
x x x x x x x
x x
x x

= + = + =
+
=

2
' 35 3.2000 1225 6000 7225 0 ' 7225 85 A = + = + = > A = =
Phng trnh c hai nghim phn bit :
1
35 85
40
3
x
+
= = (tha mn K)

2
35 85 50
3 3
x

= = (khng tha mn K)
Vy vn tc ca xe my l 40 / km h.
Vn tc ca t l
( ) 40 20 60 / km h + = .
Bi 4:
a) T gic BCHK l t gic ni tip.
Ta c :
0
90 AKB = (gc ni tip chn na ng trn)
hay ( )
0 0
90 ; 90 HKB HCB gt = =
T gic BCHK c
0 0 0
90 90 180 HKB HCB + = + =
t gic BCHK l t gic ni tip.
b)
2
. AK AH R =
D thy ( )
2
ACH AKB . . . 2
2
AC AH R
g g AK AH AC AB R R
AK AB
= = = =
c) NI KB =
OAM A c
( ) OA OM R gt OAM = = A cn ti
( ) 1 O
OAM A c MC l ng cao ng thi l ng trung tuyn (gt) OAM A cn ti
( ) 2 M
( ) ( ) 1 & 2 OAM A l tam gic u
0 0 0
60 120 60 MOA MON MKI = = =
KMI A l tam gic cn (KI = KM) c
0
60 MKI = nn l tam gic u
( ) 3 MI MK = .
D thy BMK A cn ti B c
0 0
1 1
120 60
2 2
MBN MON = = = nn l tam gic u
( ) 4 MN MB =
Gi E l giao im ca AK v MI.
D thy
0
0
60
60
NKB NMB
NKB MIK
MIK

= =

=
`
=

)
KB // MI (v c cp gc v tr so le trong bng nhau)
mt khc
( ) AK KB cmt nn AK MI ti E
0
90 HME MHE = .
Ta c : ( )
( )
0
0
90
90
dd
HAC AHC
HME MHE cmt HAC HME
AHC MHE

= =
`

=

)
mt khc HAC KMB = (cng chn KB )
HME KMB = hay ( ) 5 NMI KMB = T
( ) ( ) ( ) ( ) 3 , 4 & 5 . . IMN KMB c g c NI KB A = A = (pcm)







E
I
H
N
M
C
A
O
B
K

105








S GIO DC V O TO THI TUYN SINH LP 10 THPT
BC GIANG NM HC 2012 2013
Mn thi : Ton
Thi gian : 120 pht khng k thi gian giao
Ngy thi 30 thng 6 nm 2012

Cu 1. (2 m
1.Tnh
1
2
2 1
-
-

2 .Xc nh gi tr ca a,bit th hm s y = ax - 1 i qua im M(1;5)

Cu 2: (3 m
1.Rt gn biu thc:
1 2 3 2
( ).( 1)
2 2 2
a a
A
a a a a
- +
= - +
- - -
vi a>0,a 4
2.Gii h pt:
2 5 9
3 5
x y
x y

- =

+ =


3. Chng minh rng pt:
2
1 0 x mx m + + - = lun c nghim vi mi gi tr ca m.
Gi s x
1
,x
2
l 2 nghim ca pt cho,tm gi tr nh nht ca biu thc
2 2
1 2 1 2
4.( ) B x x x x = + - +

Cu 3: (1,5 m
Mt t ti i t A n B vi vn tc 40km/h. Sau 2 gi 30 pht th mt t taxi cng xut pht i t A
n B vi vn tc 60 km/h v n B cng lc vi xe t ti.Tnh di qung ng AB.
Cu 4: (3 m
Cho ng trn (O) v mt im A sao cho OA=3R. Qua A k 2 tip tuyn AP v AQ ca ng trn
(O),vi P v Q l 2 tip im.Ly M thuc ng trn (O) sao cho PM song song vi AQ.Gi N l giao im
th 2 ca ng thng AM v ng trn (O).Tia PN ct ng thng AQ ti K.
1.Chng minh APOQ l t gic ni tip.
2.Chng minh KA
2
=KN.KP
3.K ng knh QS ca ng trn (O).Chng minh tia NS l tia phn gic ca gc PNM .
4. Gi G l giao im ca 2 ng thng AO v PK .Tnh di on thng AG theo bn knh R.
Cu 5: (0,5m
Cho a,b,c l 3 s thc khc khng v tho mn:

2 2 2
2013 2013 2013
( ) ( ) ( ) 2 0
1
a b c b c a c a b abc
a b c

+ + + + + + =

+ + =


Hy tnh gi tr ca biu thc
2013 2013 2013
1 1 1
Q
a b c
= + +

HNG DN CHM (tham kho)
CHNH THC

106
Cu Ni dung im
1 1
2
1 2 1 2 1
2 2 2 2 1 2 1
2 1 ( 2 1).( 2 1) ( 2) 1)
+ +
- = - = - = + - =
- - + -

KL:

1
2 Do th hm s y = ax-1 i qua M(1;5) nn ta c a.1-1=5 a=6
KL:
1
2 1
2 ( 1).( 2)
( ).( 1)
( 2) ( 2) 2
2 1
( ).( 1 1) . 1
( 2)
a a a
A
a a a a a
a
a a
a a a
- -
= - + =
- - -
-
= - + = =
-

KL:

0,5


0,5
2
2 5 9 2 5 9 2 5 9 1
3 5 15 5 25 17 34 2
x y x y x y y
x y x y x x

- = - = - = = -





+ = + = = =


KL:

1
3
Xt Pt:
2
1 0 x mx m + + - =
2 2 2
4( 1) 4 4 ( 2) 0 m m m m m = - - = - + = -
Vy pt lun c nghim vi mi m
Theo h thc Viet ta c
1 2
1 2
1
x x m
x x m

+ = -

= -


Theo bi
2 2 2
1 2 1 2 1 2 1 2 1 2
2 2 2
2
4.( ) ( ) 2 4.( )
2( 1) 4( ) 2 2 4 2 1 1
( 1) 1 1
B x x x x x x x x x x
m m m m m m m m
m
= + - + = + - - +
= - - - - = - + + = + + +
= + +

Vy minB=1 khi v ch khi m = -1
KL:

0,25



0,25






0,5
3 Gi di qumg ng AB l x (km) x>0
Thi gian xe ti i t A n B l
40
x
h
Thi gian xe Taxi i t A n B l :
60
x
h
Do xe ti xut pht trc 2h30pht =
5
2
nn ta c pt

5
40 60 2
3 2 300
300
x x
x x
x
- =
- =
=

Gi tr x = 300 c tho mn K

0,25

0,25

0,25


0,25

0,25


0,25

107
Vy di qung ng AB l 300 km.
4 1
Xt t gic APOQ c
0
90 APO= (Do AP l tip tuyn ca (O) P)
0
90 AQO= (Do AQ l tip tuyn ca (O) Q)
0
180 APO AQO + = ,m hai gc ny l 2 gc i nn t gic APOQ l t gic ni
tip













0,75
2
Xt AKN v PAK c AKP l gc chung
APN AMP = ( Gc ntcng chn cung NP)
M NAK AMP = (so le trong ca PM //AQ
AKN ~ PKA (gg)
2
.
AK NK
AK NK KP
PK AK
= = (pcm)



0,75
3 K ng knh QS ca ng trn (O)
Ta c AQ^ QS (AQ l tt ca (O) Q)
M PM//AQ (gt) nn PM^ QS
ng knh QS ^ PM nn QS i qua im chnh gia ca cung PM nh
sd PS sdSM = PNS SNM = (hai gc nt chn 2 cung bng nhau)
Hay NS l tia phn gic ca gc PNM


0,75
4
Chng minh c AQO vung Q, c QG^ AO(theo Tnh cht 2 tip tuyn ct
nhau)
Theo h thc lng trong tam gic vung ta c
2 2
2
1
.
3 3
1 8
3
3 3
OQ R
OQ OI OA OI R
OA R
AI OA OI R R R
= = = =
= - = - =

Do KNQ ~ KQP (gg)
2
. KQ KN KP = m
2
. AK NK KP = nn AK=KQ
Vy APQ c cc trung tuyn AI v PK ct nhau G nn G l trng tm
2 2 8 16
.
3 3 3 9
AG AI R R = = =


0,75
5 Ta c:




0,25


G
K
N
S
M
I
Q
P
A
O

108

2 2 2
2 2 2 2 2 2
2 2 2 2 2 2
2 2
2
( ) ( ) ( ) 2 0
2 0
( ) ( ) (2 ) 0
( ) ( ) ( ) 0
( )( ) 0
( ).( ).( ) 0
a b c b c a c a b abc
a b a c b c b a c a c b abc
a b b a c a c b abc b c a c
ab a b c a b c a b
a b ab c ac bc
a b a c b c
+ + + + + + =
+ + + + + + =
+ + + + + + =
+ + + + + =
+ + + + =
+ + + =

*TH1: nu a+ b=0
Ta c
2013 2013 2013
1 1
a b a b
c a b c
= - = -



= + + =

ta c
2013 2013 2013
1 1 1
1 Q
a b c
= + + =
Cc trng hp cn li xt tng t
Vy
2013 2013 2013
1 1 1
1 Q
a b c
= + + =


0,25
S GIO DC V O TO THI TUYN SINH LP 10 THPT
YN BI NM HC 2012 2013
Mn thi : TON
Thi gian : 120 pht (khng k thi gian giao )
Kha ngy 23 thng 6 nm 2012
( thi c 01 trang, gm 05 cu)
Cu 1: 20 m
1. Cho hm s y = x + 3 (1)
a. Tnh gi tr ca y khi x = 1
b. V th ca hm s (1)
2. Gii phng trnh: 4x
2

7x + 3 = 0
Cu 2: 20 m
Cho biu thc M =
1
3 x
+
x
3+ x

x+9
x9

1. Tm iu kin ca x biu thc M c ngha. Rt gn biu thc M.
2. Tm cc gi tr ca x M > 1
Cu 3: 20 m
Mt i th m phi khai thc 260 tn than trong mt thi hn nht nh. Trn thc t, mi ngy i
u khai thc vt nh mc 3 tn, do h khai thc c 261 tn than v xong trc thi hn mt
ngy.
Hi theo k hoch mi ngy i th phi khai thc bao nhiu tn than?
Cu 4: 30 m
Cho na ng trn tm O, ng knh AB = 12 cm. Trn na mt phng b AB cha na ng trn
(O) v cc tia tip tuyn Ax, By. M l mt im thuc na ng trn (O), M khng trng vi A v B. AM
ct By ti D, BM ct Ax ti C. E l trung im ca on thng BD.
1. Chng minh: AC . BD = AB
2

.
2. Chng minh: EM l tip tuyn ca na ng trn tm O.
3. Ko di EM ct Ax ti F. Xc nh v tr ca im M trn na ng trn tm O sao cho din tch t gic
AFEB t gi tr nh nht? Tm gi tr nh nht .
Cu 5: (1,0 im)
Tnh gi tr ca biu thc T = x
2

+ y
2

+ z
2

7 bit:
x + y + z = 2 x34 + 4 y21 + 6 z4 + 45




CHNH THC

109
















S GIO DC V O TO K THI TUYN SINH L 10 THPT
LM NG MN THI : TON
Kha ngy : 26 thng 6 nm 2012
( thi gm 01 trang) Thi gian lm bi : 120 pht

Cu 1: (0,75) Tnh : 18 2 2 32 +
Cu 2: (0,75) Gii h phng trnh :
2 3 1
4 3 11
x y
x y
=

+ =


Cu 3: (0,75) Cho tam gic ABC vung ti A, ng cao AH. Bit BH = 9cm, Ch = 16cm.
Tnh di cc on thng AH, BH, AC.
Cu 4: (0,75) Cho hai ng thng (d) : y = (m-3)x + 16 (m= 3) v (d): y = x + m
2
.
Tm m (d) v (d) ct nhau ti mt im trn trc tung
Cu 5: (0,75) Cho AB l dy cung ca ng trn tm O bn knh 12cm. Bit AB = 12cm . Tnh
din tch hnh qut to bi hai bn knh OA, OB v cung nh AB.
Cu 6: (1) Cho hm s y = ax
2
(a = 0) c th (P).
a) Tm a bit (P) i qua im A(2;4)
b) Tm k ng thng (d) : y = 2x + k lun ct (P) ti 2 im phn bit.
Cu 7: (0,75) Hnh nn c th th tch l 320t cm
3
, bn knh ng trn l 8cm. Tnh din tch ton
phn ca hnh nn .
Cu 8: (1) Cho ng trn (O) ng knh AB, M l trung im ca OA. Qua M v dy cung CD
vung gc vi OA.
a) Chng minh t gic ACOD l hnh thoi .
b) Tia CO ct BD ti I. Chng minh t gic DIOM ni tip.
Cu 9: (1) Hai i cng nhn cng o mt con mng . Nu h cng lm th trong 8 gi xong
vic. Nu h lm ring th i A hon thnh cng vic nhanh hn i B 12 gi. Hi nu
lm ring th mi i phi lm trong bao nhiu gi mi xong vic.
Cu 10: (0,75) Rt gn : 37 20 3 37 20 3 + +
Cu 11: (1) Cho phng trnh : x
2
2(m-2)x - 3m
2
+2 = 0 (x l n, m l tham s )
Tm m phng trnh c 2 nghim x
1
; x
2
tha : x
1
(2-x
2
) +x
2
(2-x
1
) = -2
Cu 12: (0,75) Cho na ng trn (O) ng knh AB, v cc tip tuyn Ax v By cng pha vi
na ng trn , M l im chnh gia cung AB, N l mt im thuc on OA

( ) , N O N A = = . ng thng vung gc vi MN ti M ct Ax v By ln lt ti C v D.
Chng minh : AC = BN
CHNH THC

110



111

112

113
S GIO DC V O TO K THI TUYN SINH VO LP 10 NM HC 2012-2013
QUNG NGI Mn thi: Ton (khng chuyn)
Thi gian lm bi: 120 pht (khng k thi gian giao )


Bi 1: (1,5 im)
1/ Thc hin php tnh:
( )( )
2 1 2 1 +
2/ Gii h phng trnh:
1
2 3 7
x y
x y
=

+ =


3/ Gii phng trnh:
2
9 8 1 0 x x + =
Bi 2: (2,0 im)
Cho parapol
( )
2
: P y x = v ng thng
( )
2
: 2 1 d y x m = + + (m l tham s).
1/ Xc nh tt c cc gi tr ca m
( ) d song song vi ng thng
( )
2 2
' : 2 d y m x m m = + + .
2/ Chng minh rng vi mi m,
( ) d lun ct
( ) P ti hai im phn bit A v B.
3/ K hiu ;
A B
x x l honh ca im A v im B. Tm m sao cho
2 2
14
A B
x x + = .
Bi 3: (2,0 im)
Hai xe t cng i t cng Dung Qut n khu du lch Sa Hunh, xe th hai n sm hn xe th nht l 1
gi. Lc tr v xe th nht tng vn tc thm 5 km mi gi, xe th hai vn gi nguyn vn tc nhng dng
li ngh mt im trn ng ht 40 pht, sau v n cng Dung Qut cng lc vi xe th nht. Tm vn
tc ban u ca mi xe, bit chiu di qung ng t cng Dung Qut n khu du lch Sa Hunh l 120 km
v khi i hay v hai xe u xut pht cng mt lc.
Bi 4: (3,5 im)
Cho ng trn tm O ng knh AB = 2R v C l mt im nm trn ng trn sao cho CA > CB.
Gi I l trung im ca OA. V ng thng d vung gc vi AB ti I, ct tia BC ti M v ct on AC ti P;
AM ct ng trn (O) ti im th hai K.
1/ Chng minh t gic BCPI ni tip c trong mt ng trn.
2/ Chng minh ba im B, P, K thng hng.
3/ Cc tip tuyn ti A v C ca ng trn (O) ct nhau ti Q. Tnh din tch ca t gic QAIM theo R
khi BC = R.
Bi 5: (1,0 im)
Cho 0, 0 x y > > tha mn
2 2
1 x y + = . Tm gi tr nh nht ca biu thc
2
1
xy
A
xy

=
+
.
-------------- HT --------------

HNG DN GII:
Bi 1:
1/
( )( ) ( )
2
2
2 1 2 1 2 1 2 1 1 + = = =
2/
1 3 3 3 5 10 2
2 3 7 2 3 7 1 1
x y x y x x
x y x y x y y
= = = =


+ = + = = =


3/ Phng trnh
2
9 8 1 0 x x + = c 9 8 1 0 a b c + = = nn c hai nghim l:
1 2
1
1;
9
x x = = .
Bi 2:
1/ ng thng
( )
2
: 2 1 d y x m = + + song song vi ng thng
( )
2 2
' : 2 d y m x m m = + + khi
CHNH THC

114
2 2
2 2
1
2 2 1
1 1
1 1
1
m
m m
m m
m m m m
m
=
= =

= =

= + = +


2/ Phng trnh honh giao im ca
( ) d v
( ) P l
2 2 2 2
2 1 2 1 0 x x m x x m = + + = l phng
trnh bc hai c
2
1 0 ac m = < vi mi m nn lun c hai nghim phn bit vi mi m. Do
( ) d lun ct
( ) P ti hai im phn bit A v B vi mi m.
3/ Cch 1: K hiu ;
A B
x x l honh ca im A v im B th ;
A B
x x l nghim ca phng trnh
2 2
2 1 0 x x m = .
Gii phng trnh
2 2
2 1 0 x x m = .
2 2 2
' 1 1 2 0 ' 2 m m m A = + + = + > A = +
Phng trnh c hai nghim l
2 2
1 2; 1 2
A B
x m x m = + + = + .
Do
( ) ( )
2 2
2 2 2 2 2 2 2 2
2 2 2
14 1 2 1 2 14 1 2 2 2 1 2 2 2 14
2 6 14 2 8 4 2
A B
x x m m m m m m
m m m m
+ = + + + + = + + + + + + + + =
+ = = = =

Cch 2: K hiu ;
A B
x x l honh ca im A v im B th ;
A B
x x l nghim ca phng trnh
2 2
2 1 0 x x m = . p dng h thc Viet ta c:
2
2
. 1
A B
A B
S x x
P x x m
= + =

= =

do
( ) ( )
2
2 2 2 2 2
14 2 . 14 2 2 1 14 4 2 2 14 2
A B A B A B
x x x x x x m m m + = + = = + + = =
Bi 3:
Gi vn tc ban u ca xe th nht l x (km/h), xe th hai l y (km/h). K: x > 0; y > 0.
Thi gian xe th nht i t cng Dung Qut n khu du lch Sa Hunh l ( )
120
h
x
.
Thi gian xe th hai i t cng Dung Qut n khu du lch Sa Hunh l ( )
120
h
y
.
V xe th hai n sm hn xe th nht l 1 gi nn ta c phng trnh: ( )
120 120
1 1
x y
=
Vn tc lc v ca xe th nht l x+ 5 (km/h).
Thi gian xe th nht v t khu du lch Sa Hunh n cng Dung Qut ( )
120
5
h
x +
.
Thi gian xe th hai v t khu du lch Sa Hunh n cng Dung Qut ( )
120
h
y
.
V xe th hai dng li ngh ht
2
40
3
ph h = , sau v n cng Dung Qut cng lc vi xe th nht nn
ta c phng trnh: ( )
120 120 2
2
5 3 x y
=
+
.
T (1) v (2) ta c hpt:
120 120
1
120 120 2
5 3
x y
x y



115
Gii hpt: ( ) ( )
2
120 120
1
120 120 1
360 5 360 5 5 1800 0
120 120 2 5 3
5 3
x y
x x x x x x
x x
x y

= + = + + =


25 4.1800 7225 0 85 A = + = > A = .
Phng trnh c hai nghim phn bit:
1
5 85
40
2
x
+
= = (tha mn K)

2
5 85
45
2
x

= = (khng tha mn K)
Thay 40 x = vo pt (1) ta c:
120 120 120
1 2 60
40
y
y y
= = = (tha mn K).
Vy vn tc ban u ca xe th nht l 40 km/h, xe th hai l 60 km/h.
Bi 4:(Bi gii vn tt)
a) T gic BCPI ni tip (hs t cm).
b) D thy MI v AC l hai ng cao ca MAB P A l trc tm
ca MAB BP A l ng cao th ba
( ) 1 BP MA .
Mt khc
0
90 AKB = (gc ni tip chn na ng trn)
( ) 2 BK MA .
T (1) v (2) suy ra ba im B, P, Q thng hng.
c)
2 2 2 2
4 3 AC AB BC R R R = = =
Khi BC = R d thy tam gic OBC l tam gic u suy ra
0
60 CBA =
M QAC CBA = (gc to bi tia tip tuyn v gc ni tip cng chn AC ) do
0
60 QAC = .
D thy tam gic QAC cn ti Q (QA = QC) c
0
60 QAC = nn l tam gic u 3 AQ AC R = = .
D thy
3
;
2 2
R R
AI IB = =
Trong tam gic vung
( )
0
90 IBM I = ta c
0
3 3 3
. tan . tan 60 3
2 2
R R
IM IB B IB = = = = .
Ta chng minh c t gic QAIM l hnh thang vung
( )
0
/ / ; 90 AQ IM I = .
Do ( )
2
1 1 3 3 5 3 5 3
3 .
2 2 2 2 4 2 8
QAIM
R R R R R
S AQ IM AI R
| |
= + = + = =
|
|
\ .
(vdt).
Bi 5:
Cch 1: Ta c
2 2 1 1 1 1
1 1 2 2 2
xy xy xy
A A
xy xy A xy xy
+
= = = = +
+ +

V
1
0, 0 0 0 0 x y A A
A
> > < > >

do
min ax
1
min
m
A A
A

.
Mt khc ( )
2
2 2
1
0 2 2 1 1
2
x y x y xy xy
xy
> + > s > (v 2 0 xy > )
Do
1 1 3
1
2 2 A
> + =

. Du = xy ra khi x y = .
T
2 2
0, 0
2
2
1
x y
x y x y
x y

> >

= = =

+ =


Q
K
P
M
I
A
O
B
C

116
Lc
1
2
2
2
1
3
1
2
A

= =
+
. Vy
2
min
3
A = khi
2
2
x y = = .
Cch 2: Vi 0, 0 x y > > ta c
2 2
1 3 1 2 2 4
1
2 2 2 1 3 1 3
x y
xy xy xy
xy xy
+
> s + s > >
+ +

Do
2 2 4 2
2 2
1 1 3 3
xy
A
xy xy

= = + > + =
+ +
.
Du = xy ra khi x y = .
T
2 2
0, 0
2
2
1
x y
x y x y
x y

> >

= = =

+ =


Vy
2
min
3
A = khi
2
2
x y = = .
Cch 3:
Vi 0, 0 x y > > v
2 2
1 x y + =
Ta c
( )
( )
( )
( )
( )
2 2 2
2 4
2
2 2 2 2 2 6 2
0
3 3 1 3 1 3 1 3 1 3
x y xy
x y
xy xy xy
A A
xy xy xy xy
+

+
+ = + = = = > >
+ + + +

Du = xy ra khi
2
2
x y = = . Vy
2
min
3
A = khi
2
2
x y = = .

( )
( )
( )
2 2
2 2
2
0; 0 0 a 2 0 2 0
1
0
2 2
0
2
3 2
a a xy
A b a xy bxy a x y b a xy
b b xy
a
b a a
a x y xy
b a
a b
a

+ > > + > + > + >


+
>
| |
+ > =

|
=
\ .






















117

Bi 1 (2,0im)
1) Tm gi tr ca x cc biu thc c ngha:
3 2 x ;
4
2 1 x

2) Rt gn biu thc:
(2 3) 2 3
2 3
A
+
=
+

Bi 2 (2,0 im)
Cho phng trnh: mx
2
(4m -2)x + 3m 2 = 0 (1) ( m l tham s).
1) Gii phng trnh (1) khi m = 2.

2) Chng minh rng phng trnh (1) lun c nghim vi mi gi tr ca m.
3) Tm gi tr ca m phng trnh (1) c cc nghim l nghim nguyn.
Bi 3 (2,0 im)
Gii bi ton sau bng cch lp phng trnh hoc h phng trnh:
Mt mnh vn hnh ch nht c chu vi 34m. Nu tng thm chiu di 3m v chiu rng 2m th din
tch tng thm 45m
2
. Hy tnh chiu di, chiu rng ca mnh vn.
Bi 4 (3,0 im)
Cho ng trn O. T A l mt im nm ngoi (O) k cc tip tuyn AM v AN vi (O) ( M; N l
cc tip im ).
1) Chng minh rng t gic AMON ni tip ng trn ng knh AO.
2) ng thng qua A ct ng trn (O) ti B v C (B nm gia A v C ). Gi I l trung im ca
BC. Chng minh I cng thuc ng trn ng knh AO.
3) Gi K l giao im ca MN v BC . Chng minh rng AK.AI = AB.AC.
Bi 5 (1,0 im)
Cho cc s x,y tha mn x >0; y >0 v x + y = 1.
Tm gi tr ln nht v nh nht ca A = x
2
+ y
2
.
--------------------- Ht --------------------
Cu 1:
a) 3 2 x c ngha 3x 2
2
0 3 2
3
x x > > >
4
2 1 x
c ngha
1
2 1 0 2 1
2
x x x > > >

b)
2 2 2
2 2
(2 3) (2 3) (2 3) 2 3 (2 3)(2 3) 2 3
1
1
2 3 (2 3)(2 3)
2 3
A
+ + +
= = = = =
+ +


Cu 2:
2
(4 2) 3 2 0 (1) mx m x m + =

1.Thay m = 2 vo pt ta c:
2 2
(1) 2 6 4 0 3 2 0 x x x x + = + =
Ta thy: 1 3 +2 = 0 nn pt c 2 nghim:
1 2
0; 2 x x = =
UBND tnh bc ninh
S gio dc v o to


thi tuyn sinh vo lp 10 thpt
Nm hc 2012 - 2013
Mn thi: Ton (Dnh cho tt c th sinh)
Thi gian: 120 pht (Khng k thi gian giao )
Ngy thi: 30 thng 06 nm 2012

chnh thc

118
2. * Nu m = 0 th (1) 2 2 0 1 x x = = .
Suy ra: Pt lun c nghim vi m=0
*Nu m = 0 th ph (1) l pt bc 2 n x.
Ta c:
2 2 2 2
' (2 1) (3 2) 4 4 1 3 2 ( 1) 0 0 m m m m m m m m m A = = + + = > =
Kt lun: Kt hp 2 trng hp ta c: pt lun c nghim vi mi m (pcm)
3. * Nu m = 0 th (1) 2 2 0 1 x x = = nguyn
Suy ra: Vi m = 0 pt c nghim nguyn
* Nu m # 0 th ph (1) l pt bc 2 n x. T 2 ta c: pt c 2 nghim:
1
2
2 1 1
1
2 1 1 3 2
m m
x
m
m m m
x
m m
+
= =

= =


pt (1) c nghim nguyn th nghim
2
x phi nguyn
3 2 2
3 ( 0) 2
m
Z Z m m
m m

e e = hay m l
c ca 2 m = {-2; -1; 1; 2}
Kt lun: Vi m = { 1; 2; 0 } th pt c nghim nguyn
Cu 3:

Gi chiu di hcn l x (m); chiu rng l y (m) (0 < x, y < 17)
Theo bi ra ta c hpt :
34: 2 17 12
( 3)( 2) 45 5
x y x
x y xy y
+ = = =


+ + = + =

(tha mn k)
Vy : chiu di = 12m, chiu rng = 5m
Cu 4 :
1. Theo tnh cht tip tuyn vung gc vi bn knh
ti tip im ta c : 90
O
AMO ANO = =
AMO vung ti M A, M , O thuc ng trn
ng knh AO ( V AO l cnh huyn)
ANO vung ti N A, N, O thuc ng trn
ng knh AO (V AO l cnh huyn)
Vy: A, M, N, O cng thuc ng trn ng knh AO
Hay t gic AMNO ni tip ng trn ng knh AO
2. V I l trung im ca BC (theo gt) OI BC (tc)
AIO vung ti I A, I, O thuc ng trn
ng knh AO (V AO l cnh huyn)
Vy I cng thuc ng trn ng knh AO (pcm)
3. Ni M vi B, C.
Xt & AMB AMC c MAC chung

1
2
MCB AMB = = s MB
~ AMB ACM (g.g)
2
.
AB AM
AB AC AM
AM AC
= = (1)
Xt & AKM AIM c MAK chung
AIM AMK = (V: AIM ANM = cng chn AM
v AMK ANM = )
~ AMK AIM (g.g)
2
.
AK AM
AK AI AM
AM AI
= = (2)
T (1) v (2) ta c: AK.AI = AB.AC (pcm)
Cu 5:
E K
I
B
M
N
O
A
C

119
* Tm Min A
Cch 1:
Ta c:
( )
( )
2
2 2
2
2 2
2 1
2 0
x y x xy y
x y x xy y
+ = + + =
= + >

Cng v vi v ta c:
( ) ( )
2 2 2 2
1 1
2 1
2 2
x y x y A + > + > >
Vy Min A =
1
2
. Du = xy ra khi x = y =
1
2

Cch 2
T 1 1 x y x y + = = Thay vo A ta c :
( )
2
2 2 2
1 1 1
1 2 2 1 2( )
2 2 2
A y y y y y y = + = + = + >
Du = xy ra khi : x = y =
1
2

Vy Min A =
1
2
Du = xy ra khi x = y =
1
2

* Tm Max A
T gi thit suy ra
2
2 2
2
0 1
1
0 1
x x x
x y x y
y
y y
s s s
+ s + =

s s
s



Vy : Max A = 1 khi x = 0, y

GII CU 05
THI VO LP 10 MN TON BC NINH
2012-2013
=====================================

CU 05 :
Cho cc s x ; y tho mn x 0 ; 0 > > y v x+ y = 1
.Tm gi tr ln nht v gi tr nh nht ca biu thc A = x
2
+ y
2

I- TM GI TR NH NHT
CCH 01 :
a) Tm gi tr nh nht ca biu thc A .
Ta c x + y = 1 nn y = - x + 1 thay vo A = x
2
+ y
2
ta c :
x
2
+ ( -x + 1)
2
- A = 0 hay 2x
2
- 2x + ( 1- A) = 0 (*)
do biu thc A tn ti gi tr nh nht v gi tr ln nht khi v ch khi phng trnh (*) c nghim hay
( )
2
1
0 1 2 0 1 2 1 0 ' > > > > A A A A .Vy gi tr nh nht ca biu thc A l
2
1
khi phng
trnh (*) c nghim kp hay x =
2
1
m x + y = 1 th y =
2
1
. Vy Min A = 1/2 khi x = y = 1/2 ( t/m)
b) Tm gi tr ln nht ca biu thc A .
CCH 02 :
a) Tm gi tr nh nht ca biu thc A .
Theo Bt ng thc Bunhia ta c 1 = x + y hay
1= (x + y)
2
( )
2
1
2
2 2 2 2
> + + s y x y x . Vy gi tr nh nht ca biu thc A l 1/2 khi x = y m x + y =1
hay x =y = 1/2 ( t/m)
b) Tm gi tr ln nht ca biu thc A .

120
CCH 03 :
a) Tm gi tr nh nht ca biu thc A .
Khng mt tnh tng qut ta t

=
=
m y
m x 1
vi 1 0 s s m

M A= x
2
+ y
2
. Do A = ( 1- m)
2
+ m
2
hay A= 2m
2
- 2m +1
hay 2A = (4m
2
- 4m + 1) + 1 hay 2A = (2m- 1)
2
+ 1 hay
( )
2
1
2
1
2
1 2
2
> +

=
m
A .
Vy gi tr nh nht ca biu thc A l 1/2 khi m= 1/2 hay x = y = 1/2.
b) Tm gi tr ln nht ca biu thc A.
CCH 04 :

a) Tm gi tr nh nht ca biu thc A .
Ta c A = x
2
+ y
2
= ( x+ y)
2
- 2xy = 1 -2xy ( v x + y =1 )
m xy
( )
2
1
2
1
2 1
2
1
2
4
1
4
2
> >

> s
+
s A xy xy xy
y x
.
Vy gi tr nh nht ca biu thc A l 1/2 khi x = y = 1/2.
b) Tm gi tr ln nht ca biu thc A.
CCH 05 :
a)Tm gi tr nh nht ca biu thc A .

Xt bi ton ph sau : Vi a , b bt k v c ; d > 0 ta lun c :
( )
d c
b a
d
b
c
a
+
+
> +
2 2 2
(*) , du = xy ra khi
d
b
c
a
=
Tht vy : c ( ) ( ) + >
(
(

|
|
.
|

\
|
+
|
|
.
|

\
|
+
2
2
2
2 2
b a
y
b
x
a
y x
( )
y x
b a
y
b
x
a
+
+
> +
2 2 2
(PCM)
.P DNG
Cho a = x v b = y ,t (*) c : A= x
2
+ y
2
=
( )
2 1 1
2 2 2
y x y x +
> + m x+ y =1
Nn A
2
1
> .Vy gi tr nh nht ca biu thc A l 1/2 khi x = y = 1/2.
b) Tm gi tr ln nht ca biu thc A .
CCH 06 :
a)Tm gi tr nh nht ca biu thc A .
Ta c A = x
2
+ y
2
hay xy =
2
1 A
(*) m x + y =1 (**)
Vy t (*) ;(**) c h phng trnh

=
= +
2
1
1
A
xy
y x
,h ny c nghim
( )
2
1
0 1 2 1 0 ; 0 > > > > A A y x . Vy gi tr nh nht ca biu thc A l 1/2 khi x+ y =1 v x
2
+
y
2
=
2
1
hay x = y = 1/2.
b) Tm gi tr ln nht ca biu thc A .
CCH 07 :
a)Tm gi tr nh nht ca biu thc A .

121

Ta c A = x
2
+ y
2
= x
2
+ y
2
+ 1 - 1 m x + y =1 nn A = x
2
+ y
2
- x - y -1
Hay A =
2
1
2
1
4
1
4
1
2 2
> + |
.
|

\
|
+ + |
.
|

\
|
+ y y x x . Vy gi tr nh nht ca biu thc A l 1/2 khi x = y = 1/2.
b) Tm gi tr ln nht ca biu thc A .
CCH 08 :
a)Tm gi tr nh nht ca biu thc A .
Ta c A= x
2
+ y
2
=
( )
( ) 2 2 1
2 2 2 2 2 2 2
y x
y x
y x
y x
y
y x
x
y x
y x y x +
=
+
+
>
+
+
+
=
+
+
=
+

M x + y =1 nn A
2
1
> . Vy gi tr nh nht ca biu thc A l 1/2. khi x = y = 1/2.
b)Tm gi tr ln nht ca biu thc A .
CCH 09 :
a)Tm gi tr nh nht ca biu thc A .
Ta c x + y = 1 l mt ng thng , cn x
2
+ y
2
= A l mt ng trn c tm l gc to O bn kn
A m x > > 0 ; 0 y thuc gc phn t th nht ca ng trn trn . Do tn ti cc tr th
khong cch t O n ng thng x + y =1 phi nh hn hay bng bn kn ng trn hay A
2
1
> . Vy
gi tr nh nht ca biu thc A l 1/2 khi x =y = 1/2.

b)Tm gi tr ln nht ca biu thc A .

CCH 10 :
a)Tm gi tr nh nht ca biu thc A .
Ta c x + y =1
2
1
2
1
= + y x . Vy chng minh A
2
1
>
vi A = x
2
+ y
2
th ta ch cn chng minh
2
1
2 2
+ > + y x y x .
Tht vy :
Ta c
2
1
2 2
+ > + y x y x 0
Hay 0
2
1
2
1
2 2
>
|
.
|

\
|
+
|
.
|

\
|
y x ( lun ng ) Vy A
2
1
> . Vy gi tr nh nht ca biu thc A l 1/2 khi
x = y =1/2.
b)Tm gi tr ln nht ca biu thc A .
CCH 11 :
a)Tm gi tr nh nht ca biu thc A .
Khng mt tnh tng qut ta t 2 1
1
2
s s

=
=
m
m y
m x

.Do A = x
2
+ y
2
hay (2-m)
2
+ (m-1)
2
- A =0 hay 2m
2
- 6m +5 = A
Hay
( )
2
1
2
1
2
3 2
2
> +

=
m
A .
Vy gi tr nh nht ca A l 1/2 khi x = y = 1/2.
b)Tm gi tr ln nht ca biu thc A .
CCH 12 :
a)Tm gi tr nh nht ca biu thc A .

122
Khng mt tnh tng qut ta t 3 2
2
3
s s

=
=
m
m y
m x

.Do A = x
2
+ y
2
hay (3-m)
2
+ (m-2)
2
- A =0 hay 2m
2
- 10m +13 = A
Hay
( )
2
1
2
1
2
5 2
2
> +

=
m
A .
Vy gi tr nh nht ca A l 1/2 khi x = y = 1/2.
b)Tm gi tr ln nht ca biu thc A .
CCH 13 :
a)Tm gi tr nh nht ca biu thc A .
Ta c x + y =1 hay (x+1) + (y +1) = 3 m A = x
2
+ y
2
hay
A = (x
2
+ 2x + 1) + ( y
2
+ 2y +1) - 4 hay A = (x+1)
2
+ ( y+1)
2
- 4
,do ta t

>
>

+ =
+ =
1
1
1
1
b
a
y b
x a
. Khi ta c bi ton mi sau :
Cho hai s a , b tho mn 1 ; 1 > > b a v a + b =3 . Tm gi tr nh nht ca biu thc A = a
2
+ b
2
- 4
Tht vy : Ta c A = a
2
+ b
2
- 4 = (a+b)
2
- 2ab - 4 = 5 - 2ab ( v a+b=3)
Mt khc theo csi c :
( )
4
9
4
2
=
+
s
b a
ab do A
2
1
> . Vy gi tr nh nht ca biu thc A l 1/2 khi x = y
= 1/2.
b)Tm gi tr ln nht ca biu thc A .
CCH 14 :
a)Tm gi tr nh nht ca biu thc A .
Khng mt tnh tng qut ta t a m b
b m y
m a x
s s

=
=

( vi a > b v a - b =1 hay a = b+ 1 hay a > b )
.Do A = x
2
+ y
2
hay (a-m)
2
+ (m-b)
2
- A =0 hay
2m
2
- 2m (a+b) +(a
2
+ b
2
) = A hay
Hay ( ) | | ( ) ( )
( ) | |
2
1
2
1
2
2
2 2 2
2
2 2 2 2
> +
+
= + + + + =
b a m
A b a b a b a m A
(V a - b= 1)
Vy gi tr nh nht ca A l 1/2 khi x = y = 1/2.

b)Tm gi tr ln nht ca biu thc A .
CCH 15 :
a)Tm gi tr nh nht ca biu thc A .
Ta c x + y =1 hay y = 1 - x m y 1 0 0 s s > x
Do x
2
+ y
2
- A = 0 hay 2 x
2
- 2x +( 1 - A ) = 0 .
Khi ta c bi ton mi sau :
Tm A phng trnh 2 x
2
- 2x +( 1 - A ) = 0 (*) c nghim 1 0
2 1
s s s x x
Vi x
1
; x
2
l nghim ca phng trnh (*)
Tht vy phng trnh (*) c nghim

123
1
2
1
1
2
0 '
0
0
0 '
1
2
0
0
1
1
0
0
1
0
1 0
2
1
2
1
2 1
1 2
2 1
s s

s
s
> A

>
>
> A

s
s

>
>

s
s

>
>

s s
> >
s s s A
P
S
P
S
P
S
P
S
x
x
x
x
x x
x x
x x

Vy gi tr nh nht ca biu thc A l 1/2 khi x =y = 1/2.
b)Tm gi tr ln nht ca biu thc A .
Vy theo trn ta c gi tr ln nht ca biu thc A l 1
khi x = 0 v y = 1 hoc x= 1 v y = 0 .
II- TM GI TR LN NHT
CCH 01 :
Vy theo trn ta c gi tr ln nht ca biu thc A l 1
khi x = 0 v y = 1 hoc x= 1 v y = 0
CCH 02 :
Ta c A = x
2
+ y
2
hay xy =
2
1 A
(*) v x + y =1 m x 0 0 ; 0 > > > xy y
Do theo (*) c A 1 s . Vy gi tr ln nht ca biu thc A l 1
khi x = 0 v y = 1 hoc x= 1 v y = 0
CCH 03 :
Khng mt tnh tng qut ta t

> =
> =
0 cos
0 sin
2
2
o
o
y
x

Do A = ( ) 1 cos . sin 2 1 cos sin
2 4 4
s = + o o o o .
Vy gi tr ln nht ca biu thc A l 1
khi x = 0 v y = 1 hoc x= 1 v y = 0





















124
S GD & T H TNH


( thi c 1 trang)
M 01
K THI TUYN SINH VO LP 10 THPT
NM HC 2012 2013
Mn thi: TON
Ngy thi : 28/6/2012
Thi gian lm bi : 120 pht
Cu 1 (2im)
a) Trc cn thc mu ca biu thc:
5
.
6 1

b) Gii h phng trnh:
2 7
.
2 1
=

+ =

x y
x y

Cu 2 (2im)
Cho biu thc:
2
4 1
.
1
| |

=
|
|

\ .
a a a
P
a a a a
vi a >0 v 1 a = .
a) Rt gn biu thc P.
b) Vi nhng gi tr no ca a th P = 3.
Cu 3 (2im)
a) Trong mt phng ta Oxy, ng thng y = ax + b i qua im M(1 ; 2) v song song vi ng
thng y = 2x + 1. Tm a v b.
b) Gi x
1
, x
2
l hai nghim ca phng trnh x
2
+ 4x m
2
5m = 0. Tm cc gi tr ca m sao cho: |x
1

x
2
| = 4.
Cu 4 (3im)
Cho tam gic ABC c ba gc nhn, ni tip ng trn tm O. Hai ng cao AD, BE ct nhau ti H
(DeBC, E eAC) .
a) Chng minh t gic ABDE ni tip ng trn.
b) Tia AO ct ng trn (O) ti K ( K khc A). Chng minh t gic BHCK l hnh bnh hnh.
c) Gi F l giao im ca tia CH vi AB. Tm gi tr nh nht ca biu thc:
AD BE CF
Q .
HD HE HF
= + +
Cu 5 (1im)
Tm tt c cc gi tr ca tham s m phng trnh sau v nghim:
x
2
4x 2m|x 2| m + 6 = 0.

HNG DN CHM THI
Cu N dung m
1
a) Ta c:
5 5( 6 1)
6 1 ( 6 1)( 6 1)
+
=
+
0,5

5( 6 1) 5( 6 1)
6 1
6 1 5
+ +
= = = +

0,5
b) Ta c:
2x y 7 4x 2y 14
x 2y 1 x 2y 1
= =


+ = + =

0,5

5x 15 x 3
x 2y 1 y 1
= =


+ = =

0,5
CHNH THC

125
2
a) Vi 0 1 a < = th ta c:
2 2
4 1 4 1 1
. .
1 1
| |

= =
|
|

\ .
a a a a a
P
a a
a a a a
0,5
2
4a 1
a

=

0,5
b) Vi 0 1 a < = th P = 3
2
2
4a 1
3 3a 4a 1
a

= =
2
3a 4a 1 0 + = 0,5
a = 1 (loi) hoc
1
a
3
= (tha mn k). 0,5
3
a) ng thng y = ax + b song song vi ng thng y = 2x +1 nn:
a = 2, b = 1.
0,5
V ng thng y = 2x + b i qua im M(1 ; 2) nn ta c pt:
2(-1) + b = 2 b = 4 (tha mn b = 1). Vy a = 2, b = 4
0,5
b) Ta c :
2
' 4 m 5m (m 1)(m 4) A = + + = + + . phng trnh c 2 nghim x
1
, x
2
th ta
c: ' 0 A > m 4 s hoc m 1 > (*)
0,25
Theo nh l Vi-et, ta c:
1 2
b
x x 4
a
+ = =
v
2
1 2
c
x .x m 5m.
a
= =
0,25
Ta c:
2 2
1 2 1 2 1 2 1 2
x x 4 (x x ) 16 (x x ) 4x .x 16 = = + =
2 2
16 4( m 5m) 16 m 5m 0 = + = m = 0 hoc m = 5
0,25
Kt hp vi k(*), ta c m = 0 , m = 5 l cc gi tr cn tm.
0,25
4
a) V AD v BE l cc ng cao nn ta c:
ADB AEB 90 = =
0,5
Hai gc ADB, AEB cng nhn cnh AB di
mt gc 90 nn t gic ABDE ni tip ng
trn.
0,5
b) Ta c: ABK ACK 90 = = (gc ni tip chn na
ng trn) CK AC, BK AB (1)
Ta c H l trc tm ca tam gic ABC nn:
BH AC, CH AB (2)
0,5
T (1) v (2), suy ra: BH // CK, CH // BK.
Vy t gic BHCK l hnh bnh hnh (theo nh
ngha)
0,5
t S
BHC
= S
1
, S
AHC
= S
2
, S
AHB
= S
3
, S
ABC
= S. V ABC A nhn nn trc tm H nm bn
trong ABC A , do : S = S
1
+ S
2
+ S
3
.
0,25
Ta c:
ABC ABC ABC
BHC 1 AHC 2 AHB 3
S S S AD S BE S CF S
(1), (2), (3)
HD S S HE S S HF S S
= = = = = =
0,25
Cng v theo v (1), (2), (3), ta c:
1 2 3 1 2 3
AD BE CF S S S 1 1 1
Q S
HD HE HF S S S S S S
| |
= + + = + + = + +
|
\ .

p dng bt ng thc Csi cho 3 s dng, ta c:
0,25
H
F
E
D
K
O
C
B
A

126
3
1 2 3 1 2 3
S S S S 3 S .S .S = + + > (4) ;
3
1 2 3 1 2 3
1 1 1 3
S S S S .S .S
+ + > (5)
Nhn v theo v (4) v (5), ta c: Q 9 > . ng thc xy ra
1 2 3
S S S = = hay H l
trng tm ca ABC A , ngha l ABC A u.
0,25
5
Ta c: x
2
4x 2m|x 2| m + 6 = 0 (*). t x 2 t 0 = > th pt (*) tr thnh: t
2
2mt
+ 2 m = 0 (**),
2
'(t) m m 2 (m 1)(m 2) A = + = +
0,25
pt (*) v nghim th pt(**) phi v nghim hoc c 2 nghim t
1
, t
2
sao cho:
1 2
t t 0 s <
0,25
Pt (**) v nghim '(t) 0 (m 1)(m 2) 0 2 m 1 A < + < < < (1)
Pt (**) c 2 nghim t
1
, t
2
sao cho:
1 2
t t 0 s < . iu kin l:
' 0 ' 0
2m 0 m 0 m 2
2 m 0 m 2
A > A >

< < s


> <

(2)
0,25
Kt hp (1) v (2), ta c k cn tm ca m l: m <1.
0,25
Ch : Mi cch gii ng u cho im ti a, im ton bi khng quy trn.
































127
S GIO DC V O TO K THI TUYN SINH LP THPT
NH DNG Nm c 2012 2013

Mn thi: Ton
T gan lm b: 120 pt
Kng k t gan pt

1 1 m: Cho biu thc: A =
2 3
50 8
5 4
x x
1/ Rt gn biu thc A
2/ Tnh gi tr ca x khi A = 1
2 15 m:
1/ V th (P) hm s y =
2
2
x

2/ Xc nh m ng thng (d): y = x m ct (P) ti im A c honh bng 1. Tm tung ca im A
3 2 m:
1/ Gii h phng trnh:
2 4
3 3
x y
x y
=


2/ Gii phng trnh: x
4
+ x
2
6 = 0
Bi 4 (2 m: Cho phng trnh x
2
2mx 2m 5 = 0 (m l tham s)
1/ Chng minh rng phng trnh lun c hai nghim phn bit vi mi gi tr ca m
2/ Tm m
1 2
x x t gi tr nh nht (x
1
; x
2
l hai nghim ca phng trnh)
Bi 5 35 m: Cho ng trn (O) v im M ngoi ng trn. Qua M k cc tip tuyn MA, MB v
ct tuyn MPQ (MP < MQ). Gi I l trung im ca dy PQ, E l giao im th 2 gia ng thng BI v
ng trn (O). Chng minh:
1/ T gic BOIM ni tip. Xc nh tm ca ng trn ngoi tip t gic
2/ BOM = BEA
3/ AE // PQ
4/ Ba im O; I; K thng hng, vi K l trung im ca EA
HNG DN GII:

N dung
1 1 m:
1/ KX: x > 0
A =
2 3
50 8
5 4
x x
=
2 3
25.2 4.2
5 4
x x
chnh thc

128
=
3
2 2 2
2
x x
=
1
2
2
x
Vy vi x > 0 thi A =
1
2
2
x
2/ Khi A = 1
1
2
2
x = 1
2x = 2
2x = 4
x = 2 (Tha iu kin xc nh)
Vy khi A = 1 gi tr ca x = 2
2 15 m:
1/ V th (P) hm s y =
2
2
x

-Bng gi tr
x -4 -2 0 2 4
y =
2
2
x
8 2 0 2 8
- th (P) l ng parabol nh O(0; 0) nm pha trn trc honh, nhn trc tung lm trc i xng v i
qua cc im c ta cho trong bng trn.

2/ Cch 1.
V (d) ct (P) ti im A c honh bng 1 nn x = 1 tha mn cng thc hm s (P) => Tung ca im
A l: y
A
=
2
1
2
=
1
2

A(1;
1
2
)
e
(d) nn
1
2
= 1 m
m = 1
1
2
=
1
2

Vy vi m =
1
2
th (d): y = x m ct P ti im A c honh bng 1. Khi tung y
A
=
1
2

Cch 2
Ta c phng trnh honh giao im ca (d) v (P) l:
2
2
x
= x m x
2
2x + 2m = 0 (*)
(d) ct (P) ti im A c honh bng 1 th phng trnh (*) c nghim bng 1

129
1
2
2.1 + 2m = 0 m =
1
2

Vy vi m =
1
2
th (d): y = x m ct P ti im A c honh bng 1. Khi tung y
A
=
2
1
2
=
1
2


3 2 m:
1/ Gii h phng trnh
2 4
3 3
x y
x y
=


1
3 3
x
x y
=


1
3.( 1) 3
x
y
=


1
6
x
y
=


Vy h phng trnh c nghim duy nht (-1; -6)
2/ Gii phng trnh
x
4
+ x
2
6 = 0 (1)
t x
2
= t (t > 0)
Phng trnh (1) tr thnh: t
2
+ t 6 = 0 (2)
Ta c A = 1
2
4.1.(-6) = 25
Phng trnh (2) c hai nghim t
1
=
1 25
2.1
+
= 2 (nhn) ; t
2
=
1 25
2.1

= -3 (loi)
Vi t = t
1
= 2 => x
2
= 2 x = 2
Vy phng trnh cho c hai nghim x
1
= 2 ; x
2
= - 2
4 2 m: Cho phng trnh x
2
2mx 2m 5 = 0 (m l tham s)
1/ Ta c A = (-m)
2
1 (-2m 5)
= m
2
+ 2m + 5
= (m + 1)
2
+ 4
V (m + 1)
2
> 0 vi mi m
(m + 1)
2
+ 4 > 0 vi mi m
Hay A > 0 vi mi m
Vy phng trnh cho lun c hai nghim phn bit vi mi m
2/ V phng trnh cho lun c hai nghim phn bit vi mi m

1 2
1 2
2
. 2 5
x x m
x x m
+ =

(theo nh l Vi-et)
t A =
1 2
x x
A
2
= (
1 2
x x )
2
= x
1
2
2x
1
x
2
+ x
2
2
= (x
1
+ x
2
)
2
4x
1
x
2

A
2
= (2m)
2
4(-2m 5) = (2m)
2
+ 8m + 20
= (2m)
2
+ 2. 2m. 2 + 4 + 16 = (2m + 2)
2
+ 16 > 16
Gi tr nh nht ca A
2
= 16
Gi tr nh nht ca A l 4 khi 2m + 2 = 0 m = -1
Vy vi m = -1 th
1 2
x x t gi tr nh nht l 4
5 35 m:
1/ Ta c MB l tip tuyn ca (O) (gt)
OB MB
OBM = 90
0

B thuc ng trn ng knh OM (1)
Ta c IQ = IP (gt)
OI QP (Tnh cht lin h gia ng knh v dy cung)
OIM = 90
0

P
O
M
A
B
Q
I
E K

130
I thuc ng trn ng knh OM (2)
T (1) v (2) => BOIM ni tip ng trn ng knh OM
2/ Ta c BOM = AOM (Tnh cht hai tip tuyn ct nhau)
BOM =
1
2
BOA
m BOA = SAB
BOM =
1
2
SAB
Ta li c BEA =
1
2
SAB (nh l gc ni tip)
BOM = BEA
3/ Ta c: T gic BOIM ni tip (Chng minh trn)
BOM = BIM (Cng chn BM)
m BOM = BEA (Chng minh trn)
BIM = BEA
Mt khc BIM v BEA l hai gc v tr ng v
AE // PQ
4/ Ta c OI QP v AE // PQ (chng minh trn);
OI AE (3)
m KE = KA (gt)
OK AE (tnh cht lin h gia ng knh v dy cung) (4)
T (3) v (4), ta thy qua im O c hai ng thng OI v OK cng song song vi AE
OI v OK phi trng nhau
Ba im O, I, K thng hng


























131
S GIO DC V O TO
THI NH
K THI TUYN SINH LP 10 THPT
NM HC 2012 2013
Mn t: TON
Thi gian lm bi: 120 pht, khng k thi gian giao
Bi 1. (2,0 im)
1) Tnh:
1
A 9 4 5.
5 2
= +
+

2) Cho biu thc:
2(x 4) x 8
B
x 3 x 4 x 1 x 4
+
= +
+
vi x 0, x 16.
a. Rt gn B.
b. Tm x gi tr ca B l mt s nguyn.
Bi 2. (2,0 im)
Cho phng trnh: x
2
4x + m + 1 = 0 (m l tham s).
1) Gii phng trnh vi m = 2.
2) Tm m phng trnh c hai nghim tri du (x
1
< 0 < x
2
). Khi nghim no c gi
tr tuyt i ln hn?
Bi 3. (2,0 im):
Trong mt phng to Oxy cho parabol (P): y = -x
2
v ng thng (d): y = mx + 2 (m
l tham s).
1) Tm m (d) ct (P) ti mt im duy nht.
2) Cho hai im A(-2; m) v B(1; n). Tm m, n A thuc (P) v B thuc (d).
3) Gi H l chn ng vung gc k t O n (d). Tm m di on OH ln nht.
Bi 4. (3,5 im)
Cho ng trn (O), dy cung BC (BC khng l ng knh). im A di ng trn
cung nh BC (A khc B v C; di on AB khc AC). K ng knh AA ca ng
trn (O), D l chn ng vung gc k t A n BC. Hai im E, F ln lt l chn
ng vung gc k t B, C n AA. Chng minh rng:
1) Bn im A, B, D, E cng nm trn mt ng trn.
2) BD.AC = AD.AC.
3) DE vung gc vi AC.
4) Tm ng trn ngoi tip tam gic DEF l mt im c nh.
Bi 5.(0,5 im):
Gii h phng trnh:

4 3 2
2 2 2 2
x x 3x 4y 1 0
.
x 4y x 2xy 4y
x 2y
2 3
+ =

+ + +
+ = +


P N

N dung m
1.
(0,5)
2
5 2
A ( 5 2) 5 2 5 2 4.
5 4

= + = =

0,5
2. a. (1 )
chnh thc

132
(1,5)
Vi x 0, x 16, th:
B
2(x 4) x 8 2x 8 x( x 4) 8( x 1)
( x 1)( x 4) x 1 x 4 ( x 1)( x 4)
+ + + +
= + =
+ + +

0,25

2x 8 x 4 x 8 x 8 3x 12 x
( x 1)( x 4) ( x 1)( x 4)
+ +
= =
+ +
0,25

3 x( x 4) 3 x
( x 1)( x 4) x 1

= =
+ +
0,25
Vy
3 x
B
x 1
=
+
vi x 0, x 16. 0,25
b. (0,5 )
D thy B 0 (v x 0) > .
Li c:
3
B 3 3
x 1
= <
+
(v
3
0 x 0, x 16)
x 1
> > =
+
.
Suy ra: 0 B < 3 B e {0; 1; 2} (v B e Z).
0,25
- Vi B = 0 x = 0;
- Vi B = 1
3 x 1
1 3 x x 1 x .
4
x 1
= = + =
+

- Vi B = 2
3 x
2 3 x 2( x 1) x 4.
x 1
= = + =
+

Vy B e th x e {0;
1
;
4
4}.
0,25

Bi 2.

N dung m
1.
(1,0)
m = 2, phng trnh cho thnh: x
2
4x + 3 = 0.
Phng trnh ny c a + b + c = 1 4 + 3 = 0 nn c hai nghim: x
1
= 1; x
2
= 3.
0,5
Vy vi m = 2 th phng trnh cho c hai nghim phn bit: x
1
= 1; x
2
= 3. 0,5
2.
(1,0)
Phng trnh cho c hai nghim tri du ac < 0 m + 1 < 0 m < -1. 0,5
Theo nh l Vi-et, ta c:
1 2
1 2
x x 4
x x m 1
+ =

= +

.
Xt hiu: |x
1
| - |x
2
| = -x
1
x
2
= -4 < 0 (v x
1
< 0 < x
2
) |x
1
| < |x
2
|.
0,25
Vy nghim x
1
c gi tr tuyt i nh hn nghim x
2
. 0,25

Bi 3. (2,0 im):

N dung m
1.
(0,75)
(d) ct (P) ti mt im duy nht Phng trnh honh ca (d) v (P):
-x
2
= mx + 2 x
2
+ mx + 2 = 0 c nghim duy nht.
0,25

133
A = m
2
8 = 0 m = 2 2.
0,25
Vy gi tr m cn tm l m = 2 2. 0,25
2.
(0,75)
2
A (P) m 4 m ( 2)
n 2 B (d) n m 2
e = =


= e = +


0,5
Vy m = -4, n = -2. 0,25






3.
(0,5)
- Nu m = 0 th (d) thnh: y = 2 khong cch t O n (d) = 2 OH = 2
(Hnh 1).
y = 2
x
y
Hnh 1
3 2
-2
-2
3
2
-1
-1
1
O
1
H

x
y
(d)
Hnh 2
H
B
-3
-2
2
-1
-1
1
O
1
A







0,25
- Nu m 0 th (d) ct trc tung ti im A(0; 2) v ct trc honh ti im
B(
2
;
m
0) (Hnh 2).
OA = 2 v OB =
2 2
m | m|
= .
AOAB vung ti O c OH AB
2 2
2 2 2
1 1 1 1 m m 1
OH OA OB 4 4 4
+
= + = + =
2
2
OH
m 1
=
+
. V m
2
+ 1 > 1 m 0
2
m 1 1 + > OH < 2.
So snh hai trng hp, ta c OH
max

= 2 m = 0.

0,25

Bi 4. (3,5 im)

N dung m
1.
(0,5)
V
0
ADB AEB 90 = = bn im A, B, D, E cng thuc ng trn ng
knh AB.
0,5
2.
(1,0)
Xt AADB v AACA c:
0
ADB ACB 90 = = (
0
ACB 90 = v l gc ni tip chn na ng trn);
ABD AA' C = (hai gc ni tip cng chn cung AC)
AADB ~ AACA (g.g)
0,5

134

AD BD
AC A' C
= BD.AC = AD.AC (pcm).










0,5
3.
(1,25
Gi H l giao im ca DE vi AC.
T gic AEDB ni tip HDC BAE BAA'. = =
0,25
BAA' v BCA l hai gc ni tip ca (O) nn:
1 1
BAA' sBA' ; BCA sBA.
2 2
= =
0,25

0
1 1 1
BAA' BCA sBA' sBA sABA' 90
2 2 2
+ = + = = (do AA l ng knh)
0,25
Suy ra:
0
HDC HCD BAA' BCA 90 + = + = ACHD vung ti H.
0,25
Do : DE AC.
4.
(0,5
Gi I l trung im ca BC, K l giao im ca OI vi DA, M l giao im ca
EI vi CF, N l im i xng vi D qua I.
Ta c: OI BC OI // AD (v cng BC) OK // AD.
AADA c: OA = OA (gt), OK // AD KD = KA.
ADNA c ID = IN, KD = KA IK // NA; m IK BC (do OI BC)
NA BC.
T gic BENA c
0
BEA' BNA' 90 = = nn ni tip c ng trn
EA' B ENB = .
Ta li c: EA' B AA' B ACB = = (hai gc ni tip cng chn cung AB ca (O)).
ENB ACB = NE // AC (v c hai gc v tr ng v bng nhau).
M DE AC, nn DE EN (1)
Xt AIBE v AICM c:
EIB CIM = (i nh)
IB = IC (cch dng)
IBE ICM = (so le trong, BE // CF (v cng AA))

0,25
K
N
M
H
I
D
E
F
A'
O
B
C
A

135
AIBE = AICM (g.c.g) IE = IM
AEFM vung ti F, IE = IM = IF.
T gic DENM c IE = IM, ID = IN nn l hnh bnh hnh (2)
T (1) v (3) suy ra DENM l hnh ch nht IE = ID = IN = IM
ID = IE = IF. Suy ra I l tm ng trn ngoi tip ADEF.
I l trung im ca BC nn I c nh.
Vy tm ng trn ngoi tip tam gic DEF l mt im c nh.
0,25

Bi 5.(0,5 im):

N dung m

T (2) suy ra x + 2y 0.
p dng bt ng thc Bunhiacopxki, ta c:
2 2 2 2 2 2 2
2(x 4y ) (1 1 )[x (2y) ] (x 2y) + = + + > +
2 2 2
x 4y (x 2y) x 2y
2 4 2
+ + +
> = (3)
Du bng xy ra x = 2y.
Mt khc, d dng chng minh c:
2 2
x 2xy 4y x 2y
3 2
+ + +
> (4)

Tht vy,
2 2 2 2 2
x 2xy 4y x 2y x 2xy 4y (x 2y)
3 2 3 4
+ + + + + +
> > (do c hai v
u 0)
4(x
2
+ 2xy + 4y
2
) 3(x
2
+ 4xy + 4y
2
) (x 2y)
2
0 (lun ng x, y).
Du bng xy ra x = 2y.

0,25
T (3) v (4) suy ra:
2 2 2 2
x 4y x 2xy 4y
x 2y
2 3
+ + +
+ > + .
Du bng xy ra x = 2y.
Do (2) x = 2y 0 (v x + 2y 0).
Khi , (1) tr thnh: x
4
x
3
+ 3x
2
2x 1 = 0 (x 1)(x
3
+ 3x + 1) = 0
x = 1 (v x
3
+ 3x + 1 1 > 0 x 0)
1
y .
2
=
Vy nghim ca h cho l (x = 1; y =
1
2
).

0,5



136
S GD & T TR VINH K THI TUYN SINH VO LP 10 THPT
--------------------------------- NM HC 2011 2012
----------------------------
Mn thi: TON
Thi gian lm bi: 120 pht ( khng k thi gian giao
).

Bi 1: ( 1,5 im )
Cho biu thc A =
1 1
1
1 1 x x
+
+

1) Rt gn biu thc A.
2) Tm x A = - 3

Bi 2: ( 1,0 im )
Gii h phng trnh:

2 3 13
3 2 5 6
x y
x y

+ =

+ =


Bi 3: ( 2,5 im )
Cho hai hm s
2
2
x
y = v y = 1
2
x

1).V th ca hai hm s ny trn cng mt mt phng ta .

b) Tm ta giao im ca hai th .

Bi 4: ( 2,0 im )
Cho phng trnh: x
2
2(m + 4 )x + m
2
8 = 0 (1) , vi m l tham s.
1) Tm m phng trnh (1) c hai nghim phn bit l x
1
v x
2
.
2) Tm m x
1
+ x
2
3x
1
x
2
c gi tr ln nht.

Bi 5: ( 3,0 im )
T mt im M ngoi ng trn O bn knh R, v hai tip tuyn MA, MB n ng trn O bn
knh R ( Vi A, B l hai tip im ). Qua A v ng thng song song vi MB ct ng trn tm O ti E.
on ME ct ng trn tm O ti F. Hai ng thng AF v MB ct nhau ti I.

a) Chng minh t gic MAOB ni tip ng trn.
b) Chng minh IB
2
= IF.IA.
c) Chng minh IM = IB.

HNG DN CHM THI

BI P N IM
Bi 1

(1,5 im)
1)
1 ( 1) 1
1
x x x
A
x
+ +
=

( iu kin: 0, 1 x x > = )

0,25

1
1
x
x
+
=



0,5
chnh thc

137
2) C A = -3
1
3
1
x
x
+
=



0,25

iu kin 1 x =
0,25

1
2
x =
0,25

Bi 2

(1.0 im )
H Pt
2 6 13 2
3 6 15 2
x y
x y

+ =


0,25

2 2 x =
0,25

3 3 y =
0,25

Vy h phng trnh cho c nghim ( 2 2;3 3 )
0,25

Bi 3

( 2,5 im)
1) ( P) :
2
2
x
y =
Tp xc nh D = R


x -2 -1 0 1 2 +
2
2
x
y =

-2
1
2

0
1
2

-2







0,25


(d): y =
1
1
2
x
Cho x = 0 y = -1, A( 0;-1)
Cho x = 2 y = 0, B( 2;0)
ng thng (d) i qua hai im A( 0;-1), B( 2;0)




0,25
th


















0.25
(P)
(d)
-1 3 -3 2 0 1
-
1
2
-2

138

2) Phng trnh honh giao im ca ( d ) v ( P ) c :

2
1
2 2
x x
=


0.25

2
2 0 + = x x
0.25

1
2
x
x
=


0.25
Vi
1
1
2
x y

= =
x = -2 y = -2
Vy (d) ct (P) ti hai im M (
1
1;
2
) , N ( -2; -2)


0.25
Bi 4

(2,0 im)
1)
/
8 24 m A = +
0.25
Phng trnh (1) c hai nghim phn bit
/
0 A 8 24 0 m +
0.25
3 m 0.5
2) C : x
1
+ x
2
3x
1
.x
2
= -3m + 2m + 32 0,25
2
1 97 97
3
3 3 3
m
| |
= + s
|
\ .

0.5
Du = xy ra
1
3
m =
Vy
1
3
m = th x
1
+ x
2
3x
1
x
2
t GTLN




0,25
Bi 5

(3,0 im)
V n:
A


E F
0 M


I
B


1) C MA l tip tuyn
Nn OA MA
0
90 = OAM
Tng t
0
90 = OBM



0,25
0
180 + = OAM OBM
0,5
T gic MAOB ni tip ng trn c ng knh l OM. 0,25
Xt AIBA v AIFB
C : BIA l gc chung



139
= IAB IBF ( cng bng
1
2
s o BF )
AIBA ng dng AIFB

0.25
IB IA
IF IB
=
0.25
2
. (1) IB IF IA =
0.25
3) Ta c : AE // MB ( gt)
Nn = IMF MEA
M = MEA FAM
= IMF FAM
Xt AIMF v AIAM
C IAM l gc chung
= IMF IAM ( Chng minh trn )
AIMF ng dng AIAM



0.25
IM IA
IF IM
=
2
. IM IAIF = (2)


0.25
T (1) v ( 2 ) IB
2
= IM
2

IB = IM (pcm)

0.5





























140









chnh thc

141
S GIO DC & O TO
TNH KIN GIANG
---------------
CHNH THC
t c 01 trang
K THI TUYN SINH VO LP 10 TRNG CHUYN
NM HC 2012-2013
--------------------
Mn t: TON Kng cuyn
T gan: 120 pt Kng k t gan gao
Ngy t: 2562012

1. 15 m
1/ Rt gn: A = (3 2 11)(3 2 11) + + +
2/ Chng minh rng vi a khng m, a khc 1, b ty , ta c:
ab + a - b a- 1 b a + 1
a - 1
1 + a
=
2. 15 m
Cho (d
m
):
1
(1 )( 2)
2
m
y x m m
m

= + +
+

1/ Vi gi tr no ca m th ng thng (d
m
):
1
(1 )( 2)
2
m
y x m m
m

= + +
+
vung gc vi ng thng
(d):
1
3
4
y x =
(Cho bit hai ng thng vung gc vi nhau khi v ch khi tch h s gc bng -1)
2/ Vi gi tr no ca m th (d
m
) l hm s ng bin.

3. 3 m
1/ Chng minh rng phng trnh sau c 2 nghim phn bit
1 2
, x x vi mi gi tr m:
2
( 1) 3 0. x m x m + = Xc nh cc gi tr ca m tha mn :
2 2
1 2 2 1
3 x x x x + =
2/ Mt phng hp c 360 ch ngi v c chia thnh cc dy c s ch ngi bng nhau. Nu thm
cho mi dy 4 ch ngi v bt i 3 dy th s ch ngi trong phng khng thay i. Hi ban u s ch ngi
trong phng hp c chia thnh bao nhiu dy?

4. 1 m
Cho tam gic ABC vung ti A, ng cao AH. Tnh chu vi tam gic ABC, bit rng:
CH = 20,3cm. Gc B bng 62
0
. (Chnh xc n 6 ch s thp phn).

5. 3 m
Cho ng trn (O, 4cm), ng knh AB. Gi H l trung im ca OA, v dy CD vung gc vi
AB ti H. Ly im E trn on HD (E H v E D), ni AE ct ng trn ti F.
a) Chng minh rng AD
2
= AE . AF
b) Tnh di cung nh BF khi HE = 1 cm (chnh xc n 2 ch s thp phn)
c) Tm v tr im E trn on HD s o gc EOF bng 90
0


------ HT ------






142
P N KHNG CHUYN

I NI DUNG
1.1
A =
2
2
(3 2 11)(3 2 11) (3 2) 11 9 6 2 2 11 6 2 + + + = + = + + =
1.2
Vi a 0, a 1 v b ty ta c:
ab + a - b a- 1 b a( a-1)+( a-1) (b a + 1)( a 1) b a 1
a - 1
(1 a)( a 1) (1 + a)( a 1) 1 a
+
= = =
+ +

2.1
(d
m
):
1 m
(1 - m)(m + 2)
m + 2
y x

= + ; (d):
1
3
4
y x =
(d
m
) (d)
1 - m 1
1
m + 2 4
= 1- m = -4(m + 2) (vi m -2 v m 1 )
3m = 9 m = 3
2.2
(d
m
) l hm s ng bin khi:
1 m > 0 m < 1
m + 2 > 0 m > -2
1 m
0 2 < m < 1
m + 2 1 m < 0 m > 1
(loi)
m + 2 < 0 m < - 2




>







3.1
Phng trnh:
2
(m - 1) m - 3 = 0 x x + c:
A= [-(m 1)]
2
4 . 1 (m 3) = m
2
2m + 1 4m + 12 = (m
2
6m + 9) + 4
= (m 3)
2
+ 4 > 0 vi m
Vy phng trnh cho c 2 nghim phn bit
1 2
, x x vi mi m
Theo nh l Vit ta c:
1 2
1 2
m - 1
(I)
= m - 3
x x
x x
+ =

.
Theo ta c:
2 2
1 2 2 1 1 2 1 2
3 ( ) 3 x x x x x x x x + = + = (1)
Thay h thc (I) vo (1) ta c: (m 1)(m 3) = 3 m
2
4m = 0 m(m 4) = 0

m = 0
m = 4


Vy vi m = 0 hoc m = 4 th phng trnh c 2 nghim tha mn:
2 2
1 2 2 1
3 x x x x + =
3.2
Gi x (dy) l s dy gh lc u c chia t s ch ngi trong phng hp
(k:x
*
N e v x > 3)
S ch ngi mi dy lc u:
360
x
(ch)
Do thm cho mi dy 4 ch ngi v bt i 3 dy v s ch ngi trong phng khng thay i nn ta c
phng trnh: (
360
x
+ 4)(x 3) = 360
x
2
3x 270 = 0
x = 18
x = -15 (loi)


Vy lc u s ch ngi trong phng hp c chia thnh 18 dy.
4
*Xt AABC ( A = 90
0
) c:

0 0 0 0
B C 90 C 90 62 28 + = = =

143
*Xt AAHC ( H = 90
0
) c: AC =
HC
CosC

*Xt AABC ( A = 90
0
) c: AB = AC.tanC =
HC
tanC
CosC

V BC =
2
AC HC
CosC Cos C
=
*Chu vi tam gic ABC l:
AB + AC + BC =
HC
tanC
CosC
+
HC
CosC
+
2
HC
Cos C

=
0
0 0
HC 1 20,3 1
(tanC + 1 + ) (tan28 1 ) 61,254908 (cm)
CosC CosC Cos28 Cos28
= + + ~
5
a. hng minh:
2
=AE . AF
*Ta c: AB CD AC AD (lin hgia k v dy cung) =
ADC AFD = (cc gc nt chn cc cung tng ng bng nhau)
*Xt AADE v AAFD c:
ADC AFD = (cm trn)
A : gc chung

ADE AFD A A ~ (g-g)
2
AD AF
AD AE . AF
AE AD
= =

b. Tnh di cung nh B khi 1cm (chnh c n 2 ch s thp phn)
*Ta c: AH = OH =
OA
2 ( )
2
cm = (V H l trung im ca OA v OA = 4cm)
*Xt AAHE (
0
H 90 = ) c: tg
HE 1
HAE
AH 2
= = BAF HAE = ~27
0 0
sBF 2.BAF 54 = ~

0
0 0
BF
.OA.n 2 . 54
180 180
l
t t
= ~ ~1,88 (cm) (Vi n = s
0
BF 54 ~ )

c. Tm v tr ca im trn on s o ca gc bng 90
0

*Xt AEAO c: EH l ng cao (EH AB) cng l ng trung tuyn (v AH = OH) nn AEAO
cn ti E 1 EAH O = .
*M
2 O
EAH BAF (c ng chn cung BF)
2
= =


2
0 0 0 0 0
1 2 1 2 2 2
O
*EOF 90 O O 90 (VO EOF O 180 ) O 90 3O 180
2
= + = + + = + = =


0 0
2 O 60 EAH 30 = = tan
HE
EAH
AH
= (v AEAH vung ti H)

0
2 3
HE = AH . tanEAH 2 . tan30 (cm)
3
= =
2 1
O
F
E
H
D
C
B A

2
1
O
O
2
=
C
H
B
A


144
Vy khi im E cch H mt khong HE =
2 3
3
(cm) trn on HD th
0
EOF 90 =


S GIO DC & O TO
TNH KIN GIANG
---------------
CHNH THC
t c 01 trang
K THI TUYN SINH VO LP 10 THPT
NM HC 2012 2013
--------------------
Mn t: TON
T gan: 120 pt Kng k t gan gao
Ngy t: 0672012
1. 15 m
1) Rt gn biu thc A = 112 - 45 - 63 + 2 20
2) Cho biu thc B =
x x x x
1 1
1 x 1 x
| || |
+
+ +
| |
| |
+
\ .\ .
, vi 0 x 1
a) Rt gn B
b) Tnh gi tr biu thc B khi x =
1
1 2 +

2. 15 m
Cho ng thng (d
m
) : y = - x + 1 m
2
v (D): y = x
1) V ng thng (d
m
) khi m = 2 v (D) trn cng h trc ta , nhn xt v 2 th ca chng.
2) Tm m d trc ta Ox, (D) v (d
m
) ng quy.

3. 15 m
Trong t quyn gp ng h ngi ngho, lp 9A v 9B c 79 hc sinh quyn gp c
975000 ng. Mi hc sinh lp 9A ng gp 10000 ng, mi hc sinh lp 9B ng gp 15000
ng. Tnh s hc sinh mi lp.

4. 15 m
Cho phng trnh:
2 2
2( 2) 5 4 0 x m x m m + + + + = (*)
1/ Chng minh rng vi m < 0 phng trnh (*) lun lun c 2 nghim phn bit
1 2
, x x .
2/ Tm m phng trnh (*) c hai nghim phn bit
1 2
, x x tha h thc
1 2
1 1
1
x x
+ =

5. 4 m
Cho na ng trn tm O ng knh AB v im C trn ng trn sao cho CA = CB. Gi
M l trung im ca dy cung AC; Ni BM ct cung AC ti E; AE v BC ko di ct nhau ti D.
a) Chng minh: DE . DA = DC . DB
b) Chng minh: MOCD l hnh bnh hnh
c) K EF vung gc vi AC. Tnh t s
MF
EF
?
d) V ng trn tm E bn knh EA ct ng trn (O) ti im th hai l N; EF ct AN ti I, ct
ng trn (O) ti im th hai l K; EB ct AN ti H. Chng minh: T gic BHIK ni tip c
ng trn.

145

------ HT ------

P N
BI NI DUNG
1.1
A = 112 - 45 - 63 + 2 20 4 7 - 3 5 - 3 7 + 4 5 7 + 5 = =
1.2 a) Vi 0 x 1 ta c:
B =
x x x x ( 1) x( x 1)
1 1 1 1 (1 + x)(1 - x) 1 x
1 x 1 x 1 x 1
x x
x
| || | | || |
+ +
+ + = + = =
| | | |
| | | |
+ +
\ .\ . \ .\ .

b) Ta c:
1 2 1
x = 2 1
2 1
1 2

= =

+
B = 1 - 2 1 2 - 2 + =
2.1 (d
m
) : y = - x + 1 m
2
v (D): y = x
*Khi m = 2 th (d
m
) tr thnh: y = -x 3
Xt (d
m
): y = x 3 ta c bng gi tr:
Xt (D): y = x ta c: x = 1 y = 1
* th ca (d
m
) v (D):












*Nhn xt: ng thng (D) v ng thng (d
m
) vung gc vi nhau v tch h s ca
chng bng -1
2.2 (d
m
) : y = - x + 1 m
2
v (D): y = x
Ta c (D) ct Ox ti O. Ox, (D) v (d
m
) ng quy th (d
m
) phi i qua O khi :
1 m
2
= 0 m = 1
Vy m = 1 th Ox, (D) v (d
m
) ng quy.
3 Gi x l s hc sinh lp 9A (x eN
*
v x < 79)
S hc sinh lp 9B l: 79 x (hc sinh)
Lp 9A quyn gp c: 10000x (ng)
Lp 9B quyn gp c: 15000(79 x) (ng)
Do c hai lp quyn gp c 975000 ng nn ta c phng trnh:
10000x + 15000(79 x) = 975000
x 0 -3
y -3 0

-3 -2 -1 1
-3
-2
-1
1
x
y
(D): y = x
(d
m
): y = -x - 3
O


146
10x + 15(79 x) = 975 -5x = - 210 x = 42
Vy lp 9A c 42 hc sinh; lp 9B c: 79 42 = 37 (hc sinh)
4
1/ Phng trnh:
2 2
2( 2) 5 4 0 x m x m m + + + + = (*)
Ta c:
'
A = [-(m + 2)]
2
(m
2
+ 5m + 4) = m
2
+ 4m + 4 m
2
5m 4 = -m
Vi m < 0
'
A = -m > 0 Phng trnh (*) lun lun c 2 nghim phn bit
1 2
, x x
2/ Theo nh l Vit ta c:
1 2
2
1 2
2(m 2)
m + 5m + 4
x x
x x
+ = +

(I)
Theo ta c:
1 2 1 2
1 2 1 2
1 1
1 0
x x x x
x x x x
+
+ = = (1)
Thay (I) vo (1) ta c:
2
2
2(m + 2) - (m + 5m + 4)
0
m + 5m + 4
= (k: m -1 v m -4)
2(m + 2) (m
2
+ 5m + 4) = 0
2m + 4 m
2
5m 4 = 0
m
2
+ 3m = 0
m(m + 3) = 0

m = 0 (loi vtr i k: m < 0)
m = -3 (tha iu kin: m < 0; m 1 v m -4)


= =


Vy vi m = -3 th phng trnh (*) c 2 nghim phn bit
1 2
, x x tha h thc:

1 2
1 1
1
x x
+ =
5. a. hng minh . . B
Ta c:
0
ACB 90 = (gc ni tip na ng trn (O))

0
ACD 90 = (v k b vi ACB)
Ta li c:

0
AEB 90 = (gc ni tip na ng trn (O))
DEB = 90
0
(v k b vi AEB )
Xt AADC v ABDE c:

0
ACD DEB 90 = = (cm trn)
D : gc chung
ADC BDE A A ~ (g-g)
DA DC
DE . DA = DC . DB
DB DE
=

b. hng minh l hnh bnh hnh
Ta c: MC = MA (gt) OM AC (lin h gia k v dy cung)
CDAC (v
0
ACD 90 = )
OM // CD (cng vung gc vi AC) (1)
Mt khc: ADAB c: BE v AC l hai ng cao ct nhau ti M M l trc tm
DM l ng cao th ba DM AB
I
H
N
K
M
F
D
C
B
A
E
O

DM // CO (2)

147
M: CA = CB CA CB CO AB =
T (1) v (2) suy ra: MOCD l hnh bnh hnh.

c. . Tnh t s
MF
EF
?
Xt AMFE v AMCB c:

0
MFE MCB 90 = =
FME BMC = (i nh)
MFE MCB A A ~ (g g)
MF MC
EF CB
=
Ta li c: AC = 2MC (gt). M: CB = CA CB = 2MC

MF MC MC 1
EF CB 2MC 2
= = =

d. hng minh t gic B ni tip c ng trn.
Ta c:
1
K sBE
2
= (gc ni tip ng trn tm (O)) (3)
Ta li c:
1
NHB (sBN sEA)
2
= + (gc c nh nm trong ng trn (O))
M : EA = EN (bn knh ng trn (E)) EA EN =

1 1 1
NHB (sBN sEA) (sBN sEN) sBE
2 2 2
= + = + = (4)
T (3) v (4) suy ra: K NHB =
M NHB l gc ngoi ti H ca t gic BIHK
Vy t gic BIHK ni tip c ng trn.






















148




S GIO DC V O TO
K THI TUYN SINH VO LP 10 THPT
QUNG NH Mn thi: TON

M : 201 (th sinh ghi m vo sau ch
bi lm)
Thi gian lm bi: 120 pht

Cu 1: (1.5 im): Cho biu thc::
2 2
1 1 1
:
1 2 1
m
P
m m m m m
+ | |
= +
|
+
\ .
vi
m 0 = ,m 1 =
a)Rt gn biu thc P
b) Tnh gi tr ca biu thc P khi x=
1
2


Cu 2:(1,5im) : Cho ba -ng thng(d
1
): y= 2x+1; (d
2
): y=3; (d
3
): y=kx+5
.
a) Xc nh to giao im ca hai -ng thng d
1
v d
2
.
b) Tm k ba -ng thng trn ng quy.

Cu 3:(2.5 im) Cho ph-ng trnh bc hai n x: x
2
-2(m-1)x+2m-4=0 (m l
tham s) (1)
a) Gii ph-ng trnh (1) khi m = 3
b)Chng minh rng ph-ng trnh (1) lun c hai nghim phn bit vi
mi m.
c) Gi x
1
,x
2
l hai nghim ca ph-ng trnh (1). Tm gi tr nh nht
ca biu thc
A = x
1
2
+x
2
2


Cu 4: (3,5 im): Cho -ng trn tm O, -ng knh AB=2R. Gi M l mt
im bt k trn na -ng trn( M khng trng vi A, B). V cc tip
tuyn Ax, By, Mz ca na -ng trn. -ng thng Mz ct Ax, By ln l-t
ti N v P. -ng thng AM ct By ti C v -ng thng BM ct Ax ti D.
a) Chng minh t gic AOMN ni tip -c trong mt -ng trn.
b) Chng minh N l trung im ca AD, P l trung im ca BC
c) Chng minh AD.BC = 4R
2


Cu 5: : (1,0im) Cho a, b, c l cc s d-ng . Chng minh rng :
8
16 25
>
+
+
+
+
+ b a
c
c a
b
c b
a
.







CHNH THC

149












S GIO DC V O TO
TY NINH
K THI TUYN SINH LP 10 THPT
NM HC 2012 2013
Mn t: TON(Khng chuyn)
Ngy thi : 02 thng 7 nm 2012
Thi gian lm bi: 120 pht (khng k thi gian giao )

Cu 1 : (1im) Thc hin cc php tnh
a) A 2. 8 = b) B 3 5 20 = +
Cu 2 : (1 im) Gii phng trnh:
2
2 8 0 x x = .
Cu 3 : (1 im) Gii h phng trnh:
2 5
3 10
x y
x y
=

+ =

.
Cu 4 : (1 im) Tm x mi biu thc sau c ngha:
a)
2
1
9 x
b)
2
4 x
Cu 5 : (1 im) V th ca hm s
2
y x =
Cu 6 : (1 im) Cho phng trnh
( )
2 2
2 m 1 m 3 0 x x + + + = .
a) Tm m phng trnh c nghim.
b) Gi
1
x ,
2
x l hai nghim ca phng trnh cho, tm gi tr nh nht ca biu thc
1 2 1 2
A x x x x = + + .
Cu 7 : (1 im) Tm m th hm s 3 m 1 y x = + ct trc tung ti im c tung bng 4.
Cu 8 : (1 im) Cho tam gic ABC vung ti A c ng cao l AH. Cho bit AB 3cm = , AC 4cm = .
Hy tm di ng cao AH.
Cu 9 : (1 im) Cho tam gic ABC vung ti A. Na ng trn ng knh AB ct BC ti D. Trn cung
AD ly mt im E. Ni BE v ko di ct AC ti F. Chng minh t gic CDEF l mt t gic ni tip.
Cu 10: (1 im) Trn ng trn (O) dng mt dy cung AB c chiu di khng i b hn ng knh.
Xc nh v tr ca im M trn cung ln AB sao cho chu vi tam gic AMB c gi tr ln nht.


I GII

Cu 1 : (1im) Thc hin cc php tnh.
a) A 2. 8 16 4 = = =
b) B 3 5 20 3 5 2 5 5 5 = + = + = .
Cu 2 : (1 im) Gii phng trnh.
chnh thc

150

2
2 8 0 x x = .
( ) ( )
2
' 1 1. 8 9 0 A = = > , ' 9 3 A = = .

1
1 3 4 x = + = ,
2
1 3 2 x = = .
Vy
{ } S= 4; 2 .
Cu 3 : (1 im) Gii h phng trnh.

2 5 5 15 3 3
3 10 3 10 9 10 1
x y x x x
x y x y y y
= = = =



+ = + = + = =

.
Vy h phng trnh cho c nghim duy nht
( ) 3;1 .
Cu 4 : (1 im) Tm x mi biu thc sau c ngha:
a)
2
1
9 x
c ngha
2
9 0 x =
2
9 x = 3 x = .
b)
2
4 x c ngha
2
4 0 x >
2
4 x s 2 2 x s s .
Cu 5 : (1 im) V th ca hm s
2
y x = .

BGT

x
2 1 0 1 2
2
y x = 4 1 0 1 4










Cu 6 : (1 im)
( )
2 2
2 m 1 m 3 0 x x + + + = .
a) Tm m phng trnh c nghim.
( ) ( )
2
2 2 2
' m 1 1. m 3 m 2m 1 m 3 2m 2 A = + + = + + = .
Phng trnh c nghim ' 0 A > 2m 2 0 > m 1 > .
b) Tm gi tr nh nht ca biu thc
1 2 1 2
A x x x x = + + .
iu kin m 1 > .
Theo Vi-t ta c :
1 2
2m 2 x x + = + ;
2
1 2
m 3 x x = + .
( )
2
2 2
1 2 1 2
A 2m 2 m 3 m 2m 5 m 1 4 4 x x x x = + + = + + + = + + = + + > .

min
A 4 = khi m 1 0 + = m 1 = (loi v khng tha iu kin m 1 > ).
Mt khc : ( ) ( )
2 2
A m 1 4 1 1 4 = + + > + + (v m 1 > ) A 8 > .

min
A 8 = khi m 1 = .
t lun : Khi m 1 = th A t gi tr nh nht v
min
A 8 = .
Cc 2: iu kin m 1 > .

151
Theo Vi-t ta c :
1 2
2m 2 x x + = + ;
2
1 2
m 3 x x = + .
2 2
1 2 1 2
A 2m 2 m 3 m 2m 5 x x x x = + + = + + + = + + .
V m 1 > nn
2 2
A m 2m 5 1 2.1 5 = + + > + + hay A 8 >
Vy
min
A 8 = khi m 1 = .
Cu 7 : (1 im)
th hm s 3 m 1 y x = + ct trc tung ti im c tung bng 4.
m 1 4 = m 5 = .
Vy m 5 = l gi tr cn tm.
Cu 8 : (1 im)

Ta c:
( )
2 2 2 2
BC AB AC 3 4 5 cm = + = + = .
AH.BC AB.AC =
( )
AB.AC 3.4
AH 2, 4 cm
BC 5
= = = .

Cc 2:

2 2 2
1 1 1
AH AB AC
= +

2 2 2 2 2 2
2
2 2 2 2 2
AB .AC 3 .4 3 .4
AH
AB AC 3 4 5
= = =
+ +
.
( )
3.4
AH 2, 4 cm
5
= = .
Cu 9 : (1 im)


GT
ABC A ,
0
A 90 = , na
AB
O;
2
| |
|
\ .
ct BC ti D,
E AD e , BE ct AC ti F.
KL CDEF l mt t gic ni tip
Ta c :
( ) ( )
1 1 1
C sAmB sAED sADB sAED sBD
2 2 2
= = =
( C l gc c nh ngoi ng trn).
Mt khc
1
BED sBD
2
= ( BED gc ni tip).

1
BED C sBD
2
= =
T gic CDEF ni tip c (gc ngoi bng gc i trong).
Cu 10: (1 im)

152


GT
( ) O , dy AB khng i, AB 2R < , M AB e
(cung ln).
KL
Tm v tr M trn cung ln AB chu vi tam gic
AMB c gi tr ln nht.

Gi P l chu vi MAB A . Ta c P = MA+MB+AB.
Do AB khng i nn
max
P ( )
max
MA+MB .
Do dy AB khng i nn AmB khng i. t sAmB o = (khng i).
Trn tia i ca tia MA ly im C sao cho MB= MC.
MBC A cn ti M 1 1 M 2C = (gc ngoi ti nh MBC A cn)
1 1
1 1 1 1 1
C M sAmB sAmB
2 2 2 4 4
o = = = = (khng i).
im C nhn on AB c nh di mt gc khng i bng
1
4
o .
C thuc cung cha gc
1
4
o dng trn on AB c nh.
MA+MB= MA+MC= AC (v MB= MC).
( )
max
MA+MB
max
AC AC l ng knh ca cung cha gc ni trn.

0
ABC 90 =
0
1 2
0
1 1
B B 90
C A 90

+ =


+ =

1 2 A B = (do 1 1 B C = ) AMB A cn M.
MA= MB MA MB = M l im chnh gia ca AB (cung ln).
Vy khi M l im chnh gia ca cung ln AB th chu vi MAB A c gi tr ln nht.















153



















S GIO DC V O TO K THI TUYN SINH VO LP 10 THPT
CAO NG NM HC 2012 - 2013
Mn thi: TON
Ngy thi : 22/06/2011
Thi gian lm bi: 120 pht


Cu 1: (4,0 im)
a) Tnh: 36 ; 81 .
b) Gii phng trnh: x 2 = 0.
c) Gii phng trnh: x
2
4x + 4 = 0.

Cu 2: (2,0 im)
Mt mnh vn hnh ch nht c chu vi 400m. Bit chiu di hn chiu rng 60m. Tnh chiu di v
chiu rng mnh vn .

Cu 3: (1,0 im)
Cho tam gic ABC vung ti A, bit AB = 3cm, AC = 4cm.
a) Tnh cnh BC.
b) K ng cao AH, tnh BH.

Cu 4: (2,0 im)
Cho ng trn tm O, bn knh R; P l mt im ngoi ng trn sao cho OP = 2R. Tia PO ct
ng trn (O; R) A (A nm gia P v O), t P k hai tip tuyn PC v PD vi (O; R) vi C, D l hai tip
im.
a) Chng minh t gic PCOD ni tip.
b) Chng minh tam gic PCD u v tnh di cc cnh tam gic PCD.

Cu 5: (1,0 im)
Tm gi tr nh nht ca biu thc: A =
2
2
4 1 x x
x
+

CHNH THC

154



















S GIO DC V O TO
LNG SN

K THI TUYN SINH LP 10 THPT
NM HC 2012 2013
Mn thi: TON
Thi gian lm bi: 120 pht (khng k thi gian giao )
Ngy thi: 27 tng 06 nm 2012
t gm: 01 trang

Cu I 2 m.
1.tnh gi tr biu thc:
A =
( )
2
3 1 1 + B =
12 27
3
+

2. Cho biu thc P =
1 1 1
2 :
1 1 1 1 1
x
x x x x
| |

|
+ +
\ .

Tm x biu thc P c ngha; Rt gn P . Tm x P l mt s nguyn

Cu II 2 m.
1. V th hm s : y = 2x
2

2. Cho phng trnh bc hai tham s m : x
2
-2 (m-1) x - 3 = 0
a. Gii phng trnh khi m= 2
b. Chng minh rng phng trnh lun c hai nghim phn bit x
1
; x
2
vi mi gi tr ca m.
Tm m tha mn
1 2
2 2
2 1
1
x x
m
x x
+ =
Cu III (1,5 m.
Trong thng thanh nin on trng pht ng v giao ch tiu mi chi on thu gom 10kg giy vn
lm k hoch nh. nng cao tinh thn thi ua b th chi on 10A chia cc on vin trong lp thnh hai
t thi ua thu gom giy vn. C hai t u rt tch cc. T 1 thu gom vt ch tiu 30%, t hai gom vt ch
tiu 20% nn tng s giy chi on 10A thu c l 12,5 kg. Hi mi t c b th chi on giao ch tiu
thu gom bao nhiu kg giy vn?
Cu IV (3,5 m.
CHNH THC

155
Cho ng trn tm O,ng knh AB, C l mt im c nh trn ng trn khc A v B. Ly D l
im nm gia cung nh BC. Cc tia AC v AD ln lt ct tip tuyn Bt ca ng trn E v F
a, Chng minh rng hai tam gic ABD v BFD ng dng
b, Chng minh t gic CDFE ni tip
c, Gi D
1
i xng vi D qua O v M l giao im ca AD v CD
1
chng minh rng sooe o gc
AMC khng i khi D chy trn cung nh BC
Cu V (1 m.
Chng minh rng Q =
4 3 2
3 4 3 1 0 x x x x + + > vi mi gi tr ca x

p n :
Cu I 2 m.
1. A.
( )
2
3 1 1 + = 3 B
12 27
3
+
= 5
2. K : x >1
P =
2
1 x

P l mt s nguyn
{ } 1 (2) 1; 2 x U e =
=>
{ } 2;5 x =
Cu II 2 m.
1. HS t v
2. a) x = -1 hoc x = 3
b ) C
2
' ( 1) 3 0 m m A = + > => Pt lun c 2 nghim phn bit
Theo Vi t c :
1 2
2 2 x x m + =

1 2
. 3 x x =
Theo bi :
1 2
2 2
2 1
1
x x
m
x x
+ =
=>
3 3 2
1 2 1 2
( 1)( ) x x m x x + = =>
2 2
1 2 1 2 1 2 1 2
( ) ( ) 3 ( 1)( ) x x x x x x m x x ( + + =


=>
2 2
(2 2) (2 2) 3.( 3) ( 1)( 3) m m m ( =

=>
2
(2 2) 4 8 13 9( 1) m m m m ( + =


=>
3 2 2
8 16 26 8 16 26 9 9 0 m m m m m m + + + = =>
3 2
8 24 33 17 0 m m m + =
=>
2
( 1)(8 16 17) 0 m m m + = =>
2
1
8 16 17 0( )
m
m m Vn
=

+ =


Vy m = 1 l gi tr cn tm
Cu III (1,5 m.
Gi s kg giy vn t 1 c b th chi on giao l x (kg) ( k : 0 < x <10)
S kg giy vn t 2 c b th chi on giao l y (kg) ( k : 0 < x <10 )
Theo u bi ta c hpt:
10
1, 3 1, 2 12, 5
x y
x y
+ =

+ =


Gii h trn ta c : (x; y ) = (5;5)
Tr li : s giy vn t 1 c b th chi on giao l 5 kg
S giy vn t 2 c b th chi on giao l 5 kg
Cu IV (3,5 m.
1. AABD v ABFD
c : ZADB= ZBDF = 90
0

ZBAD = ZDBF ( Cng chn cung BD)
=>
AABD ABFD
C
A
A
D
A
A
E
A
A
F
A
A
M

156
2. C : ZE = (SdAB- SdBC): 2 ( Gc ngoi ng trn)
= SdAC: 2
= ZCDA
=> T gic CDFE ni tip




3. D dng chng minh c t gic ADBD
1
l hnh ch nht
C : ZAMC = ZAD
1
M + ZMAD
1
( Gc ngoi tam gic AD
1
M)
= (SdAC: 2) + 90
0

M AC c nh nn cung AC c nh=> ZAMC lun khng i khi D chy trn cung nh BC
Cu V (1 m.

Q =
4 3 2
3 4 3 1 x x x x + +
=
4 3 2 2 3
( 2 ) (1 3 3 ) x x x x x x + + +
=
2 2 3
( 1) (1 ) x x x +
=
2 2
(1 ) ( 1) x x x + =
2 2
1 3
(1 ) ( )
4 4
x x x + + =
2 2
1 3
(1 ) ( ) 0
2 4
x x x
(
+ >
(


S GD & T HA NH K THI TUYN SINH VO LP 10 NM HC 2012- 2013
TRNG THPT CHUYN HONG VN TH
THI MN TON CHUNG
Ngy t: 29 tng 6 nm 2012
Thi gian lm bi: 120 pht (khng k thi gian giao )
t gm c 01 trang
---------------------------------------------------------------------------------------------------------

PHN I. TRC NGHIM(2 im)
(Th sinh khng cn gii thch v khng phi chp li bi, hy vit kt qu cc bi ton sau vo t giy
thi)
1. Biu thc A = 2 1 x + c ngha vi cc gi tr ca x l
2. Gi tr m 2 ng thng (d
1
): y = 3x 2 v (d
2
): y = mx + 3m 1 ct nhau ti 1 im trn trc tung
l....
3. Cc nghim ca phng trnh 3 5 1 x = l...
4. Gi tr ca m phng trnh x
2
(m+1)x - 2 = 0 c 2 nghim x
1
, x
2
tha mn
x
1
2
x
2
+ x
1
x
2
2
= 4 l...

PHN II. T LUN (8 im)

Bi 1. (2 im)
a) Gii h phng trnh
1 1
5
2 3
5
x y
x y

+ =


b) Cho tam gic ABC vung ti A (AB > AC). ng phn gic AD chia cnh huyn BC thnh 2 on
theo t l
3
4
v BC = 20cm. Tnh di hai cnh gc vung.

CHNH THC

157
Bi 2. (2 im) Tm mt s c hai ch s, bit rng ch s hng chc ln hn ch s hng n v l 5 v nu
em s chia cho tng cc ch s ca n th c thng l 7 v d l 6.

Bi 3.(3 im) Cho tam gic ABC c ba gc nhn ni tip trong ng trn tm O, bn knh R. Cc ng
cao AD, BE, CF ca tm gic ct nhau ti H. Chng minh rng:
a) T gic BCEF ni tip c.
b) EF vung gc vi AO.
c) Bn knh ng trn ngoi tip tam gic BHC bng R.

Bi 4. (1 im) Trn cc cnh ca mt hnh ch nht t ln lt 4 im ty . Bn im ny to thnh mt
t gic c di cc cnh ln lt l x, y, z , t. Chng minh rng
25 s x
2
+ y
2
+ z
2
+ t
2
s 50. Bit rng hnh ch nht c chiu di v chiu rng l 4 v 3.

P N

PHN I. TRC NGHIM(2 im)
1. Biu thc A = 2 1 x + c ngha vi cc gi tr ca x l:
1
2
x >
2. Gi tr m 2 ng thng (d
1
): y = 3x 2 v (d
2
): y = mx + 3m 1 ct nhau ti 1 im trn trc tung
l
1
3
m = .
3. Cc nghim ca phng trnh 3 5 1 x = l: x = 2; x =
4
3
.
4. Gi tr ca m phng trnh x
2
(m+1)x - 2 = 0 c 2 nghim x
1
, x
2
tha mn
x
1
2
x
2
+ x
1
x
2
2
= 4 l m = -3.

PHN II. T LUN(8 im)
Bi 1. (2 im)
a) Gii h phng trnh:
1 1
5 (1)
2 3
5 (2)
x y
x y

+ =


iu kin: , 0. x y =
Ly (1) cng (2) theo v, ta c:
3 2 2
0 3 2
3
x
y x y
x y
= = = , th vo (1) ta c pt:

1 3 5 1
5 5 2 1
2 2 2
x x
x x x
+ = = = = (tha mn k 0 x = )
Vi
1 1
2 3
x y = = (tha mn k 0 y = )
Vy h phng trnh cho c 1 nghim
1 1
( ; ) ( ; )
2 3
x y =
b) t di cnh AB = x (cm) v AC = y (cm); k: x > y > 0
Theo tnh cht ng phn gic v nh l pitago ta c:

158
D
B
A
C
2 2 2 2 2 2 2 2
3
3 3
4
4 4
9
20 16 20
16
y
y x
y x
x
x x x x y

= =




+ = = + =


3
12
4
16
16
y y x
x
x

= =


Vy di cnh AB = 16 (cm) ; AC = 14 (cm)

Bi 2. (2 im) Gi s cn tm c 2 ch s l ab , vi , {0,1, 2,3, 4,5, 6, 7,8,9}, 0 a b a e = .
Theo gi thit ta c h phng trnh:
5 5 5 5 8
10 7( ) 6 3 6 6 2 2 2 2 3
a b a b a b a b a
a b a b a b a b a b b
= = = = =



+ = + + = = = =

(t/m k)
Vy s cn tm l: 83

Bi 3.(3 im)
a) V BE, CF l ng cao ca tam gic ABC
0
; 90 BE AC CF AB BEC CFB = =
E, F thuc ng trn ng knh BC
T gic BCEF ni tip.
b) EF vung gc vi AO.
Xt AAOB ta c:

0 0
1 1
90 90
2 2
OAB AOB = = s
0
90 AB ACB = (1)
Do BCEF ni tip nn AFE ACB = (2)
T (1) v (2) suy ra:
0 0
90 90 OAB AFE OAB AFE OA EF = + = (pcm)
c) Bn knh ng trn ngoi tip ABHC bng R.
Gi ' ( ) H AH O = . Ta c:
0
90 ' ' HBC ACB HAC H AC H BC = = = = (3)
0
90 ' ' HCB ABC HAB H AB H CB = = = = (4)
T (3) v (4) ' ( . . ) BHC BH C g c g A = A
M ABH'C ni tip ng trn tm O, bn knh R ABHC cng ni tip ng trn c bn knh R,
tc l bn knh ng trn ngoi tip ABHC bng R.

Bi 4. (1 im) Gi s hnh ch nht c di cc cnh c t nh
hnh v.
Vi: 0s a, b, e, f 4 s v a+b = e+f = 4;
0s c, d, g, h 3 s v c+d = g+h = 3.
Ta c:
2 2 2 2 2 2 2 2 2 2 2 2
; ; ; x h a y b c z d e t f g = + = + = + = +
2 2 2 2 2 2 2 2 2 2 2 2
( ) ( ) ( ) ( ) x y z t a b c d e f g h + + + = + + + + + + + (*)

159
- Chng minh:
2 2 2 2
50 x y z t + + + s .
V , 0 a b > nn
2 2 2
( ) 16 a b a b + s + = . Tng t:
2 2 2 2 2 2
9; 16; 9 c d e f g h + s + s + s .
T (*)
2 2 2 2
16 9 16 9 50 x y z t + + + s + + + = (1)
- Chng minh:
2 2 2 2
25 x y z t + + + > .
p dng bt ng thc Bu - nhi - a- cp xki , ta c:
2
2 2 2 2 2 2 2
( ) 16
(1 1 )( ) (1. 1. )
2 2
a b
a b a b a b
+
+ + > + + > =

Tng t:
2 2 2 2 2 2
9 16 9
; ;
2 2 2
c d e f g h + > + > + > .
T (*)
2 2 2 2
16 9 16 9
25
2 2 2 2
x y z t + + + > + + + = (2)
T (1) v (2)
2 2 2 2
25 50 x y z t s + + + s (pcm)











S GD & T HA NH THI TUYN SINH VO 10 NM HC 2012-2013
THI MN: TON
Ngy thi: 19/ 07/ 2012
Thi gian lm bi: 120 pht (khng k thi gian giao )


Cu 1. (3,0 im)

1. Tm iu kin c ngha ca biu thc:
a)
1
1 x
; b) 2 x .

2. Phn tch a thc thnh nhn t :
a)
2
5 + x x ; b)
2 2
7 10 + x xy y

3. Cho tam gic ABC vung ti A; AB = 2 cm, AC = 4 cm. Tnh di cnh BC.


Cu 2. (3,0 im)

1. Gii phng trnh: 2(x + 5) + (x 3)(x + 3) = 0.

2. a) V th hm s y = 3x + 2 (1).

CHNH THC

160
b) Gi A, B l giao im ca th hm s (1) vi trc tung v trc honh.
Tnh din tch tam gic OAB.

Cu 3. (1,0 im) Mt phng hp c 320 gh ngi c xp thnh tng dy v s gh mi dy u bng
nhau. Nu s dy gh tng tng thm 1 v s gh mi dy tng thm 2 th trong phng c 374 gh. Hi trong
phng c bao nhiu dy gh v mi dy c bao nhiu gh?

Cu 4. (2,0 im)

Cho ng trn tm O, bn knh R v im M sao cho MO = 2R. Qua im M k cc tip tuyn MA,
MB vi ng trn (O). Hai ng cao BD v AC ca tam gic MAB ct nhau ti H
1) Chng minh t gic AHBO l hnh thoi.
2) Tnh gc AMB.

Cu 5. (1,0 im) Cho hai s thc x, y tha mn:
2 2
+ s + x y x y . Chng minh rng: 2 + s x y

Ht




P N THI TUYN SINH MN TON VO 10 HA NH NM HC 2012-2013

Cu 1. (3,0 im)

1. Tm iu kin c ngha ca biu thc:
a) iu kin: x 1 0 x 1 = = ; b) iu kin: x 2 0 x 2 > >

2. Phn tch a thc thnh nhn t :
a)
2
5 ( 5) + = + x x x x ;
b) Cch 1: Phng php tch, thm bt s hng:

2 2 2 2
7 10 ( 2 ) (5 10 ) ( 2 ) 5 ( 2 ) ( 2 )( 5 ) + = = = x xy y x xy xy y x x y y x y x y x y
Cch 2: S dng nh l: Nu pt bc hai
2
ax bx c 0(a 0) + + = = c 2 nghim phn bit x
1
, x
2
th:
2
1 2
ax bx c a(x x )(x x ) + + = .
p dng vo bi ton trn ta xem pt:
2 2
7 10 0 + = x xy y nh l 1 pt bc hai n x, tham s y.
Ta c
2 2 2
(7y) 4.10y 9y 3y A = = A = ;
1 2
7y 3y 7y 3y
x 2y; x 5y
2 2
+
= = = =
Suy ra:
2 2
7 10 ( 2 )( 5 ) + = x xy y x y x y

3. Cho tam gic ABC vung ti A; AB = 2 cm, AC = 4 cm. Tnh di cnh BC.

V tam gic ABC vung ti A, nn theo nh l Pitago ta c:
C
4

c
m


161


2 2 2 2 2
BC AB AC 2 4 20 BC 20 2 5(cm) = + = + = = =


Cu 2. (3,0 im)
1. Gii phng trnh:
( ) ( )( ) 2 x+5 x 3 x 3 0 + + =

2
2
2
2x 10 x 9 0
x 2x 1 0
(x 1) 0
x 1 0
x 1
+ + =
+ + =
+ =
+ =
=


2. a) V th hm s y = 3x + 2 (1).

+ Cho x 0 y 2 = =
+ Cho
2
y 0 x
3
= =
+ th hm s y = 3x + 2 l mt ng thng i qua 2 im (0;2) v
2
( ;0)
3

b) T cch v th hm s y = 3x + 2 ta c:
+ Giao ca th hm s (1) vi trc Oy l A(0;2)
+ Giao ca th hm s (1) vi trc Ox l B
2
( ;0)
3

Suy ra din tch AOAB l :
OAB
1 1 2 2
S OA.OB . | 2 | . | |
2 2 3 3
A
= = = (vdt)
Cu 3. (1,0 im) Mt phng hp c 320 gh ngi c xp thnh tng dy v s gh mi dy u bng
nhau. Nu s dy gh tng tng thm 1 v s gh mi dy tng thm 2 th trong phng c 374 gh. Hi trong
phng c bao nhiu dy gh v mi dy c bao nhiu gh?
Gii: Gi s dy gh trong phng hp l x (dy) (
*
x e )
Gi s gh trong mi dy l y (gh) (
*
y e )
V phng hp c 320 gh ngi c xp thnh tng dy v s gh mi dy u bng nhau nn ta c
phng trnh: xy 320 = (1)
V s dy gh tng tng thm 1 v s gh mi dy tng thm 2 th trong phng c 374 gh nn ta c
phng trnh: (x 1)(y 2) 374 + + = (2)
T (1) v (2) ta c h phng trnh:
xy 320
(x 1)(y 2) 374
=

+ + =


2
320
320
y
y xy 320 xy 320
x
x
xy 2x y 2 374 2x y 52 320
2x 52 x 26x 160 0
x

= = =



+ + + = + =


+ = + =


O x
y
2
A
B
2
3


162
2 2
320 320
y y x=10
x x
y 32
x 26x 160 0 x 26x 160 0

= =




=


+ = + =

hoc
x=16
y 20



Vy trong phng hp c 10 dy gh v mi dy c 32 gh
Hoc l trong phng hp c 16 dy gh v mi dy c 20 gh

Cu 4. (2,0 im)

Cho ng trn tm O, bn knh R v im M sao cho MO = 2R. Qua im M k cc tip tuyn MA,
MB vi ng trn (O). Hai ng cao BD v AC ca AMAB ct nhau ti H.
1) Chng minh t gic AHBO l hnh thoi.

Ta c: OAMA (V MA l tip tuyn vi ng trn (O))
BHMA ( V BH l ng cao trong AMAB)
OA // BH (1)
Tng t ta c:
OB MB
OB/ /AH
AH MB

(2)
T (1) & (2) suy ra t gic AHBO l hnh bnh hnh,
mt khc li c OA = OB nn t gic AHBO l hnh thoi.
2) Tnh gc AMB.
D thy MO l ng phn gic trong ca gc AMB AMB 2AMO = .
V tam gic OAM vung ti A nn ta c:
0
OA 1
sin AMO AMO 30
MO 2
= = =
0
AMB 60 = .
Cu 5. (1,0 im) Cho hai s thc x, y tha mn:
2 2
+ s + x y x y . Chng minh rng: 2 + s x y
Cch 1:
Nhn xt:
2
(x y)
xy ; x, y
4
+
s e .
Tht vy:
2
2 2
(x y)
xy (x y) 4xy (x y) 0; x, y
4
+
s + > > e (ng)
Do t gi thit:
2 2
+ s + x y x y

2
( ) 2 + s + + x y x y xy

2
2
( )
( )
2
+
+ s + +
x y
x y x y

2
( ) 2( ) + s + x y x y
C
D
B
A
M O
H

163
( )( 2) 0 + + s x y x y (*)
V
2 2
0; , + > + > e x y x y x y , nn ta xt cc trng hp sau:
- Nu
2 2
0 0 0 2 + = = = + = s x y x y x y
- Nu
2 2
0 0 + = + > x y x y , t (*) suy ra: 2 0 2 + s + s x y x y
T suy ra: 2 + s x y . Du bng xy ra khi x = y = 1.
Cch 2: p dng BT Bu nhi a cp xki: x, y e , ta c:
2 2 2 2 2
(1.x 1.y) (1 1 )(x y ) + s + +
2 2 2
(x y) 2(x y ) + s +
2
(x y) 2(x y) + s +
(x y)(x y 2) 0 + + s (*)
V
2 2
0; , + > + > e x y x y x y , nn ta xt cc trng hp sau:
- Nu
2 2
0 0 0 2 + = = = + = s x y x y x y
- Nu
2 2
0 0 + = + > x y x y , t (*) suy ra: 2 0 2 + s + s x y x y
T suy ra: 2 + s x y . Du bng xy ra khi x = y = 1.




S GIO DC V O TO K THI TUYN SINH LP 10 THPT CHUYN
QUNG NAM Nm c: 2012 2013
Ka t: Ngy 4 tng 7 nm 2012
Mn: TON (Ton chung)
Thi gian lm bi: 120 pht ( khng k thi gian giao )

Cu 1: (2,0 im)
Cho biu thc:
( )
x 2 3x 3
A 4x 12
x 3
| |
+
= +
|

\ .
.
a) Tm iu kin ca x biu thc A c ngha.
b) Rt gn biu thc A.
c) Tnh gi tr ca A khi x 4 2 3 = .
Cu 2: (2,0 im)
a) Xc nh cc h s a, b ca hm s y = ax + b, bit th ca n l ng thng song song vi ng
thng y = 2x + 1 v i qua im M(1 ; 3).
b) Gii h phng trnh (khng s dng my tnh cm tay):
CHNH THC

164

2x y 3
2x y 1

+ =


Cu 3: (2,0 im)
Cho parabol (P):
2
1
y x
2
= v ng thng (d): y = (m 1)x 2 (vi m l tham s).
a) V (P).
b) Tm m (d) tip xc vi (P) ti im c honh dng.
c) Vi m tm c cu b), hy xc nh ta tip im ca (P) v (d).
Cu 4: (4,0 im)
Cho tam gic ABC vung ti A. Qua C k ng thng d vung gc vi AC. T trung im M ca cnh
AC k ME vung gc vi BC (E thuc BC), ng thng ME ct ng thng d ti H v ct ng thng AB
ti K.
a) Chng minh: AMK = CMH, t suy ra t gic AKCH l hnh bnh hnh.
b) Gi D l giao im ca AH v BM. Chng minh t gic DMCH ni tip v xc nh tm O ca ng
trn ngoi tip t gic .
c) Chng minh: AD.AH = 2ME.MK.
d) Cho AB = a v
0
ACB 30 = . Tnh di ng trn ngoi tip t gic DMCH theo a.






S GIO DC V O TO K THI TUYN SINH LP 10 THPT CHUYN
QUNG NAM Nm c: 2012 2013
Kha thi: Ngy 4 tng 7 nm 2012
Mn: TON (Ton chung)
Thi gian lm bi: 120 pht ( khng k thi gian giao )
HNG DN CHM THI
(Bn hng dn ny gm 02 trang)
Cu N dung m
Cu 1
(2,0)
a)
(0,5)
iu kin: x 0
v x = 3
0,25
0,25
b)
(1,0)
Bin i c:
( )
2
2 3 3 3 x x x + =

( )( )
( )
3 3 3
4 12 2 3
x x x
x x
= +
+ = +

A =
( )
( )( )
( ) ( )
2
3
.2 3 2 3
3 3
x
x x
x x

+ =
+

0,25

0,25

0,25


0,25
c)
(0,5)
Bin i c:
( )
2
4 2 3 3 1 x = =
Tnh c: A = 2
0,25

0,25
CHNH THC

165
Cu 2
(2,0)
a)
(1,0)
+ V ng thng y = ax + b song song vi ng thng y = 2x + 1 nn a =
2 (khng yu cu nu b 1)
+ Thay ta im M (1 ; 3) v a = 2 vo y = ax + b
+ Tm c: b = 1

0,5
0,25
0,25
b)
(1,0)
2 3
2 1
x y
x y

+ =


2 2
2 3
y
x y
=


+ =


Tnh c: y = 1
x = 2
Vy nghim ca h phng trnh cho l: (x ; y) = ( 2 ; 1)

0,25

0,25
0,25

0,25
Cu 3
(2,0)
a)
(0,5)
+ Lp bng gi tr ng (chn ti thiu 3 gi tr ca x trong phi c gi tr x =
0).
+ V ng dng ca (P).
0,25

0,25
b)
(1,0)
+ Phng trnh honh giao im ca (P) v (d):

2
1
x (m 1)x 2
2
=
x
2
2(m 1)x +4 = 0
+ Lp lun c:
( )
2
' 0
1 4 0
'
0
1 0
A =

=


>
>

m
b
m
a


= =

>

m 1 hoc m 3
m 1

+ Kt lun c: m = 3



0,25


0,25


0,25

0,25
c)
(0,5)
+ Tm c honh tip im:
b' m 1 3 1
x 2
a 1 1

= = = =
+Tnh c tung tip im: y = 2 v kt lun ng ta tip im l (2; 2).

0,25
0,25


Cu N dung m
Cu 4
(4,0)
Hnh
v
(0,25)























0,25
a)
(1,0)
+ AM = MC (gt) ,
0
KAM HCM 90 , AMK CMH = = = ()
+
( ) AMK CMH g.c.g A = A
0,25
0,25

166
+ suy ra: MK = MH
+ V MK = MH v MA = MC nn t gic AKCH l hnh bnh hnh.
0,25
0,25
b)
(1,0)
+ Nu c: CA BK v KE BC , suy ra M l trc tm tam gic KBC.
+ Nu c: KC // AH v BM KC, suy ra BM AH.
+
0 0 0
HDM HCM 90 90 180 + = + = => T gic DMCH ni tip.
+
0
MCH 90 = => Tm O ca ng trn ngoi tip t gic DMCH l trung
im MH.

0,25
0,25
0,25

0,25
c)
(1,0)
+ Chng minh c hai tam gic ADM v ACH ng dng (g.g)
+ ( )
2
. . 2 . AC=2AM
AM AD
AM AC AH AD AM AH AD v
AH AC
= = =
2
.
(1)
2
AH AD
AM =
+ Ta li c: MC
2
= ME.MH v MH=MK nn MC
2
= ME.MK (2)
+ Mt khc: MC = MA (gt) (3)
T (1), (2), (3) =>
.
.
2
AH AD
ME MK = => AH.AD = 2ME.MK
0,25



0,25

0,25


0,25
d)
(0,75)
+ AABC vung ti A, gc C = 30
0
nn AC = a 3 .
+
0
ACB MHC 30 = = (cng ph gc CMH) => MH = 2MC
M AC = 2MC nn: MH = AC = a 3 .
+ di ng trn ngoi tip t gic DMCH l:

MH a 3
C 2 2 a 3
2 2
| |
| |
= t = t = t
|
|
\ .
\ .

0,25


0,25


0,25



d
(0,75)
+ Tam gic ABC vung ti A nn: AC = AB.cotC = a 3 .
+
0 0
CMH 90 ACB 60 = =
=>
0
MC AC
MH AC a 3
cos 2cos60 CMH
= = = =
Din tch hnh trn (O):
+
2
2
2
(O)
MH a 3 3
S a
2 2 4
| |
| |
= t = t = t
|
|
\ .
\ .

0,25


0,25

0,25













167






























S GIO DC V O TO K THI TUYN SINH LP 10 THPT CHUYN
QUNG NAM Nm c: 2012 2013
Ka t: Ngy 4 tng 7 nm 2012
Mn: TON (Chuyn Ton)
Thi gian lm bi: 150 pht (khng k thi gian giao )


Cu 1: (1,5 im)
a) Rt gn biu thc: A =
a a 6 1
4 a a 2



(vi a 0 v a 4).
b) Cho
28 16 3
x
3 1

. Tnh gi tr ca biu thc:


2 2012
P (x 2x 1) = + .
Cu 2: (2,0 im)
a) Gii phng trnh: 3(1 x) 3 x 2 + = .
CHNH THC

168
b) Gii h phng trnh:
2
2
x xy 4x 6
y xy 1

+ =

+ =


Cu 3: (1,5 im)
Cho parabol (P): y = x
2
v ng thng (d): y = (3 m)x + 2 2m (m l tham s).
a) Chng minh rng vi m 1 th (d) lun ct (P) ti 2 im phn bit A, B.
b) Gi y
A
, y
B
ln lt l tung cc im A, B. Tm m |y
A
y
B
| = 2.
Cu 4: (4,0 im)
Cho hnh ch nht ABCD c AB = 4 cm, AD = 2 cm. ng thng vung gc vi AC ti C ct cc
ng thng AB v AD ln lt ti E v F.
a) Chng minh t gic EBDF ni tip trong ng trn.
b) Gi I l giao im ca cc ng thng BD v EF. Tnh di on thng ID.
c) M l im thay i trn cnh AB (M khc A, M khc B), ng thng CM ct ng thng AD ti N.
Gi S
1
l din tch tam gic CME, S
2
l din tch tam gic AMN. Xc nh v tr im M
1 2
3
S S
2
= .
Cu 5: (1,0 im)
Cho a, b l hai s thc khng m tha: a + b 2.
Chng minh:
2 a 1 2b 8
1 a 1 2b 7
+
+ >
+ +
.

--------------- Ht ---------------





S GIO DC V O TO K THI TUYN SINH LP 10 THPT CHUYN
QUNG NAM Nm c: 2012 2013
Ka t: Ngy 4 tng 7 nm 2012
Mn: TON (Chuyn Ton)
Thi gian lm bi: 150 pht ( khng k thi gian giao )

HNG DN CHM THI
(Bn hng dn ny gm 03 trang)
Cu N dung m
Cu 1
15 m

a) (0,75) A =
a a 6 1
4 a a 2



(a 0 v a 4)
A =
( a 2)( a 3) 1
(2 a)(2 a) a 2
+

+

=
a 3 1
2 a 2 a

+


= 1

0,25

0,25

0,25
CHNH THC

169
b) (0,75) Cho
28 16 3
x
3 1

. Tnh:
2 2012
P (x 2x 1) = +
2 2
(4 2 3) 4 2 3 ( 3 1)
x
3 1 3 1 3 1

= = =

= 3 1

2
x 2x 1 1 + =

2 2012
P (x 2x 1) 1 = + =

0,25

0,25

0,25
Cu 2
20 m


a) (1,0) Gii phng trnh: 3(1 x) 3 x 2 + = (1)
Bnh phng 2 v ca (1) ta c:
3(1 x) 3 x 2 3(1 x)(3 x) 4 + + + =
3(1 x)(3 x) 1 x + =

2
3(1 x)(3 x) 1 2x x + = +

2
x x 2 0 + = x = 1 hoc x =2
Th li, x = 2 l nghim .

0,25


0,25
0,25

0,25
b) (1,0) Gii h phng trnh:
2
2
x xy 4x 6 (1)
y xy 1 (2)

+ =

+ =

(I)
Nu (x;y) l nghim ca (2) th y 0.
Do : (2)
2
y 1
x
y

= (3)
Thay (3) vo (1) v bin i, ta c:
4y
3
+ 7y
2
+ 4y + 1 = 0
(y + 1)(4y
2
+ 3y + 1) = 0 (th sinh c th b qua bc ny)
y = 1
y = 1 x = 2
Vy h c mt nghim: (x ; y) = (2 ; 1).
0,25

0,25


0,25



0,25



Cu N dung m
Cu 3
15 m

a) (0,75) (P): y = x
2
, (d): y = (3 m)x + 2 2m.
Chng minh rng vi m 1 th (d) lun ct (P) ti 2 im phn bit A, B
Phng trnh honh giao im ca (P) v (d):
x
2
= (3 m)x + 2 2m.
x
2
+ (3 m)x + 2 2m = 0 (1)
A = (3m)
2
4(2 2m) = m
2
+ 2m + 1
Vit c: A = (m + 1)
2
> 0, vi m 1 v kt lun ng.


0,25
0,25
0,25
b) (0,75) Tm m |y
A
y
B
| = 2 .
Gii PT (1) c hai nghim: x
1
= 2 v x
2
= m 1
Tnh c: y
1
= 4, y
2
= (m 1)
2

|y
A
y
B
| = |y
1
y
2
| = |m
2
2m3|
|y
A
y
B
| = 2 m
2
2m 3 = 2 hoc m
2
2m 3 = 2
m = 1 6 hoc m = 1 2
0,25


0,25



170
0,25
Cu 4
40 m













a) (1,0) Chng minh t gic EBDF ni tip trong ng trn.
Ta c:
ADB ACB =
AEC ACB = ( cng ph vi BAC)
ADB AEC =
t gic EBDF ni tip











0,25
0,25
0,25
0,25

b) (1,5) Tnh ID
Tam gic AEC vung ti C v BC AE nn: BE.BA = BC
2


2
BC
BE 1
BA
= =
BE//CD
IB BE 1
ID CD 4
= =

BD 3
ID 4
=

4
ID BD
3
= v tnh c: BD = 2 5

8 5
ID
3
= (cm)
0,25

0,25

0,25

0,25


0,25

0,25

Cu N dung m
Cu 4
(tt)
c) (1,5 im) Xc nh v tr im M S
1
=
3
2
S
2

t AM = x, 0 < x < 4
MB = 4 x , ME = 5 x
Ta c:
AM .AM 2.
MB MB 4
AN BC x
AN
BC x
= = =



1
1
S BC.ME 5 x
2
= = ,
2
2
1 x
S AM.AN
2 4 x
= =


S
1
=
3
2
S
2
5 x =
3
2
.
2
x
4 x
x
2
+ 18x 40 = 0
x = 2 (v 0 < x < 4)
Vy M l trung im AB .
0,25
0,25
0,25

0,25

0,25


0,25

171
Cu 5
10 m

Cho a, b 0 v a + b 2. Chng minh :
2 a 1 2b 8
1 a 1 2b 7
+
+ >
+ +

Bt ng thc cn chng minh tng ng vi:
1 2 8
1 1 2 7
+ >
+ + a b

Ta c:
1 2
1 2 1 a b
+
+ +
=
1 1 1
2
1
1 1
( 1)( )
2
2
a
b
a b
+ >
+
+
+ +
(1) (bt Csi)

1
1
1 7
2
( 1)( )
2 2 4
+ + +
+ + s s
a b
a b (bt C si)

2 8
7 1
( 1)( )
2
>
+ + a b
(2)
T (1) v (2) suy ra:
1 2 8
1 1 2 7
+ >
+ + a b

Du = xy ra ch khi : a + 1 = b +
1
2
v a + b = 2 a =
3
4
v b =
5
4


0,25

0,25



0,25





0,25














172












CHNH THC

173









174






175









176
S GIO DC O TO K THI TUYN SINH VO LP 10 THPT
VNH LONG NM HC 2012 2013
Mn thi : TON
Thi gian lm bi : 120 pht, khng k thi gian giao


Cu 1: 25 m Gii phng trnh v h phng trnh:
a) 2x 1 = 3
b)
2
12 35 0 + = x x
c)
2 3 13
3 9
+ =

+ =

x y
x y


Cu 2: 25 m
a) V ng thng (d): y = 2x 1
b) Chng minh rng ng thng (d) tip xc vi parabol (P): y = x
2

c) Tm a v b ng thng (d): y = ax + b song song vi ng thng (d) v i qua im M(0; 2).

Cu 3: 10 m
Tm tham, s thc m phng trnh x
2
2mx + m 1 = 0 c mt nghim bng 0. Tnh nghim cn
li.

Cu 4: 10 m
Rt gn biu thc:
a a a a
A 1 1
a 1 a 1
| || |
+
= +
| |
| |
+
\ .\ .
, vi a 0, a 1 > =

Cu 5: 2 m
Cho tam gic ABC c ba gc nhn ni tip trong ng trn tm O. Gi AH v BK ln lt l cc
ng cao ca tam gic ABC.
a) Chng minh t gic AKHB ni tip ng trn. Xc nh tm ca ng trn ny
b) Gi (d) l tip tuyn vi ng trn (O) ti C. Chng minh rng ABH HKC = v HK OC .

Cu 6: 1 m
Tnh din tch xung quanh v th tch ca mt hnh nn c ng knh ng trn y d = 24 (cm) v
di ng sinh 20 = (cm).















CHNH THC

177

S GIO DC V O TO K THI TUYN SINH VO LP 10 THPT
TNH RA-VNG TU Nm c 2012 2013

MN THI: TON
Ngy thi: 05 thng 7 nm 2012
(Thi gian lm bi: 120 pht, khng k thi gian giao )
Bi 1: 30 m
a) Rt gn biu thc: A = 5 3 2 48 300 +
b) Gii phng trnh: x
2
+ 8x 9 = 0
c) Gii h phng trnh:
21
2 9
x y
x y
=

+ =


Bi 2: 15 m Cho parabol (P): y =
1
4
x
2
v ng thng (d): y =
1
2
x + 2
a) V (P) v (d) trn cng mt mt phng ta
b) Tm ta giao im ca (P) v (d) bng php tnh.
Bi 3: 15 m
Hai i cng nhn cng lm mt cng vic. Nu hai i lm chung th hon thnh sau 12 ngy. Nu
mi i lm ring th di mt s hon thnh cng vic nhanh hn i hai l 7 ngy. Hi nu lm ring th
mi i phi lm trong bao nhiu ngy hon thnh cng vic ?

Bi 4: 35 m
Cho ng trn (O) ng knh AB. V tip tuyn Ax vi ng trn (O). Trn Ax ly im M sao
cho AM > AB, MB ct (O) ti N (N khc B). Qua trung im P ca on AM, dng ng thng vung gc
vi AM ct BM ti Q.
a) Chng minh t gic APQN ni tip ng trn.
b) Gi C l im trn cung ln NB ca ng trn (O) (C khc N v C khc B).
Chng minh: BCN OQN =
c) Chng minh PN l tip tuyn ca ng trn (O).
d) Gi s ng trn ni tip ANP A c di ng knh bng di on OA.
Tnh gi tr ca
AM
AB


Bi 5: 05 m
Cho phng trnh
( )
2 2
2 1 1 0 x m x m m + = (m l tham s). Khi phng trnh trn c nghim
1 2
, x x
, tm gi tr nh nht ca biu thc: ( ) ( )
2 2
1 2
1 1 M x x m = + +

CHNH THC

178
p n b n

a) T gic APQN c
o o
APQ ANQ 90 APQ ANQ 180 = = + =
b) Ta c PA = PM v PQ AM QM = QB OQ // AM OQ AB
OQN NAB = (cng ph vi ABN)
BCN NAB = (cng chn NB)
BCN OQN =
c) Cch 1: OQN NAB = t gic AONQ ni tip.
Kt hp cu a suy ra 5 im A, O, N, Q, P cng nm trn mt ng trn
o
ONP OAP 90 ON NP = = NP l tip tuyn ca (O)
Cch 2: PAN PNA = (do APAN cn ti P)
ONB OBN = (do AONB cn ti O)
Nhng PAN OBN = (cng ph vi NAB)
PNA ONB =
M
o o
ONB ONA 90 PNA ONA 90 PNO ON PN + = + = = NP l tip tuyn ca (O)
d) Gi I l giao im ca PO v (O), suy ra I l tm ng trn ni tip tam gic APN
R
OE EI
2
= = (R l bn knh ng trn (O)) AIE A u
3
AE R
2
=
AEO A PAO A (g-g)
R 3
AE EO 2PA MA AE
2
3
R
PA AO 2AO AB EO
2
= = = = =










179
S GIO DC V O TO THI TUYN SINH LP 10 THPT
TNH HU GIANG NM HC 2012 2013
MN: TON
Thi gian lm bi: 120 pht (khng k thi gian giao )
thi c 01 trang
Bi 1: (0,5 im) Rt gn biu thc:
2 1
8 2
2 1
6 3
+
+
+

= A

Bi 2: (1,5 im) Khng s dng my tnh cm tay, hy gii phng trnh v h phng trnh sau:
a) 0 20
2
= + x x b)

= +
=
1 2
5 2
y x
y x


Bi 3: (2,0 im)
a) V th (P) ca hm s: y = -2x
2

b) Tm to cc giao im ca (P) v ng thng (D): y = x 1 bng php tnh.

Bi 4: (2,0 im) Cho phng trnh ( ) 0 3 1 2
2
= + m x m x (m l tham s)
a) Chng minh phng trnh lun c 2 nghim phn bit.
b) Gi hai nghim ca phng trnh l
2 1
, x x . Xc nh m gi tr ca biu thc
2
2
2
1
x x A + = nh
nht

Bi 5: (4,0 im) Cho ng trn (O; R) v mt im S bn ngoi ng trn v hai tip tuyn SA, SB v
ng thng a i qua S ct ng trn (O; R) ti M, N vi M nm gia S v N (ng thng a khng i qua
tm O).
a) Chng minh SOAB
b) Gi I l trung im ca MN v H l giao im ca SO v AB; hai ng thng OI v AB ct nhau ti
E. Chng minh: OI.OE = R
2

c) Chng minh t gic SHIE ni tip ng trn
d) Cho SO = 2R v MN = R 3 . Tnh din tch tam gic ESM theo R




















CHNH THC

180

S GIO DC V O TO
N TRE

THI TUYN SINH 10
TRUNG HC PH THNG CHUYN N TRE NM HC 2012 2013
MN TON (chung)
T gan 120 pt kng k pt
Cu 1 (2,0 im). Khng dng my tnh b ti, hy rt gn cc biu thc sau:
a) A =
( )
2 3
6 5
5 3 6 3
| |
+
|
+
\ .

b) B =
2x x x 1 x x 1
x x 1 x x 1


+ + +
, (vi x > 0)

Cu 2 (2,5 im). Gii phng trnh v h phng trnh sau:
a)
( ) ( )
2
2 2
x x 1 3 x x 1 4 0 + + =
b)
2 6
11
x y
4 9
1
x y

+ =



Cu 3 (2,5 im).
a) Chng minh rng phng trnh
2
2 3 8 0 x mx m + = lun c hai nghim phn bit x
1
; x
2
vi mi
m. Vi gi tr no ca m th hai nghim x
1
; x
2
tha mn
( )( )
1 2
x 2 x 2 0 <
b) Cho x, y, z l ba s thc dng tha:
2 2 2
x y z 1 + + = . Chng minh rng:
3 3 3
2 2 2 2 2 2
1 1 1 x y z
3
x y y z z x 2xyz
+ +
+ + s +
+ + +

ng thc xy ra khi no?

Cu 4 (3,0 im). Cho na ng trn tm O ng knh AB. T A, B v cc tip tuyn Ax, By v pha c
cha na ng trn (O). Ly im M thuc on thng OA; im N thuc na ng trn (O). ng trn
(O) ngoi tip tam gic AMN ct Ax ti C; ng thng CN ct By ti D.
a) Chng minh t gic BMND ni tip.
b) Chng minh DM l tip tuyn ca ng trn (O).
3/ Gi I l giao im ca AN v CM; K l giao im ca BN v DM. Chng minh IK song song AB.











N T

181

S GIO DC V O TO THI TUYN SINH 10
N TRE TRUNG HC PH THNG CHUYN N TRE
NM HC 2012 2013
MN TON CHUYN
T gan 120 pt kng k pt


Bi 1: 3 m
Cho biu thc
( )
x 8 x 1
A x 2 :
x x 8 x 2 x 4 2 x
(
+
= + +
(
+ + +

vi x >0
1/ Rt gn biu thc A.
2/ t
8
B x
x 6 A
= +
+
. Tm x biu thc B t gi tr nh nht
Bi 2:
Gii cc phng trnh v h phng trnh sau
1/
2 2
2x 8x x 4x 16 4 + + =
2/
( )
2 3
3 x 2 10 x 1 + = +
3/
2x y xy 13
1 1
15 2
x 1 y 2
=

| |

+ =
|

+
\ .


Bi 3:
1/ Xc nh tt c cc gi tr ca m phng trnh
2
x 2x 2m 5 0 + = c hai nghim phn bit
x
1
; x
2
. Vi gi tr no ca m th hai nghim x
1
; x
2
tha iu kin
( )( )
1 2 2 1
x mx x mx 10 =
2/ Cho ba s thc dng a, b, c. Chng minh rng
2 2 2
a b c a b c
b 3c c 3a a 3b 4
+ +
+ + >
+ + +

Bi 4:
Cho tam gic ABC nhn, v ng cao AH. Gi E, F ln lt l hnh chiu ca H ln hai cnh AB,
AC. ng thng qua A vung gc vi EF ct cnh BC ti D.
1/ Chng minh ng thng AD i qua tm ng trn ngoi tip ca tam gic ABC.
2/ Gi I, K ln lt l hnh chiu ca D ln hai cnh AB, AC. Chng minh tam gic DIK ng dng
vi tam gic HEF.
3/ Chng minh
2
2
BH BD AB
.
CD CH AC
=









N T

182








183


184


185

186








187
S GIO DC & O TO
AN GIANG
---------------
CHNH THC


SBDPHNG..
THI TUYN SINH VO LP 10
Nm c 2012-2013
--------------------
Mn: TON
Kha ngy 11 -7 -2012
T gan lm b : 120 pt
Kng k t gan pt
Ngy t: 12-7-2012


1. 25 m

a) Rt gn A = + 2 16 - 6 9 36
b) Gii phng trnh bc hai : x
2
2 2 x +1 = 0
c) Gii h phng trnh :
3 7
2 3
x y
x y
=

+ =


2. 20 m

Cho hm s y = x + 1 (*) c th l ng thng ( d )
a) Tm h s gc v v th hm s (*)
b) Tm a (P): y = ax
2
i qua im M (1 ;2).Xc nh ta giao im ca ng thng (d) v Parabol (P)
vi a va tm c .

3. 20 m

Cho phng trnh x
2
2 (m+1) x + m
2
+ 3 = 0
a) Vi gi tr no ca m th phng trnh c hai nghim phn bit.
b) Tm m phng trnh c hai nghim tha tch hai nghim khng ln hn tng hai nghim.

4. 35 m

Cho ng trn ( O) bn knh R = 3 cm v mt im I nm ngoi ng trn, bit rng OI =
4cm.T I k hai tip tuyn IA v IB vi ng trn (A,B l tip im).
a) Chng minh t gic OAIB ni tip.
b)T I k ng thng vung gc vi OI ct tia OA ti O.Tnh OO v din tch tam gic IOO .
c) T O k OC vung gc BI ct ng thng BI ti C.Chng minh OI l tia phn gic ca AO'C

------ Ht------










188

189

190

























191












192








193





194





195




196



























197















198







199

You might also like